Urologia PDF

Descargar como pdf o txt
Descargar como pdf o txt
Está en la página 1de 63

.

- ♦
CTO
""""""

Libro CTO
de Medicina y Cirugía
,
Indice
01. semiología urológica ........ ......... ·················· .. ···· ............... 1 09. Disfunción eréctil ............... ................ ......................... .. .... .....47
1.1 . Repaso anatómico básico ....... . ... 1 9. 1. Introducción .. .......................................................... ........... ................... .... 47
1.2. Definiciones........................................................................................................ 1 9.2. Prevalencia... .. .... .. ... . .. . .. .. .. ................... .. .......................47
1.3. Diagnóstico diferencial de la hematuria macroscópica .......2 9 .3. Etiología..................... ................ ...................... .. ....................47
9 .4. Factores de riesgo ......... . .. . 47
9.5. Diagnóstico ............................. .................................. . ... 48
02. Infecciones de las vías urinarias ..... .................. .............. ...... 4 9.6. Tratamiento ................... ................ . .......... 48
2.1. Patogénesis y etiología .. . . . . .. . . . ....................... 4
2.2. Diagnóstico ...................................................................................... ..............4
2.3. Tipos de 1vu y su tratamiento... ............................. 5
1o. Infertilidad masculina . .................................................. .................... 50
2.4. Gangrena de Fournier ......... ...... .. ........... .. . ................ .7 10.1. Generalidades ................................ .............................................................. 50
2.5. Tuberculosis genitourinaria ......... ......... ........... .. 8 10.2. Clínica............. .. . .. .... . .... . . ............... .......... ...................... 50
2.6. Cistitis intersticial.... .. . ................ ......... .......... . . .. .... . . ........... 9 10.3. Diagnóstico .. ................ .... ........... .. ........................ ..... ... ............ 50
10.4. Tratamiento .......................... . . 51

03. Urolitiasis .......... .............................................................................. . ................... 12


3.1. Epidemiología........ ........................................................ . . .. .. ..... ...... . 12
11. Trasplante renal ................ .... .. .. ..................................................... 53
3.2. Manifestaciones clínicas y su manejo agudo ..... . 11 .1. Indicaciones ............. ....... ... ....... . ... ... .. .. ...... ... . .... .. .. 53
3.3. Evaluación y tratamiento de la litiasis renal... .......................... .. 14 ( 11.2. contraindicaciones ... .... . .. 53
11 .3. Complicaciones . .. .. . . .. .. . .... ... . ..... 53
11.4. lnmunosupresión en trasplante ... . .... 54
04. Hiperplasia y carcinoma prostático .................. ............20

::;: .~~.'.:~~:: : ::::. ::::::::::: :::::::::: : : : ~~ ¡12.


~!~;~:~~ i;i:!:i~; ~ Traumatismos del aparato genitourinario ..... . ..... ss
12.1. Lesiones del riñón ....... ...... 55
12.2. Lesiones del uréter......... ..... 56
05. Tumores renales ..................................................................................... 30 i 12.3. Lesiones de la vejiga... . ...... 56
5.1. Carcinoma de células renales 12.4. Lesionesdela uretra ......................... ................................. .. 57
(adenocarcinoma renal, hipernefroma) . ...........30 ( 12.5. Lesiones del pene .......... ......................... .... ........... 57
5.2. Otros tumores ... .. 32 : 12.6. Lesiones de los testículos .............................. . . ..................... 57

06. Tumores testiculares ................................................ . .... ..... ...... ... 34 j Recommended reading 1......................... .................. ............................... 59
6.1. Etiología y epidemiología ..................... ................................. ................... 34 ¡
6.2. Anatomía patológica .... ............. 34 i R dd d.
. . ............. :. ecommen e rea ing 2 .................... ............................... .......................... 60
6.3. el 1n1ca.......... ...... . 35
6 .4. Diagnóstico .. . .. . ....... ....... ...................... . 35 :
6.5.
6.6. ~:=~~~t~~: 1
~i~~renci~ ·: .................. : :::::: :::: : :::::::::::::::::::: ~: j Solucionario ......................... ............................. . . ......................................... ......... ....61

07. carcinomas del tracto urinario ............................................40 ¡ Bibliografía ................................................. ......... ....................................................... ......62
7.1. Carcinoma vesical......................................... . ..................... 40 :
7.2. Tumores del tracto urinario superior...... ·••··············· 42 :

oa. uropatía obstructiva ................................................................ ........... 45 \


8.1. Características ................................ ............ ............ . ............................ 45 \
8.2. Clínica.. ...... ..... .... . ..... ..... ...... . ............. 45 :
8.3. Diagnóstico . ........................... .. .. .. .. .. . ..................45 \
8.4. Tratamiento .. ... 46 :


01
Semiología urológica

1.2. Definiciones
Este tema puede ayudar aobtener una visión general dela materia yaasociar algunos Hematuria microscópica. Presencia de más de 5 hematíes por campo.
conceptos básicos, pero no se debe emplear en él demasiado tiempo. La causa más frecuente en ambos sexos es la litiasis. La causa más común
en varones mayores de 50 años es la hiperplasia be nigna de próstata .
Hematuria macroscópica. Orina de aspecto roj izo a simple vista
debido a ta presencia de más de 100 hematíes por campo. En los
pacientes fumadores, en ausencia de otros síntomas, se debe sospe-
1.1. Repaso anatóm ico básico char tumor maligno de vías urinarias.
Piurla. Presencia de más de 10 leucocitos por campo. Altamente ines-
Rlilones. Órganos retroperitoneales. Medidas normales: 10 x 5 x 4 cm. pecífica, pero en presencia de síntomas urina rios, hay que sospechar
Ultrasonográficamente se divide en corteza (glomérulos y túbulos) infección.
y médula (colectores). Los sistemas colectores se componen de Síndrome miccionat. Presencia de polaquiuria (aumento en la fre-
cálices menores (8-12) que se unen para formar cálices mayores cuencia miccional), urgencia miccional (necesidad imperiosa e irrefre-
(2-3). nable de orinar) y disuria (molesti as urinarias inespecíficas referidas
Uréter. Longitud aproximada de 25-30 cm, diámetro aproximado de como ardor, escozor... ).
5-7 mm. Se extiende en una trayectoria cercana al músculo psoas . Incontinencia urinaria. Pérdidas involunta rias de ori na (Tabla 1.1 ).
En el hombre, pasa por debajo del conducto deferente; en la mujer, Existen los siguientes tipos principales:
por debajo de las arterias uterinas. Continua . De día y de noche, en todas las posiciones. La causa
Vejiga. Órgano pélvico hueco. Capacidad fisiológica : 400 mi ± SO mi. más frecuente es la fístula urinaria (en pacientes con anteceden-
Dentro del piso vesical se encuentra el trígono donde desembocan los tes quirúrgicos previos), y la segunda el uréter ectópico (causa
meatos ureterales. más frecuente en las niñas).
Uretra. En la mujer, es corta (longitud aproximada : 3-5 cm ; diámetro De esfuerzo. Se desencadena con el aumento de presión abdo-
aproximado: 8-10 mm); en el hombre, tiene una longitud aproximada m inal (al reír, toser, cargar con peso) . Generalmente, se produce
de 15-20 cm divididos en uretra posterior (prostática, membranosa) por déficit de soporte de la musculatura perineal (por ello, es
y uretra anterior (peneana) . recomendable revisar los antecedentes obstétricos, pacientes
Próstata. Glándula que forma parte del sistema reproductor mascu- obesas, pacientes añosas .. .).
lino, ubicada debajo del cuello vesical. Peso aproxim ado: 20 g, en una De urgencia . El paciente siente ganas de orinar, pero no le da
persona adulta joven. tiempo a llegar al baño (provocadas por contracciones involunta -
Testiculos. Órganos glandulares que producen esperma y hormonas rias del músculo detrusor) .
sexuales (testosterona). Las dimensiones normales pueden variar Mixta . Generalmente, es una combinación de las dos anteriores.
entre 4-8 cm de longitud y 2-4 cm de ancho. En el polo superior, se Paradójica . Escape de orina debido a la sobredistensión vesical.
localiza el epidídimo, que continúa cauda lmente formando el con- El ejemplo característico es el paciente prostatico con retención
ducto deferente. urinaria. La presión intravesical supera la presión de cierre del

Síntomas Incontinencia de urgencia Incontinencia de esfuerzo

llrqPn(1.l trllu;:('oo r 1• p P nfinn dt• orin.tr)

Aumt->nto de 1,) fr ett1l!flC1d rn1 tc io nJI SI No


~ C.ipuc1ddd de llt~q,ir <1! bdno de\pu é'> dl• •~pn tir e l de~eo d e orinar No Sí

o~,per tdl S~ f)cUit ,r cll bc1n o r:lur ,111 te lct noche sr Generalmente no

f.-.c.,p~ dur,1ntP l,1 ,1ct 1v 1dad f1 sica No Sí

C ttl t1d,1d df.• o nnJ ll\Cdp1ui.1 en cadc1 epi~od10 de incontine nc i.1 Abundante, si se produce Generalmente escasa

Tabla 1.1. Diagnóstico diferencial de la clínica de incontinencia urinaria femenina

,,,. •
Libro CTO de Medicina y Cirugía, 6.ª ed. ENARM

esfínter uretral, produciéndose un escape de orina paradójico Final. Sangrado del cuello vesical.
(no puede orinar, sin embargo, se le escapa la orina). Total. Sangrado vesical o del tracto urinario alto. Se debe recor-
Enuresis. Pérdidas de orina producidas exclusivamente durante el dar que un sangrado importante de cualquier parte del aparato
sueño. Si el niño es mayor de 6 años, debe ser estudiado. genitourinario puede provocar hematuria total.
Cólico renoureteral. Dolor lumbar frecuentemente irradiado a genita- Hematuria con coágulos. Indica un problema urológico. La causa más
les, de carácter agudo, cuya intensidad no se modifica por los cambios frecuente en mujeres es la cistitis hemorrágica, aunque la primera
posturales, y que se suele acompañar de náuseas, vómitos y malestar causa a descartar es una neoplasia urotelial, máxime en el pa ciente
general. Es muy frecuente que sea bilateral. Su semiología se desarro- fumador.
llará más a detalle en el capítulo dedicado a la litiasis. Hematuria sin coágulos. Generalmente asociada a nefropatia, puede
ir acompañada de cierto grado de proteinuria, así como de ci lindros
eritrocitarios o de hematíes dismórficos en el sedimento urinario, rara
vez etiología por patología urológica.

Los cilindros hemáticos aparecen en las glomerulonefritis que producen


síndrome nefrítico, como en la postinfecciosa.

Nemotecnia P PON T T T T H IS para recordar las causas de hematuria :


Periodo (Endometriosis)
Próstata
Obstrucción
Nefritis
Trauma

1.3. Diagnóstico diferencial Tumor


Tuberculosis
de la hematuria macroscópica Hernatológicos
Infección
Stones: Litiasis
Según el momento de aparición:
Inicial. Sangrado uretral o prostático.

✓ La causa más frecuente de hematuria microscópica es la litiasis (en la ✓ La causa más habitual de hematuria es la cistitis hemorrágica, pero lo
población general, en ambos sexos}. primero que_se ha de descartar es el tumor urotelial. Se debe descartar
patología oncológica en población adulta mayor.
✓ La causa más común de hematuria microscópica en varones de más de
SO años es la hiperplasia benigna de próstata. ✓ Los hematíes dismórficos en el sedimento orientan a nefropatía de ori-
gen glomerular.
✓ La hematuria con coágulos indica un problema urológico.

Paciente femenino de 78 años de edad, la cual acude a consulta por pre- 1) Multiparidad.
sencia de salida de orina de forma involuntaria al momento de cargar ob- 2) Tabaquismo.
jetos pesados, al estornudar y al reírse. El tipo de Incontinencia que refiere 3) Divertículo vesical.
la paciente es: 4) Cirugía de pelvis.

1) Esfuerzo. En una paciente oon presencia de fístula vesico-vaginal, qué tipo de Incon-
2) Urgencia. tinencia puede desarrollarse:
3) Rebosamiento.
4) Continua. 1) Esfuerzo.
2) Urgencias.
Es factor de riesgo para desarrollar el tipo de incontinencia de la paciente 3) Rebosamiento.
anterior, excepto: 4) Continua.


01. Semiología urológica I UR

A 7-year--old boy is brought to our clinic wlth a history of nocturia, 5-6 times A 63-year-old woman presents to the emergency department complainlng
a week. Regarding this clinical case, which of the following is false? of frequent urine leakage. She says that these leaks are more prominent
when coughlng, laughlng or sneezing. She also clalms that at times, she Is
1) lt should be considered monosymptomatic nocturnal enuresis provided unable to reach a toilet in time. lt is true that:
that the patient does not have daytime symptoms.
2) The usual clínica! course tends to resolve spontaneously. 1) Symptoms are consistent with emergency urinary incontinence and
3) Usually, these kids continue presenting urinary sph incter problems until should be treated with anticholinergic agents.
adulthood. 2) Symptoms are consistent with emergency urinary incontinence and the
4) In 60% of cases, fam ily history may be encountered. patient should initially perform pelvic floor exercises.
3) lf these exercises result ineffective, a suburethral mesh placement is
indicated.
4) In this case, pelvíc floor muscle training will not be effective .


Infecciones de las vías urinarias
02
Una vez que las bacterias han alcanzado la vía urinaria, tres factores deter-
minan el desarrollo de la infección :
Este tema es el más importantede esta asignatura. Hay que tener en cuenta la tuberculosis Virulencia del microorganismo.
genitourinaria, la cistitisintersticial, la pielonefritis ylas infecciones urtnarias durante el Tamaño del inoculo.
embarazo, temas que suelen ser preguntados yrentables al momento de ser evaluados. Mecanismos de defensa del huésped.

Hay que distinguir entre :


Infecciones en la comunidad . La mayoría están producidas por gér-
La infección de las vías urinarias (IVU) puede clasificarse de varias formas. menes gramnegativos, principalmente Escherichia coli (responsable
Se puede hacer una división anatómica entre IVU altas (infecciones rena- del 85%) y, en menor proporción, Proteus, Klebsie//o o Pseudomonas.
les) e IVU bajas (cistouretritis, prostatitis). Asimismo, la clasificación puede Entre los grampositivos, únicamente tiene relevancia Staphylococcus,
basarse en la asociación o no de complicaciones. Una IVU no complicada que produce el 10-15% de las IVU en mujeres jóvenes (segundo ger-
es un cuadro clínico caracterizado por la presencia de escozor miccional, men más frecuente en esta población).
urgencia y frecuencia, acompañado o no por hematuria terminal, dolor Infecciones nosocomiales. Los gérmenes gramnegativos continúan
hipogástrico y, más raramente, febrícula . Dentro de este grupo, se podrían siendo los más frecuentes. Si bien E. coli es el más habitual, su fre-
incluir las pielonefritis no complicadas, que se presentan como cuadros cuencia desciende hasta el 50%, y adquieren mayor importancia Pro-
febriles con hipersensibilidad en fosa lumbar, fiebre. náuseas o vómitos, y teus, Klebsiella, Pseudomonas, Enterobactery Serrana. El 25% restante
sin los factores que convie rten la IVU en "complicada", como son: presencia está ocasionado por gé rmenes grampositivos como estreptococos y
de catéteres, uropatía obstructiva, reflujo vesicoureteral, anomalías anató- estafilococos. Condido albicans puede aparecer principalmente en
micas, insuficiencia o trasplante renales . La tVU en el varón debe conside- pacientes diabéticos, cateterizados o con tratamientos antibióticos
rarse esencialmente "complicada" de entrada. prolongados.

La reaparición de una infección tras el tratamiento puede deberse a reinfec-


ción o recidiva . El primer término expresa la infección nueva por un germen
distinto al inicial, mientras que recidiva indica infección por el mismo ger-
men. Esta última es mucho más infrecuente que la re infección y puede estar Uropatógenos que requieren cultivos especiales: anaerobios, micobac-
terias, Chlamydia.
ocasionada por litiasis infectiva, prostatitis crónica, fístulas vaginales o intes-
tinales, divertículos vesicales infectados, cuerpos extraños, necrosis papilar
infectada y otras causas que generan un reservorio de microorganismos que La afectación de la vía urinaria superior parece también producirse por
difícilmente se eliminan con el antibiótico. ascenso de los gérmenes a lo largo del uréter. La diferenciación, aunque poco
específica, se debe basar en los hallazgos clínicos (fiebre, dolor lumbar, esca-
lofríos) v analítica elemental (leucocitosis, velocidad de sedimentación alta).
2.1. Patogénesis y etiología
Existen tres posibles vías por las que los microrganismos pueden alcan- 2.2. Diagnóstico
zar la vía urinaria : hematógena, linfática y ascendente. La vía linfática
carece de im portancia real, y la diseminación hematógena tampoco es El diagnóstico de IVU. además de por la clínica, se define por el cultivo de
frecuente . La más común es la ascendente in ici ada en la uretra . Proba- orina. Dado que es frecuente el crecimiento de bacterias que han contami-
blemente por esta razón es mucho más habitual la IVU en mujeres, dado nado las muestras, se utiliza un criterio estadístico sobre la base del recuento
que su uretra es muy corta y ancha, y ello favorece el paso de microorga- de colonias del urocultivo, considerando como significativo, clásicamente, el
nismos hacia niveles más altos del tracto genitourinario (TGU). Otro dato crecimiento de más de 105 colonias por mililitro. En determinadas circuns-
que apoya la importancia de la vía ascendente es la frecuencia de infec- tancias, recuentos menores de colonias pueden ser suficientes: 10 3 UFC/ml
ción tras el cateterismo uretral , que es del 1% en los pacientes ambulan- en mujeres sintomáticas, más de 10' en pielonefritis clínicas o en varones,
tes, y en 3-4 días alcanza a casi la totalidad de los pacientes sondados co n y más de 102 en muestras de cateterismos limpios o cualquier recuento,
sistemas de drenaje abiertos. En los pacientes hospitalizad os, el riesgo si se recoge mediante punción-aspiración suprapúbica. Cifras mayores de
de infección alcanza un 5% por cada día de sondaje, incluso con sistemas 105 UFC/ml pueden igualmente reflejar contaminación, principalmente si
cerrados . crec en dos o rnás esµ~cies .


02. Infecciones de las vías urinarias I UR

En el adulto, la presencia de piuria (más de 10 leucocitos/mm 3) se relaciona nales de la vía urinaria . En estos casos, si la clínica es muy sugestiva (disuria,
estrechamente con la tVU en presencia de síntomas; no así en el niño, en el polaquiuria, urgencia urinaria). se puede iniciar tratamiento empírico. Si el
que puede acompañar a tos cuadros febriles. diagnóstico es dudoso, se puede corroborar con análisis de orina con tira
reactiva . En caso de requerirse un cultivo (indicado ante la sospecha de pie-
lonefritis, síntomas que no se resuelven, síntomas atípicos, embarazadas),
2.3. Tipos de IVU y su tratamiento recuentos de 103 UFC/ml son suficientes para el diagnóstico. Las pautas cor-
tas de tratamiento (3 dias) son las de elección para este grupo de pacientes.
Lógicamente, en el tratamiento de la IVU es fundamental et empleo de anti- En el tratamiento de primera línea, se encuentran nitrofurantoína, fosfomi-
microbianos. El número empleado de éstos es elevado y las pautas de trata- cina, pivmecilinam y trimetoprim/sulfametoxazol (este úl ti mo sólo se debe
miento muy variables. A continuación, se repasan las opciones terapéuticas considerar en sitios donde las tasas de resistencia de E. cofi son menores al
según el tipo de IVU. 20%). Las fluoroquinolonas no están consideradas como primera opción.

Bacteriuria asintomática IVU complicada


La bacteriuria asintomática se define como bacteriuria significativa (10' Se consideran IVU complicadas las relacionadas con los sigu ientes
UFC/ml) en, al menos, dos urocultivos con el mismo germen, tomados con supuestos:
una semana de diferencia en ausencia de síntomas. La bacteriuria asinto- Embarazadas. Aunque las pautas cortas de tratamiento pueden con-
mática no debe tratarse salvo en los casos en los que conlleva un riesgo de siderarse, actualmente aún son recomendados los esquemas conven-
infección clínica o daño orgán ico, como ocurre en niños menores de 5 años, cionales de antibioticoterapia de 7 días. Los fármacos que pueden
tengan o no patología urológica asociada. Asimismo, debe ser tratada en el considerarse para estos pacientes son: penicilinas o sus derivados,
embarazo y como profilaxis previa a una cirugía urológica (Tabla 2.1 }. cefalosporinas, fosfom icina, nitrofurantoína (excepto en deficiencia
Pacientes sin factores de riesgo. La bacteriuria asintomática no pre- de glucosa-6-fosfato deshidrogenasa y en el último trimestre), trime-
senta riesgo de daño rena l, no requiere screening ni tratamiento. toprim (no en el primer trimestre del embarazo) y sulfam idas (no en el
Pacientes con bacteriuria asintomática e infecciones recurrentes. tercer trimestre del embarazo) .
No es recomendable tratar si no hay síntomas; sin embargo, se deben Hombres. Generalmente, existe un factor predisponente y el más
buscar factores predisponentes. En varones, se debe sospechar pros- común de todos es el relacionado con la próstata, por lo que los tra-
tatitis crónica como primera opción. tamientos deben ir enfocados en fármacos que penetren dentro del
Embarazadas. La bactj!riuria asintomática se correlaciona con_epi- tejido prostático. Se recomiendan esquemas de 7 días con fluoroqui-
sodios de pielonefritis, así como con parto pretérmino, por lo que el nolonas o trimetoprim/sulfametoxazol.
screening y el tratam iento están recomendados en estas pacientes. El Pacientes con enfermedad renal c;rónica. La elección del antibiótico
esquema que puede emplearse es amoxicllina sola o en combinación suele estar influenciada por la tasa de filtrado glomerular (TFG) del
con nitrofurantoína por 7 días. paciente. Sin embargo, muchos fármacos tienen índices terapéuticos
Situaciones especiales: en diabetes mellitus controlada no se reco- bajos que no requieren modificar su dosis, excepto cuando la TFG es
mienda tratamiento, en la mal controlada se puede considerar tra- menor de 20 ml/min o los fármacos con nefrotoxicidad comprobada
tamiento por aumento de incidencia de IVU complicada; vejiga (p. ej., am inoglucósidos).
neurogénica; derivaciones urinarias como conductos ileales; lesión
espinal; catéter urinario; o nefrostomías, frecuentemente están colo- IVU recurrente
nizados, por lo que no requieren tratamiento.
La IVU recurrente se define como la aparición de tres o más cuadros de
lnd1cac1ones de trat.im,ento IVU en 12 meses, o de dos cuadros en 6 meses. Se puede realizar profi-
laxis con trimetoprim/sulfametoxazol o una fluoroquinolona (en función
Menores de cinco aros
Embarazadas de la sensibilidad del germen aislado en el último episodio) en dosis única,
Previamente a cirugía urológica días alternos, durante 6 meses. Si tras la retirada se presentaran nuevas
Tabla 2.1. Bacteriuria asintomática: indicaciones de tratamiento recurrencias, puede reinstaurarse el tratamiento durante periodos más
prolongados (1-2 años). Es aconsejable la ingesta abundante de agua y
realizar micciones frecuentes y cumplir una serie de reglas básicas higié-
nico-dietéticas.

Si los episodios tienen relación con el coito, se puede administrar un compri-


Proteus es intrínsecamente resistente a la nitrofurantofna, ya que alcali- mido de trimetoprim/sulfametoxazol o una quinolona después del mismo.
niza la orina gracias a su ureasa, y este grupo de antibióticos únicamen-
En mujeres posmenopáusicas, el tratamiento con estrógenos tópicos vagi-
te es útil en medio ácido.
nales disminuye la frecuencia de infecciones.

IVU no complicada
La IVU no complicada se define como la infección esporádica y aguda, ya Indicaciones para solicitar urocultivo: sospecha de pielonefritis, sínto-
sea cistitis o pielonefritis, limitada a pacientes del sexo femenino, mujeres mas que no se resuelven, síntomas atípicos, embarazadas.
premenopáusicas no embarazadas, sin alteraciones anatómicas ni funcio-


Libro CTO de Medicina y Cirugía, 6.ª ed. ENARM

Pielonefritis aguda
El cuadro clínico se caracteriza por dolor en flanco, náusea o vómito, fiebre y
sensibilidad en ángulos costodiafragmáticos. En el diagnóstico, el uroanálisis El antígeno prostático específico puede elevarse en presencia de prostati-
tis; y puede tardar hasta 3 meses en normalizarse posterior al tratamiento.
y el urocultivo deben ser indicados de rutina; asimismo, como estudio de ima-
gen inicial, se realiza USG de la via urinaria superior buscando obstrucción del
flujo urinario. La tomografía computarizada (TC) simple, el urograma excretor La prostatitis crónica bacteriana suele presentarse como molestias perinea-
o el gammagrama renal deben considerarse según la evolución del cuadro. les o genitales, síntomas irritativos (polaquiuria, tenesmo, escozor) y epi-
sodios de IVU recurrentes causados por el mismo organ ismo. En el líquido
En los casos de gravedad leve-moderada, puede plantearse terapéutica oral prostático se evidencian más de 10 leucocitos por campo de gran aumento,
con trimetoprim/sulfametoxazol (en desuso en nuestro medio por el elevado y macrófagos que contienen cuerpos ovales grasos. El tratam ien o de e
índice de resistenci as). fluoroquinolonas o ~-lactámicos. En pacientes graves estar guiado por los cu ltivos, tanto de orina como de fluido obtenid o por
u hospitalizados, es preciso tratamiento parenteral, y el espectro de antimi- masaje prostático, y prolongarse entre 4-16 semanas.
crobianos incluye ampicilina (enterococo) , ureidopenicilinas (Pseudomonos).
cefalosporinas de segunda o tercera generación, e incluso aminoglucósidos. El síndrome doloroso pelviano crónico (SDPC) hace referencia a moles tias
Nunca se emplearán pautas corta s. La duración del tratamiento debe ser por genitourinarias de más de 3 meses de evolución con cultivos habituales
14 días, vía parenteral u oral, y la respuesta clínica se valora a las 48-72 horas. negativos. Presenta etiopatogenia multifactorial : infecciosa por gérme es
atípicos (ureaplasma, micoplasma , Chlamydio .. .}, psicológica (estrés), neu -
En varones, siempre se debe sospechar de factor predisponente, siendo el rológica (similar a fibromialgia y colon irritable). El tratamiento incluye anti-
más frecuente la obstrucción por crecimiento prostático. bióticos, antiinflamatorios, antidepresivos y rehabilitación del suelo pélvico.

Prostatitis Se denomina prostatodlnla a un cuadro clínico similar en el que predomi-


nan las molestias perineales o genitales con cultivos negativos y menos de
La infección aguda del tejido prostático se presenta como un cuadro séptico 10 leucocitos por campo en el líquido prostático. Su causa es desconocida y
con afectación general del paciente, fiebre elevada, síndrome miccional , el tratamiento difícil, empleándose actualmente Cl-bloqueadores o relajan-
artromialgias y dificultad micciona l (Tabla 2.2). En el examen rectal, la prós- tes musculares como terapia inicial.
tata aparece muy dolorosa e inflamada. El germen más habitual es f. coli.
Durante la inflamación aguda, los antibióticos penetran adecuadamente,_pero Orqu iepididimitis
una vez que ésta cede, la penet ración es más pobre. Por ello, se deben utilizar
cursos largos de t ratamiento (3-4 semanas) para intentar evitar la persisten- En varones menores de 3S años, se considera, en el plano teórico, una enfer-
cia de focos que den pie a una prostatitis crónica . Entre los antimicrobianos medad de transmisión sexual, y los agentes más frecuentes son Chlamydio
empleados, están aminoglucósidos, cefalosporinas de tercera generación, trachomatis y Neísseria gonorrhoeoe. El tratamiento es con ceftriaxona 125-
fluoroqu inolonas (las que mejor difunden el tejido prostático) y el trimeto- 250 mg en dosis única intramuscular, o azitromicina 1 g dosis única vla oral,
prim/sulfametoxazol (en población en la que la resistencia es menor del 20%). o doxiciclina oral durante 7-10 días (Tabla 2 .3). En mayores de 35 años, el

Irritativo con + ± > 1OLeucocitos/campo + Amlnoglucósidos, cefalosporinas 3G,


reagudizaciones, fluoroquinolonas, 6-12 semanas
sin fiebre ni leucocitosis
Dolor genitourinario Tipo A:. inílamatoria: Antibióticos (gérmenes atlplcos)
> 3 meses > 1Oleucocitos/campo Rehabilitación suelo pélvico
Tipo 8: inflamatoria: Antidepresivos
< 10 leucocitos/campo
Tabla 2.2 . Diagnóstico diferencial de las prostatitis

-
Asintomática (~)
Exudación uretral matutina (d'l espectinomicina
~
Epldidimoprostatitis, salpingitls, síndrome
Fitz-Hugh-Curtis, gonococemla diseminada
Cultivo en medio de Thayer-Martin (no en faríngeas)
.,,,,,
_,,,
Ciprofloxacino
(déficit c,-C,, menstruación, embarazo,
auxotipo AHU)
Chlamydla rrachomatis, Similar a las UG, pero con menos signos y Contacto 7-15 días- Excluir gonorrea Tetraciclinas o macrólidos
Ureaplasma urealyricum síntomas por Gram y cultivo.

Tabla 2.3. Diagnóstico diferencial de las uretritis


Epididimltis, proctitis, cervicitis, EIP C. inclusión•Giemsa IFD, medios celulares

_,,
~

• ~
- --1!
02. Infecciones de las vías urinarias I UR

patógeno más frecuente es E. coli y el tratamiento sería con quinolonas o Pielonefritis enfisematosa
cefalosporinas 10-14 días.
Es un cuadro infrecuente pero muy grave, con una mortalidad del 9-40%,
Absceso renal presente de forma exclusiva en el paciente diabético. El microorganismo
causante más habitual es E. co/i. Aparecen áreas de necrosis y gas en parén-
Los abscesos medulares o corticales suelen proceder de un foco de pie- quima renal. El hallazgo radiológico típico es la presencia de gas intrarrenal
lonefritis contiguo o de diseminación hematógena de 5tophylococcus o perirrenal en la TC. la resolución del cuadro requiere tratamiento antibió-
aureus, procedente de focos cutáneos en sujetos adictos a drogas por tico intensivo y drenaje quirúrgico o nefrectomía.
vía parenteral. El urocu ltivo, en este último caso, puede ser negativo.
El diagnóstico más fiable se rea liza mediante TC. Deben tratarse con Pielonefritis xantogranulomatosa
antibióticos por vía intravenosa y, dependiendo del tamaño y de la evo-
lución , se hace obligatorio el drenaje mediante punción percutánea o Es una forma de presentación poco común de la pielonefritis bacteriana
quirúrgicamente. crónica, se debe a infecciones recurrentes complicadas o uropatía obstruc-
tiva. Los síntomas clásicos son: dolor en flanco, fiebre, malestar general,
Absceso perirrenal hiporexia y pérdida de peso. La TC es la técnica de elección y evidencia un
parénquima desestructurado. En la histología se observan macrófagos car-
Se localiza entre la cápsula renal y la fascia de Gerota. Lo más frecuente gados de lipidos (células xatomatosas) .
es que un absceso cort ical se abra a este espacio, pero puede ocurrir
también por diseminación hematógena. El germen más frecuente es E.
coli, y 5. aureus en los casos de diseminación hematógena (Figura 2.1 ).
El diagnóstico es similar al absceso renal, y su tratamiento pasa por el
drenaje percutáneo o quirúrgico, con la adecuada cobertura antibió - La infección de vias urinarias es la infección nosocomial más frecuen -
te, y la segunda causa más frecuente de fiebre en pacientes posqui-
tica .
rúrgi cos .

IVU asociada a catéteres


La IVU constituye la infección hospitalaria más frecuente, y los catéteres
urinarios son la principal fuente de sepsis. Se calcula que el 1% de cateteris-
mos ambulatorios transitorios sufren una IVU posterior, y que la mayorla de
pacientes con catéter permanente presentan una bacteriuria significativa al
cuarto día de su colocación. Esta bacteriuria puede hacerse sintomática en
forma de cuadros de cistitis, hematuria o episodios febriles, muchas veces
autolimitados. Entre los factores que aumentan el riesgo de IVU asociada a
catéter urinario se pueden enumerar:
Sexo femenino.
Edad avanzada.
Mala técnica de sondaje.
Sistemas de drenaje abiertos.
Falta de higiene local.

Entre los antibióticos disponibles, parece que las quinolonas son los que
mejor eliminan la película biológica de los catéteres infectados, favore-
ciendo así el tratamiento de la infección; en cualquier caso, éste únicamente
se recomienda si existe sintomatología o en el momento de la retirada del
catéter, por el mayor riesgo de IVU sintomática y sepsis.

2.4. Gangrena de Fournier


La gangrena de Fournier es una infección agresiva polimicrobiana del peri-
neo, región perianal y genitales externos. Es una subclasificación de la fas-
citis necrotizante. Los principales factores de riesgo son inmunosupresión
Figura 2.1 . TC en la que se muestra un absceso perirrenal (lo más común, por diabetes mellitus), cirugía uretral o perianal reciente,


Libro CTO de Medicina y Cirugía, 6.ª ed. ENARM

e índice de masa corporal (IMC) elevado. El diagnóstico es eminentemente Tras la inhalación del bacilo, se produce una diseminación hematógena
clínico, y se caracteriza por inflamación dolorosa del área perineal o escrotal (primoinfección) con siembra de bacilos en ambos riñones en el 90% de
acompañada de sepsis. A la exploración, hay 20nas necróticas, edematiza- los casos . Sin embargo, la enfermedad clínica generalmente es unilateral. El
das y crepitantes con salida de secreción purulenta fétida (F ig ura 2.2). periodo de latencia entre la "siembra" y la enfermedad clínica oscila entre
10-40 años, y afecta principalmente a pacientes por debajo de los 50 años.
La lesión inicial microscópica se localiza en los glomérulos en forma de gra-
nulomas. microscópicos. Al avanzar la enfermedad, se produce afectación
más distal hasta la aparición de una papilitis necrotizante, momento en el
que ya puede existir paso de bacilos a la vía excretora, donde, por procesos
inflamatorios, ocasionará estenosis a nivel de los infundfbulos caliciales, pel-
vis y uréter, con hidronefrosis secundaria. Las lesiones renales pueden cavi-
tarse y calcificarse, y llegar a producir una destrucción total del parénquima
(fenómeno que se denomina " riñón masticH).

Clínica
Los hallazgos clínicos son escasos. En el 70% de los pacientes, los síntomas
son leves. Lo más frecuente es la aparición de mícrohematuría, dolor vago
en flanco o cólico renal. La afectación vesical, sin embargo, sí produce
sintomatología florida con un síndrome cistítico rebelde, donde la pola-
Figura 2.2. Gangrena de Fournier. Imagen donada de la colección
privada del Dr. Felipe de Jesús Medina Toscano quiuria (secundaría a la disminución de la capacidad ves ical) es lo más
llamativo. En varones, es frecuente la aparición de orquiepidídimitis cró-
El tratamiento consiste en desbridación quirúrgica en las primeras 24 horas nica que no responde a la terap ia habitual. En el 90% de los pacientes, el
del diagnóstico para reducir la mortalidad, y antibioticoterapia que incluya análisis urinario es anormal. Típicamente aparece plurla ácida con uro-
cefalosporina de tercera generación, aminoglucósido o metronidazol y clin- cultivo negativo. la prueba de laboratorio más importante es el cultivo
damicina . de Mycobacterium tuberculosis en medio selectivo (Lowenstein-Jensen),
ya que los medios de tinción rápida (Ziehl-Neelsen, auramina), aunque
válidos, pueden dar falsos positivos por contaminación con Mycobocte-
2.5. Tuberculosis genitourinaria rium smegmotis.

La tuberculosis genitourinaria está ocasionada, generalmente, por Myco- Diagnóstico


bacterium tuberculosis. El aparato genitourinario es el sitio más frecuente
de afectación extrapulmonar (tras la adenitis tuberculosa). Un 5% de los El cultivo en medio de Lowenstein-Jensen es positivo en el 90% de los
pacientes con tuberculosis activa presentan afectación del tracto genitou- pacientes con enfermedad activa, aunque deben obtenerse, al menos,
rinario (Figura 2.3). tres muestras de días diferentes para mejorar la sensibilidad, ya que
el paso de bacilos a orina no es constante. Además, el resultado tarda
entre 1-3 meses. Actualmente, lo más rentable es realizar una PCR de
Obliteración
ureteral orina en busca del ARN del bacilo. Hasta el 80% tiene pruebas cutáneas
positivas. Radiológicamente, el 90% de los pacientes presentan urogra-
mas alterados. El hallazgo más sugestivo es la presencia de cavidades
que comunican con el sistema colector. Inicialmente, estas cavidades
son mínimas y dan un aspecto "mordisqueado o arrosariado" a los cáli -
ces. Según la enfermedad avanza, pueden encontrarse estenosis infun-
dlbulares, ureteropiélicas, en unión ureterovesical o vejigas pequeñas de
aspecto rígido . En el punto más evolucionado de la enfermedad, el riñón
puede encontrarse anulado, disminuido de tamaño y con calcificaciones
parenquimatosas.
Microvejiga
Tratamiento
El tratamiento médico de la enfermedad activa no difiere sustancialmente
del de la tuberculosis pulmonar en cuanto a fármacos y periodo de trata-
miento. Puede ser necesario el tratamiento quirúrgico, dependiendo de
la complicación asociada, generalmente estenosis de la vía excretora e
hidronefrosis. El esquema actual de la Organización Mundial de la Salud
Estenosis
ureteral distal \• ~ Epldidimltis (OMS) consiste en rifampicina + ísoniacida + pirazinamida + etambutol por
2 meses, seguido de rífampicina e isonlacida por 4 meses. Sí hay factores de
_riesgo, se extiende el tratamiento por 9-12 meses (recurrencia, VIH, ínmu-
Figura 2.3 . Lesiones de la tuberculosis geni tourinaria nosupresión) .


r:
~
02. Infecciones de las vías urinarias I UR

Diagnóstico

~
,,,. Nemotecnia para tratamiento: PERI (plraz.inamida, etambutol, rifampi-
El diagnóstico es, básicamente, por exclusión de otra patología que pueda

..,,,.
ocasionar un cuadro similar (infección bacteriana, tuberculosis, litiasis o
cina e isoniacida) . . tumor vesical), apoyado en los hallazgos cistoscópicos sugestivos:
Petequias submucosas, principalmente trigonales, que aparecen al
distender la vejiga (glomerulaciones) .
2.6. Cistitis intersticial • Úlceras de Hunner.

,,,. Aunque no es un cuadro infeccioso, se incluye en el presente capítulo esta La biopsia vesical, además de descartar la presencia de carcinoma in situ

...
entidad inflamatoria vesical de origen desconocido. En este sentido, se u otra patología, revela en algunos casos un infiltrado intersticial de mas-
esgrimen dos teorías no demostradas: la autoinmunitaria y la de un déficit tocitos .
en el recubrimiento urotelial por glucosaminoglucanos .
Tratamiento
Clínica

•... Aunque esta enfermedad raramente supone una amenaza para la vida de la

..
Suele presentarse en mujeres entre 30-70 años, como un cuadro cistítico paciente, su morbilidad es elevada. Desgraciadamente, las diversas alterna-
crónico en el que destacan disuria, polaquiuria con nicturia y dolor suprapú - tivas de tratamiento únicamente pueden encaminarse a una mejoría sinto-
bico, acompañados en ocasiones de hematuria (20-30%). mática, en la mayoría de los casos con resultados discretos:
Distensión hidráulica vesical.

..--...
Amitriptilina oral.
Instilación con dimetilsulfóxido (DMSO).
Corticoides tópicos o sistémicos.
Denervación vesical.
Existen muchas más causas de síndrome dstítico: cistitis aguda, tubercu-
losis, carcinoma in situ ... Cistoplastias de aumento.
Cistectomía .

....
..--... ✓ La causa más frecuente de infección de vías urinarias (IVU) es Escherichia ✓ Si se recoge la muestra urinaria mediante punción suprapúbica, cual-

..
co/i, tanto a nivel comunitario como nosocomlal. quier número de bacterias es significativo.

✓ ✓ La bacteriuria asintomática se trata en gestantes, menores de 5 años y

,,.
El origen más frecuente de uretritis es Chlamydia trachomatis.
previamente a la cirugía urológica, o si la especie implicada es Proreus.
✓ La causa habitual de orquiepididimitis depende de la edad: Chlamydia

....... ✓
y gonococo si es menor de 35 años; enterobacterias, si es mayor de esa
edad.

El diagnóstico definitivo de IVU es microbiológico: más de 105 UFC/ml.


✓ Los sistemas de drenaje cerrados son preferibles a los abiertos, pues la
tasa de infección es menor.

...... No obstante, este criterio varía con el sistema de recogida .

.....
..
Paciente masculino de 58 af'los de edad, con antecedentes de Importancia de cambios inflamatorios crónicos, sin presencia de ectasia. ¿Cuál serla el
de DM de 15 años evolución, no refiere apego a tratamiento. El paciente diagnostico probable?

....
acude al servicio de urgencias por presencia de náusea, vómito, y dolor
a nivel de región lumbar bilateral de predominio derecho. Signos vitales: 1) Cistitis crónica.
TA 130/90 mmHg, FC 192 1pm, Temp 38.3ºC. Se realiza estudio de examen 2) Prostatitis.
general de orina observándose presencia de abundantes bacterias, hema- 3) Litiasis renal.
turia microscópica, piuria. Se realiza ultrasonido observándose presencia 4) Pieionefritis.

,,.
,_. •
Libro CTO de Medicina y Cirugía, 6.ª ed. ENARM

El agente patógeno más frecuente de infección de vías urinarias es: Una paciente de 27 años acude al servicio de urgencias por dolor en fosa
renal derecha, fiebre de 39ºC, escalofríos y s/ndrome mlcclonal acompa-
1) Escherichia cofi. ñante. Es alérgica a penicilinas. Señale la respuesta correcta:
2) Proteus.
3) Klebsiefla. 1) No será necesario descartar patología urinaria obstructiva en este caso,
4) Staphy/ococcus saprophyticus. ya que presenta un claro síndrome miccional.
2) Para poder hacer el diagnóstico de pielonefritis se deberán conocer pri-
Es la v/a de diseminación de infección de vías urinarias más frecuente mero los datos referidos a la función renal.
es : 3) Se deberá iniciar tratamiento empírico con un ~-lactámico.
4) Si en las primeras horas evoluciona favorablemente, podrá continuar el
1) Hematógena. tratamiento de forma ambulante.
2) Ascendente.
3) Linfática . Un paciente de 83 ailos sondado de forma permanente acude a la consulta
4) Cutánea. tras detectársele dos cultivos positivos tomados con una semana de dife-
rencia. Asegura encontrarse asintomático. La actitud más adecuada será :
Masculino de 59 años, con diagnóstico de hiperplasia prostática, sin otros
problemas de salud, portador de sonda uretral permanente, presenta bac- 1) Iniciar tratamiento antibiótico según antibiograma de los cultivos ob-
teriuria (> 10s unidades formadoras de colonias) en dos urocultivos. ¿Cuál tenidos.
es la actitud terapéutica más conveniente? 2) Tranquilizar al paciente y seguir con su pauta habitual de recambio de
sonda.
1) Tratamiento antibiótico de amplio espectro. 3) Realizar cambio de sonda de forma inmediata con tratamiento anti-
2) Tratamiento antibiótico según antibiograma . biótico.
3) Continuar con sus cambios de sonda . 4) Realizar cambio de sonda de forma inmediata con profilaxis antibiótica
4) Antisépticos en vejiga urinaria. de 4 días.

Es un criterio para clasificar a un paciente con infección de vías u!inarlas Hombre de 54 años, con cuadro clínico de 6 meses de evolución carac-
complicada: terizado por disnea de medianos esfuerzos; en últimas 6 semanas fiebre,
astenia, adinamia y pérdida de peso, y 4 días antes disuria y polaquiuria.
1) Antecedente de cirugía de apendicitis. La radiografía de tórax muestra derrame pleural derecho e infiltrados algo-
2) Paciente femenino 55 años de edad. donosos generalizados. Prueba de VIH negativa. Se realiza urocultivo que
3) Paciente con luxación congénita de cadera. resulta negativo, examen general de orina reporta microhematuria. ¿Cuál
4) Paciente embarazada. es el diagnóstico más probable?

Paciente masculino de 24 años, acude a revisión médica por presentar fie- 1) Pielonefritis.
bre elevada, acompañada de dolor, inflamación y enrojecimiento testicular 2) Bacteriuria asintomática.
izquierdo. En relación con el cuadro clínico, ¿cuál de las siguientes afirma- 3) Tuberculosis extrapulmonar.
ciones es INCORRECTA? 4) Neumonía adquirida en la comunidad.

1) El diagnóstico más probable es epididimitis. Con base en su diagnóstico, ¿cuál es la duración del tratamiento?
2) Los patógenos más frecuentes son Chlamydia trachomatis y Neisseria
gonorrhoeae. 1) 9 meses.
3) El tratamiento de elección es vancomicina + gentamicina. 2) 6 meses.
4) El tratamiento de elección puede ser ofloxacino. 3) 12 meses.
4) 24 meses.
Ante un paciente que presenta febrícula persistente, crisis renoureterales
breves, piuria estéril, orina con pH ácido, microhematurla persistente, con Femenino de 29 años de edad, cursando con embarazo de 12 semanas de
citología urinaria negativa y epldldlmos indurados, ¿en qué enfermedad se gestación, examen de rutina muestra infección de vías urinarias. Paciente
debe pensar primero? niega sintomatologfa urinaria. ¿Cuál es la conducta a seguir?

1) Sarcoidosis. 1) Hallazgo incidental, no necesita tratamiento.


2) Carcinoma vesical. 2) Tratamiento con amoxicilina durante 3 días.
3) Carcinoma renal. 3) Tratamiento con amoxicilina durante 7 días.
4) Tuberculosis urogenital. 4) Se recomienda a la paciente que consuma jugo de arándano y se pres-
cribe ácido ascórbico .


02. Infecciones de las vías urinarias I UR

...... A 52-year-old patient presents to the emergency department with com-


plaints of fever, chills, perineal pain and pallor. In the morning, he un-
derwent a trans- rectal prostate blopsy. On physical examination, persistent
low blood pressure and high fever are found. Hls laboratory tests reveal
A 23-year-old woman, who is in her 13th week of pregnancy, is brought
to the emergency department complaining of hematuria, high fever, chills
and left flank pain. In the previous days, she presented dysuria, polyakyuria
which made her recall a previous cystitis episode, so she took fosfomycin .

-.......
eievated WBC count. Which of the following sentences Is incorrect? In this patient, whlch of the following options is correct?

1) Hemodynamic stabílization is mandatory. 1} An abdominal X-ray must be performed to rule out an obstructive cause.
2) This ciinical cond ition is very rare after a transrecta i biopsy, since it isn' t a 2) She may receive a 21-days treatment with levofloxacin.
frequent compl ication of this procedure. 3) A fetal wellness test must be performed.
3) Long-term antibiotic treatment is indicated. 4) In pregnant women, cystitis can lead to acute pyelonephritis in 10% of cases .

-......
4) lt 's unlikely that he needs to undergo a surglcal procedure to treat his
disease. Which of the following is the best diagnostic test for genitourinary tu-
berculosis?
A 23-year-old woman visits her physiclan because a culture performed a
week ago shows >10,000 CFU/ml on non-resistant E.col/. Currently, she 1) Agar-agar culture .

......
doesn't have any symptoms, mentionlng she had cystitis once during her 2) Ziehl-Nielsen staining.
teenage years. Blood tests reveal: Cr 0.7 mg/dl, WBC 9000/mm 3 and Hb 3) Urine PCR test.
13.2 g/dl. Pregnancy test is positive. Which of the following is the correct 4) Lowenstein culture.
management for this patient?

......
Which of the followlng is not true regardlng recurrent urinarytract infections?
1) Treatment is not necessary, since it's an asymptomatic bacteriuria.
2) She will probably develop sympt oms in a few days . 1) lt's diagnosed when a patient presents four or more episodes per year.
3) She needs treatment, given the high likelihood of developing acute pye- 2) lt usually appears in young male patients who are sexually active.
lonephritis . 3) Sometimes it may be assocíated with Staphylococcus saprophyticus.
4) Cephalexin 250 mg/q6 hours may be an effective treatment option . 4} The first step in management consists in taking hygiene measures.


i
t
\
03
~
,. l
[ Urolitiasis

3.2. Manifestaciones clínicas


Tema fundamental en esta asignatura. Se debe conocer muy bien la actitud ante la litiasis y su manejo agudo
en general, ante los distintos tipos de cálculos y, especialmente, todo lo relacionado con el
tratamiento. Es un tema rentable yagradecido, así que hay que em~ear el tiempo necesario. El dolor agudo del cólico renal es la manifestación más típica de la litiasis
La tabla-resumen de urolítiasiS puede ser de gran ayuda. renal. El dolor se produce por la sobredistensión de la vía urinaria tras la
obstrucción de esta por el cálculo. Es lógico, por tanto, que el cálculo deba
desplazarse desde su origen calicial para producir sintomatología aguda.
3.1 . Epidemiología Ocasionalmente, se observan cuadros de dolor vago renal en relación con
litiasis caliciales no desplazadas.
La litiasis del aparato urinario es una patología frecuente en la población
mundial, y en México representa un problema endémico, constituyendo El cólico renal o crisis renoureteral suele aparecer de forma progresiva
una causa de consulta y de atención urgente con potencial daño estructural sobre la fosa lumbar afectada, irradiándose por el naneo hacia la ingle y
y repercusión funcional en todo el aparato urinario. los genitales (Figura 3.1 ). El paciente generalmente se encuentra afectado,
con dolor que no cede con reposo, por lo que cambia de postura continua-
Son diversas las moléculas que forman parte de los cálculos. Su incidencia mente. Puede acompañarse de un cortejo vegetativo con náuseas, vómitos
varía según el país, e incluso según las áreas geográficas dentro del mismo y diaforesis. El dolor irradiado hacia la ingle indica, generalmente, que el
país. Se pueden distinguir seis grupos de componentes: cálculo ha alcanzado el uréter. Cuando se encuentra en vecindad de la vejiga
Oxalato cálcico. o dentro de esta, puede aparecer un cuadro irritativo, similar al síndrome
Fosfato cálcico. míccional, con polaquiuría, disuria y tenesmo vesical.
Fosfato no cálcico.
Compuestos purínicos (ácido úrico, urato amónico, urato sódico, xan -
tina, 2,8-dihidroxiadenina). Sudoración - - - ' i
Aminoácidos (cistina).
Otros (carbonato cálcico, sulfamidas ... ).

Los cálculos de oxalato cálcico son los más frecuentes (en torno al 65%),
seguidos por los infecciosos y los de ácido úrico (alrededor del 15% cada
RX simple de abdomen
uno). Más baja es la incidencia de los de fosfato cálcico (5%) y cistina (1-3%).
con cálculo a nivel de l3

La terce ra década de la vida es la edad media de primera aparición de la


litiasis, salvo en las de cistina, que suelen ser de aparición más prematura.

-
Se deben tener en cuenta los factores de riesgo para la formación de cálcu-
los: antecedentes familiares, infecciones, hiperparatiroidismo, enfermeda-
des gastrointestinales y alteraciones anatómicas, entre otras.
Dolor de inicio

Síndrome miccional
Hematuria

Las infecciones urinarias son más frecuentes en mujeres que en varo-


nes. Por eso, los cálculos de estruvita tamb ién lo son. Figura 3.1. Diagnóstico de urolitiasís

Los cálculos infecciosos de estruvita y, en menor medida, los de ácido úrico y


La enfermedad litiásica recidiva en el 40% de los casos, con una media de cistína, pueden crecer modelando las cavidades renales (litiasis coraliforme
un nuevo cálculo cada 2-3 años. Se entiende por recidiva la aparición de o "en asta de venado" (F igura 3. 21), manifestándose no como cólico sino
una nueva litiasis de la misma composición y en la misma localización, en un como infecciones urinarias de repetición , dolor lumbar sordo, hematuria o
intervalo menor de 4 años entre un cálculo y otro. incluso insuficiencia renal terminal.


03. Urolitiasis I UR

para la litiasis; aunque su alto coste hace que todavía no esté extendido su
uso, permite evaluar todo tipo de cálculos (Figura 3.3).

En niños y embarazadas, la ecografía urinaria es la técnica de imagen de


elección.

La ecografla debe ser el estudio de imagen inicial en el servicio de ur-


gencias; sin embargo, la TC simple es la prueba con el mejor rendimien-
to diagnóstico, por lo tanto, si se cuenta con el recurso, es el estudio de
elección en el paciente con litiasis ureteral.
La radiografía simple se sugiere utilizar únicamente si no se encuentran
disponibles la ecografía o la TC.

Figura 3.2 . Litiasis coraliforme o "en asta de venado"

Diagnóstico
El análisis básico de orina muestra generalmente hematuria y leucocituria .
Una piuria importante apoyaría la posibilidad de infección sobreañadida,
aunque ninguno de estos datos es realmente determinante.

Exámenes básicos que se han de realizar en Urgencias a pacientes con cólico


renal :
Uroanálisis.
Análisis de sedimento urinario y/o urocultivo.
Química sanguínea: electrolitos séricos, creatinina sérica, ácido úrico
y PCR.
Biometría hemática.

Figura 3.3. Tomografía computarizada, corte coronal de lito en unión


El análisis de la composición de los cálculos se debe realizar en todos los ureterovesical
cuadros iniciales, y las técnicas para hacerlo son con espectroscopia o
difracción de rayos X.

En teoría, el 70% de los cálculos son visibles en una radiografía simple de


abdomen, aunque este porcentaje es considerablemente menor en las
radiografías urgentes sin preparación intestinal. Radiológicamente, la mayo-
ría de los cálculos son radioopacos, exceptuando los de ácido úrico y algu-
nas otra s composiciones infrecuentes (sulfamidas, xantina, indinavir). La Tratamiento
ecografía es el estudio de imagen inicial en el servicio de urgencias y debe
realizarse, de preferencia, en los pacientes con sospecha de nefrolitiasis. El manejo agudo del cólico renal se basa en el control del dolor. Pa ra esto,
Permitirá visualizar incluso las litiasis radiotransparentes, con el inconve- es preciso conseguir una disminución de la presión dentro de la vía urinaria,
niente de no ser vistas aquellas ubicadas en el trayecto ureteral (salvo las siendo los fármacos de primera línea los antiinflamatorios i.v, que disminu-
zonas cercanas a la vejiga o al riñón). También se podrá evaluar el grado de yen el dolor y la diuresis al inhibir la síntesis de prostaglandinas. Asimismo,

--,,,.,,,,.
hidronefrosis. se pueden usar espasmolíticos, que disminuyen la presión intraureteral al
1 relajar la pared del uréter. Las recomendaciones para el manejo del cuadro
La urografía ofrece información morfológica y funcional de ambos riñones. agudo son: de primera elección, diclofenaco 75 mg i.v y/o metamizol sódico
Debe tenerse en cuenta que, durante el cólico renal, puede observarse una 2 gr i.v en infusión lenta, indometacina o ibuprofeno. Como fármaco de
1
anulación funcional, sin que signifique necesariamente deterioro de dicha segunda línea para el control del dolor o terapia de rescate, se recomienda
unidad renal. Mediante esta técnica, se puede diag_nosticar todo tipo de cál- clonixinato de lisina 100 mg i.v, tramado! 50 mg i.v, o morfina 5 mg i.v, en
1
culos, ya sean radiotransparentes o radioopacos. El principal inconveniente dosis única. Se pueden utilizar a-bloqueadores para reducir la recurrencia

----,,,. de este procedimiento es la introducción de contraste yodado, que está de cuad ros.
1
contraindicado en los pacientes con alergia, creatinina mayor de 2 mg/dl,
mieloma múltiple o deshidratación importante. Según las guías clínicas, la En los pacientes con infección de vías urinarias y cólico renoureteral, pre-
1
urografía intravenosa (UIV) ha sido desplazada actualmente por la TC heli- via toma de muestra para cultivo de orina deberá iniciarse manejo antibió-
coidal sin contraste, que se ha convertido en el nuevo estudio de referencia tico con fluoroquinolona, inhibidor de betalactamasa de amplio espectro,


1

1
Libro CTO de Medicina y Cirugía, 6.ª ed. ENARM

cefalosporinas de 2ª o 3! generación, o aminoglucósidos como antibióticos l ' · · ·1 · l ' . 15 Litiasis


itiasis ca cica Litiasis úrica itias infecciosa
empíricos, por 7 a 14 días. (oxalato o fo sfato) c1stmIca (estruvital

Hipercakiuria idiopática Gota primaria Cistinuria Infecciones


Existe una serie de situaciones en las que el cólico renal se convierte en Hipercalciuria secundaria Hemopatías por gérmenes
una urgencia que prec isa hospitalización y, eventualmente, manipulación a hipercalcemia Enfermedades productores
Hiperurlcosurla digestivas deureasa
invasiva :
Hiperoxaluria lngesta excesiva
Obstrucción grave, principalmente si se acompaña de litiasis mayor de Hipocltraturia de purinas •
5 mm u obstrucción bilateral. Acidosis renal tubular Fármacos
Dolor incontrolable. distal Litiasis úrica
Litiasis cálcica idiopática ldiopática
Riñón único. - - ---- -------- - --- -
Tabla 3.1. Tipos de litiasis. Situaciones que favorecen su aparición
Sepsis de origen urológico .
Falla renal aguda o anuria.
Mujer gestante. Edad temprana de aparición
Litiasis bilateral
Riñón trasplantado.
Utiásis en riñón único o malformado
Cornpo5ición poco frecuente
Asimismo, en pacientes diabéti- Litiasis recidivante
cos, por el mayor ri esgo de com- Nefroclacinosis
litiasis coraliforme
plicaciones, es aconsejable, si no
el ingreso, al menos una observa -
Tabla 3.2. Pacientes con indicación de estudio metabólico
ción estricta . Una situación simi-
lar ocurre durante el embarazo, A Litiasis cálcica
en el que una dilatación leve de
la vía urinaria puede considerarse En la mayoría de las ocasiones, se desconoce el origen de la litiasis cálcica,
"fisiológica", pero obstrucciones aunque se puede hacer una aproximación a los factores de riesgo que influ -
más importantes o la aparición yen en su aparición . Sólo en un pequeño porcentaje de casos existe una
de fiebre hacen aconsejable la enfermedad de base que puede ser tratada, y de esta forma desaparece la
colocación de un catéter ureteral, formación de cálculos cálcicos .
además de terapia expulsiva con Figura 3.4. Catéter doble J . Hipercalciuria idiopática. Es la causa más frecuente de litiasis cál-
tamsulosina 0.4 mg cada 24 horas derecho. Litiasis ureteral derecha. cica . Se define como una excreción urinaria de calcio mayor de 30D
(Figura 3.4). Litiasis coraliforme izquierda mg/24 h en el va rón y 250 mg/24 h en la mujer. De cara a su manejo,
las tiazidas dism inuyen el calcio urinario, red uciendo la formación
de litiasis. La admin istración de citrato potásico ayuda a evitar la
hipopotasemia y aumenta el citrato urinario, que es inhibidor de la
litogénesis (Tabla 3.3) .
En el manejo agudo del cólico renoureteral en el servicio de urgencias, Hlperurlcosuria. Excreción en orina de más de 800 mg/24 h en el
los AINE i.v (diclofenaco o metamizol} son de primera elección, y como varón o 750 mg/24 h en la mujer. Además de favorecer la litia sis úrica,
segunda línea o terapia de rescate se administra clonixinato de lisina,
la hiperuricosuria constituye un factor de riesgo para la formación de
tramado! o morfina.
cálculos de calcio, probablemente por nucleación heterogénea sobre
núcleos de ácido úrico o urato sódico. Generalmente se debe a un
exceso de purinas en la dieta.
3.3. Evaluación y tratamiento Hiperoxaluria. Se considera como tal la excreción en orina de más de
40 mg/24 h. Existe una hiperoxaluria primaria, que es consecuencia
de la litiasis renal de un defecto enzimático autosómico recesivo; no tiene tratamiento
y generalmente conduce a insuficiencia renal por liti as is recidivante .
En este apartado se puede dividir en dos partes: estudio de la litiasis con la El único tratamiento que existe actualmente es el trasplante hepá-
finalidad de instaurar un tratamiento preventivo de su formación, y estudio tico, que suele ir unido al renal, aunque algunos casos responden a
y tratam iento de la litiasis ya formada . piridoxi na. No obstante, la mayoría de los casos de hiperoxaluria son
secundarios a malabsorción de ácidos grasos por enfermedades cró-
Estudio y tratamiento preventivo nicas pancreatobiliares, derivación intestina l para el tratamiento de la
obesidad mórbida, resección ileal, enfermedad inflamatoria intestinal,
La evaluación del paciente con litiasis se basa en un estudio metabólico para hipercalciuria coincidente o por falta de calcio en la dieta, lo que per-
determinar qué factores son modificables, en un intento de evitar la recidiva mite que exista mayor cantidad de oxalato intestinal para su absorción.
(Tabla 3.1 ). Este estudio debe reservarse para aquellos pacientes con alta La intoxicación por etilenglicol y metoxiflurano puede producir hipe-
probabilidad de recidiva, aunque cada vez más autores indican que debe roxaluria, así como la ingesta de vitamina C en altas dosis. En todos
realizarse a todos los paciente s {Tabla 3.2). estos casos secundarios, el tratamiento con colestiramina, una dieta
pobre en grasas y la corrección de la malabsorción, en la medida de lo
Desde un punto de vista práctico, las litiasis se pueden dividir en las de posible, suelen ser medidas eficaces.
composición cálcica y las de otras composiciones, ya que el primer grupo Hipocitraturia. Excreción de citrato inferior a 30D mg/24 h. General-
supone la mayoría de los casos (70-80%) tratados habitualmente. mente se asocia a otras anomalías urinarias. Aunque de causa deseo-


03. Urolitiasis I UR

nacida, puede contribuir una dieta rica en proteínas, hipopotasemia, sódico. Una alternativa es la acetazolamida en dosis de 250 mg/día. Cuando,

......
enfermedad intestinal o infección urinaria . además, la uricemia es alta, puede tratarse con alopurinol.
Hiperparatiroldismo primario (véase Sección de Endocrinología).
Supone la causa más frecuente de hipercalciuria conocida . C. Litiasis cistínica
Acidosis tubular renal distal (véase Sección de Nefro/ogío) . Enfer-
medad autosómica recesiva . Consiste en la imposibilidad del túbulo La cistinuria es un trastorno autosómico recesivo en el que existe un defecto

...... distal para excretar hidrogeniones a la orina (orinas persistentemente


alcalinas) con aumento de la eliminación de calcio a la orina . Exis-
ten formas incompletas que se observan en pacientes formadores de
cálculos de oxalato cálcico y con hipercalciuria idiopática. En estos
de absorción, a nivel intestinal y tubular proximal, de los aminoácidos dibá-
sicos: cistina, ornitina, lisina y arginina (COLA) , aunque parece que puede
existir un trastorno en el que únicamente se ve afectada la cistina, lo que
indicaría que, además de un mecanismo de transporte común, existe uno

......
probablemente la acidosis tubular no juegue un papel importante y independiente para la cistina .
responden a tiazidas.
otras circunstancias que favorecen la litiasis cálcica: Sarcoidosis, sín- Los niveles de cistina en orina de 24 horas son superiores a 100 mg, de
drome de Cush ing, diuresis escasa, déficit de inhibidores o anomalías hecho, los homocigotos pueden excretar más de 600 mg/día. El diagnóstico

......
en el pH urinari o (alcalosis). se realiza identificando los característicos cristales hexagonales en orina, o
Litiasis cálcica idiopática. Aproximadamente en el 20% de los pacien- por una prueba positiva de nitroprusiato sódico (la orina se tiñe de azul en
tes con litiasis cálcica no se demuestra ninguna anomalía en el estudio paci entes afectados por esta enfermedad : test de Brand) .
metabólico.

.....
El tratamiento consiste en aumentar la diuresis diaria (más de 3 1/día), alca-
Absor t1 va'.l. Resor t 1vas Re n<1 \e c;, linizar la orina por encima de 7,5 y, en caso de que esto sea insuficiente,
puede iniciarse tratamiento con D-penicilam ina (250 mg/6 h) o a-mercap-
Aporte excesivo · Hlperparatiroidismo Acidosistubular distal topropionilglicina (250 mg/6 h).
Síndrome de .Bumett . lnmovillzaclón ldlopática

......
(leche y alcalinos) Tumorales D. Litiasis infecciosa
Hipervitaminosis D Enf. Paget .
ldiopática Sd de Cushlng
Los cálculos infecciosos de estruvita o de fosfato amonico magnesIco
Sarcoidosls
(MgNH,PO,_6 H2O) se desarrollan en un ambiente alcalino, producido por

......
Tabla 3.3. Causas más frecuentes de hipercalciuria infección persistente de gérmenes que hidrolizan la urea, aumentando la
cantidad de amonio urinario. Los principales gérmenes que poseen ureasa,
además de diversas especies de Proteus, son Pseudomonos, Klebsiel/a,
Serratia y Enterabacter. La presencia de cuerpos extraños (sondas vesicales ,
suturas) favorece su formación .

La causa más frecuente de hipercalcémía en un paciente ambulatorio


es el hiperparatiroidismo primario. En cambio, la hipercalcemia más fre- Para su tratamiento, se han empleado diversos métodos, generalmente
cuente en uno ingresado es la de origen neoplásico . . ineficaces. La antibioterapia únicamente mantiene estéril la orina durante
los cursos de tratamiento. Parece más prometedor el uso de inhibidores
de la ureasa con ácidos hidroxámicos. Estos son moléculas análogas a la
urea que forman un complejo enzima-inhibidor irreversible. Se utilizan bási·
camente dos sustancias de esta naturaleza : el ácido propiónico y el aceto·
hidroxámico. Su empleo suele venir acompañado de cefaleas, temblores,
La furosemida, al revés que las tiazidas, aumenta el calcio urinario. trombosis venosas u otros síntomas neurológicos, por lo que tampoco son
de gran aceptación.

B. Litiasis úrica Todo lo relativo al estud io de la nefrolitiasis expuesto anteriormente se


puede repasar en la Tabla 3.4.
El áci do úrico no disociado es poco soluble en orina. Con un pH urinario de
5, la solubilidad del ácido úrico es únicamente de 100 mg/I, mientras que Tratamiento de la litiasis ya formada {Figura 3.5)
con un pH de 7 es de 1.580 mg/1. Esto demuestra la gran importancia del
pH urinario en la formación de cálculos de ácido úrico. Aparte de estos, Los cálculos ya formados no expulsables (> 6 mm) precisan tratamiento
también existe una pequeña proporción de cálculos de urato monosódico "agresivo'; es decir, necesitan ser extraídos quirúrgicamente o fragmenta•
y urato amónico. dos de forma que puedan ser expulsados espontáneamente. Las diversas
formas de tratamiento son las siguientes:
El objetivo del tratamiento es reducir el ácido úrico excretado y aumentar Cirugla. Ha sido el t ratamiento estándar hasta la aparición de la litotri-
el pH urinario, ya que los cálculos más frecuentes en pacientes hiperuricé- cia extracorpórea. Aún hoy día, es preciso recurrir a la cirugía cuando
micos son los de ácido úrico. Por otra parte, este tipo de cálculos son los fracasan las ondas de choque o en determinados casos para reducir la
que mejor responden al tratamiento médico mediante quimiólisis por alca- masa litiásica (cálculos coraliformes).
linización urinaria. Pueden administrarse diversos álcalis; el citrato potásico Endourología. La manipulación endoscópica de la vía urinaria es cada
impediría el teórico riesgo de formación de cálculos cálcicos por su efecto día más accesible gracias a las mejoras técnicas. Puede realizarse
inhibidor, pero tamb ién pueden tratarse con bicarbonato sódico o citrato extracción directa del cálculo mediante diversos tipos de pinzas o


Libro CTO de Medicina y Cirugía, 6.ª ed. ENARM

Mujer Varón Varón = Mujer


Hipercalciuria idiopática Infección por gérmenes ureasa (+} Gota(SO%) Ostinuria
ldiopática ldiopática (< 50%)
Hiperuricosuria (20%) Hlperuricemias secundarlas
Alcalino Ácido Ácido
Radioopacos Radlotransparentes Radlolúcidos
Formas prismáticas polimórficas Aglomerados de cristales Cristales hexagonales en prismas
Cristales 'en ataúd' desorganizados, a veces formando o láminas
masas continuas

Cristales de fosfato Ca

q1l~
fll '"'~~◊
CJ
Wo
o
Cristales de Oxea Cristales de estruvita Cristales de ácido úrico Cristales de cistina
Hlpercalciuria idiopática: tiazidas Ácido propiónico y ácido Alcalinizar la orina Forzar diuresis Ongest/1 hldrica)
Hiperoxaluria primaria: plrldoxlna acetohldroxámico Alopurlnol Alcallnlzar orina ·
Hiperoxalurla secundaria: colestlramlna Antibloterapla (si hay hlperuricemla) D·penicilamina
·--·------------------------- ---··-- ----·------- ~ ocasionesdrugla - - - - - - · - Dieta de bajo contenido proteico______ (si n<:> _
~y_~~s~~~ra) _.. ·-·--- ____ __
Tabla 3.4. Tabla-resumen de las nefrolitlasis

cestillas, o bien fragmentar previamente el cálculo mediante diver- Litotricia extracorpórea por ondas de choque (LEOC). Las ondas de
sas fuentes de energía, como electrohidráulica, ultrasónica o láser. Se choque se transmiten a través de los tejidos corporales con la misma
puede acceder hasta el cálculo mediante ureterorrenoscopia (URS) o impedancia acústica que el agua hasta alcanzar la litiasis, sobre la que
nefrolitotomla percutánea (NLPC). produce fenómenos de compresión y descompresión que conduci-

Manejo de la urolltiasls

Crisis cuadro Estable cuadro

No complicado
agudo

Complicado
crónico

¿Edad?
¿Tipo de cálculos?
j
¿Periodicidad de la clínica?
¿Tipo de síntomas?
¿Viabilidad renal?
Tratamiento sintomático: •Obstrucción grave
• Espasmolfticos y · Infección, fiebre
antiinílamatorios • Dolor incoercible
· Reposo e hidratación • Riñón único Tratar la condición
preexistente

• Cálcica: acidificar (no útil si oxalato), · Extracorpórea (LEOC) · Ureterotomla


citratos, tiazidas colestiramina · Percutánea ultrasonogránca · Pielolltotomia
y dieta baja en grasas, rica en calcio, con microlumbotomía · Nefrectomia
si hiperoxaluria · Endoscópica vía ureteral
· Úrka: alcalinizar, alopurinol
• Estruvlta: acetohidroxámico
Ingreso y tratamiento agresivo: • Cistina: D-penicilamina, vit. 86 , Contraindicada en:
• Drenaje (catéter o nefrostomía) y alcalinizar •Embarazo
•Tratamiento parenteral: · Infección
- Antibióticos · Obstrucción distal
- Remontar hemodinámica · Aneurismas
- Equilibrio electrolítico •Coagulopatias
- Narcóticos -Obesidad
· Vigilancia estrecha · Arritmia cardíaca

Figura 3 .5 . Manejo de la litiasis renal ya formada


03. Urolitiasis I UR

rán a su fragmentación. La gran mayoría de los cálculos renales son está recomendada la resonancia magnética (RM). Toda mujer con diag-
susceptibles de tratamiento mediante LEOC. Las únicas limitaciones nóstico de cólico renoureteral debe ser hospitalizada y el tratamiento se
serían aquellos cálculos no localizables por su pequeño tamaño(< 2-5 basa en manejo analgésico, de preferencia con acetaminofén o en su caso

......
mm), dureza del cálculo mayor de 1.000 UH u obesidad, también par- oxicodona; sin embargo, en caso de ser necesario se realiza cirugía. La
ticularidades anatómicas de la vla excretora y paciente, función renal y litotricia extracorpórea por ondas de choque está contraindicada en el
tipo de litotriptor disponible. La presencia de hipertensión arterial no embarazo.
controlada facilita el riesgo de hemorragia durante la sesión de litotri-
cia, luego deberá ser estabilizada previamente a la misma y constituye, complicaciones

...... en cierto modo, por ello, contraindicación relativa de LEOC (Tabla 3.5) .

Absolutas Relati va s
(pre ci san de control pre vio al tratam1 cntol
La expulsión de fragmentos litiásicos puede ocasionar un cólico renal y, con
menor frecuencia, obstrucción ureteral (steinstrasse o "calle litiásica"). Esta
posibilidad es mayor ante litiasis de gran tamal'lo, por lo que en algunos de

......
Embarazo Alteraciones de la coagulación estos casos se puede colocar un catéter de derivación urinaria (nefrostomía.
Obstrucción distal Aneurisma aórtico
o doble J) antes de la LEOC para disminuir este riesgo, generalmente en
Infección activa Alteraciones del ritmo cardiaco. marcapasos
Malformaciones óseas o desfibriladores · litiasis superiones a 2 cm.
graves Obesidad
Hipertensión arterial descontrolada

.....
Derivadas del efecto directo de las ondas de choque, pueden aparecer
Tabla 3.5. Contraindicaciones de LEOC contusiones renales manifestadas como hematuria, hematomas rena-
les, equimosis o eritema cutáneo y, en grado máximo, rotura renal. La
hematuria se considera la complicación más frecuente de la litotricia.
A Tratamiento de la litiasis en el embarazo Más controvertida es la teórica relación de la LEOC con la aparición de

...... El diagnóstico y manejo de litiasis en mujeres gestantes varía ligeramente.


Para el diagnóstico el estudio inicial es el ultrasonido, y en caso de duda
hipertensión arterial, ya que no está demostrada en las últimas revisio-
nes publicadas, aunque sí la relación entre hematoma renal post-LEOC e
hipertensión arterial.

......
..... ✓


Los cálculos más frecuentes son los de oxalato cálcico .

Globalmente, la litiasis es más común en el varón, salvo las de estruvita,


más comunes en mujeres.


Las tiazidas son útiles para la hipercaiciuria idiopática.

Los cálculos asociados a las resecciones ileales o a la enfermedad infla-


matoria intestinal son de oxalato cálcico.

...--
✓ La radiograffa de abdomen no permite ver algunos cálculos, como los ✓ Precipitan en medio ácido: ácido úrico y cistina. Precipitan en medio al-
de urato. Sin embargo, la ecografía puede verlos, independientemente calino las que contienen fosfatos (fosfato amónico magnésico o estruvi-
de su composición . ta, y el fosfato cálcico).

...... ✓ La TC simple, por ser la prueba con el mejor rendimiento diagnóstico, es


el estudio de elección en el paciente con litiasis ureteral.


En el tratamiento de la litiasis por ácido úrico es beneficioso alcalinizar
la orina.

Los cálculos de oxalato NO se ven alterados por el pH (al Oxal, el pH le da

......
✓ En el servicio de urgencias, se debe iniciar tratamiento con AINES por vla
intravenosa para control del dolor en pacientes con cólico renoureteral. igual).
Se puede utilizar diclofenaco, o como segunda opción el metamlzol só-
dico, pero con infusión lenta. La administración de clonixinato de lisina ✓ Los cálculos de estruvita se relacionan con microorganismos producto-
vía intravenosa se recomienda como fármaco de segunda línea para el res de ureasa, como Proteus.

...... ✓
control del dolor o terapia de rescate.

Litiasis radiotransparentes: Sulfamidas, lndinavir, Urato, Xantinas (SIUX).


Las de cistina son radiolúcidas; y el resto, radioopacas .
✓ Las contraindícaciones absolutas para la LEOC son: embarazo, infección
activa y obstrucción de las vías urinarias distal al cálculo.

......
----

Libro CTO de Medicina y Cirugía, 6.ª ed. ENARM

Mujer de 44 aílos de edad acude a consulta por presencia de dolor localizado 1) Alopurinol vía oral.
a nivel de reglón lumbar Izquierda, de t1po cólico de una Intensidad 8/10 con 2) Ureteroscopía con extracción del cálculo.
irradiación hacia genitales, el cual se acompaña de slntomatolog/a vaga!. Se 3) Nefrolitotomfa endoscópica percutánea.
realiza ultrasonido renal observándose presencia de ectasia renal grado 11, en 4) Alcalin ización de la orina por vía oral.
unidad renal izquierda, con presencia de imagen hiperecoica con presencia
de sombra acústica posterior de lxl cm . Laboratorios: creatinina 3.5 mg/dl, Mujer de 50 aflos, diabética insulinodependiente, con infecciones urinarias
urea 65 mg/dl. ¿Cuál es el tratamiento de elección en este caso? y cólicos nefríticos de repetición. Acude a Urgencias por dolor en fosa renal
izquierda de 5 días de evolución, asociado en las últimas 24 horas a fiebre,
1) Tratamiento analgésico y antibioticoterapia. escalofrlos y malestar general. Analítica sanguínea: plaquetopenia, leuco-
2) Administración de citrato de potasio vía oral. citosis y disminución de la actividad de la protrombina. Examen general
3) Quimiol isis con D-Penicilamina. de orina normal. Radiografía de abdomen con claras imágenes de litiasis.
4) Colocación de catéter ureteral doble J izquierdo+ antibioticoterapia. USG renal : dilatación moderada de sistema excretor izquierdo. ¿Cuál es la
conducta más adecuada?
Es la composición más común de los litos renales:
1) Solicitar hemocultivos y urocultivo para establecer la necesidad de anti-
1) Oxalato de ca lcio. bioterapia.
2) Ácido úrico. 2) Realizar urografía intravenosa para intentar fil iar la causa .
3) Cistina 3) Hidratar a la paciente bajo observación rigurosa, y repetir ecograffa a las
4) Estruvita . 48 horas.
4) Colocar catéter doble J o practicar nefrostomía percutánea de forma in -
Se considera el estudio con mayor sensibilidad y especificidad para el diag- mediata con cobertura antibiótica.
nóstico de litiasis de origen urológlco:
Masculino de 26 años de edad ingresa al servicio de urgencias por pre-
1) Ultrason ido con transductor de 7.SHz. sentar dolor tipo cólico localizado en fosa renal derecha, acompañado de
2) Urotomografía. fiebre, náuseas y vómito en cuatro ocasiones, además de presentar hema-
3) Resonancia magnética. turia franca . ¿Cuál es el tratamiento indicado en este caso?
4) Urografía excretora .
1) Manejo ambulatorio con ketorolaco, butilh ioscina y metoclopramida.
Son litos que por su composición no pueden ser visualizados en estudios de 2) Hospitalización, ketorolaco, antiespasmódico y diurético.
radiografía de abdomen : 3) Hospitalización, diclofenaco, antiespasmódico.
4) Manejo ambu latorio con paracetamol, antibioterapia empírica y cita a la
1) Oxalato de calcio . consulta externa .
2) Fosfato de calcio.
3) Xantinas. ¿Cuál es el estudio de imagen de elección?
4) Estruvita .
1) TC de abdomen .
Es una indicación para tratamiento quirúrgico de litiasis: 2) Placa de abdomen.
3) USG renal bilateral.
1) Lito de composición de indinavir. 4) Urografía excretora.
2) Litiasis rena l en paciente inmunosuprimido .
3) Litiasis renal en edad pediátrica. En el estudio de imagen solicitado, reportan cálculo de uretero derecho,
4) Paciente monorreno. tercio medio de 12 mm con hldronefrosls leve. ¿Cuál sería la mejor opción
de tratamiento para este paciente?
Hombre de 29 ai'ios, con antecedentes de dolor tipo cólico en fosa renal
izquierda que cedió con tratamiento analgésico, se le realiza una radiogra- 1) Terapia expulsiva con bloqueador a.
fía simple de abdomen donde no se localizan alteraciones. Sin embargo, al 2) Ureteroscopía derecha.
practicar una tomografía abdominal simple, se aprecia un lito en colector 3) AINE + bloqueador de canales de calcio.
inferior de 4 x 3 mm y 400 UH de densidad. El pH de la orina fue de 5,5. Asi- 4) Tamsulosina + hidroclorotiazida .
mismo, se observan cristales de urato, 9-12 hematíes por campo y escasa
leucocituria. ¿Cuál sería el tratamiento más apropiado?


... 03. Urolitiasis I UR

~
_..
.... A 28-week pregnant patlent is brought to the emergency department with
a sudden onset of severe left flank paln, lrradlated to her groln. She doesn't
report dysuria, even in the previous days. She also presents nausea and vo-
miting. She is afebrile. No similar prevlous episodes were recorded. Which
of the following el auses is the most appropriate?
A 34-year-old woman is hospitalized with complaints of right flank an iliac
fossa pain that commenced seven hours ago. Onset was sudden, and the
pain is slightly irradiated to her genitalia. She also presented with sweating,
nausea, vomits and chills. Her physical examination revea Is a body tempe-
rature 36.s•c and blood pressure 120/75 mmHg. A plain abdominal X-ray

---- 1) The ftrst test to perform is a plain x-ray film, since it'II enable us to detect
film shows a calculus. After analgesic treatment, the problem apparently
resolved so she was discharged; 10 days later, she comes to your outpatient

...
lithiasis, if present. cllnlc with the results of a no contrast CT that revea Is a 10 mm calculus In
2) lt would be reasonable to think that the patient's symptoms are con- the proximal segment of the right ureter. Laboratory studies do not show
sistent with a lithiasic reno-ureteral colic, since it's the most common any abnormal parameters. Which of the following is the recommended
urinary complication in pregnant patients. procedure in thls case?

---- 3) The first diagnostic test performed is a contrast-enhanced CT sean, the


current gold standard for diagnosis of urinary lithiasis.
4) Ultrasound may be used to analyze whether a urinary derivation device
would be indicated.
1) An emergent urology referra l must be performed .
2) Perform a scheduled ureterolithotomy.
3) Conservative management may be tried first, by admin istering propionic

...--
acid.
A 34-year-old woman is admitted to our emergency department with a his- 4) Perform a flexible ureteroscopy.
tory of right flank and right iliac fossa paln for the least seven hours. The paln
onset was sudden and she also presented sweating, nausea, vomiting and A patient presents to the emergency department with a history of sud-

.....--
chills . Physical examination reveals a temperature 36.s•c and blood pressure den pain after receiving extracorporeal shock wave lithotripsy to treat
of 120/75 mmHg. Which of the following statements is correct in this case? a ureteric calculus. What is the most frequent compiication of thls pro•
cedure?
1) The first diagnostic test to perform in this patient is a contrast enhanced
1) Hematuria .

......
CTscan .
2) lnitial treatment should include urine alkalinizing agents. 2) Steinstrasse.
3) Plain x-ray film will show typical staghorn ca lculus . 3) Hypertension.
4) The first step in treatment is administration of analgesics . 4) Rena l hematoma .

......
----
..--...
..--
...--
.....-- •
04
Hiperplasia y carcinoma prostático

La HPB está compuesta de una proliferación variable de elementos glandula-


res, musculares y del estroma, que en su crecimiento comprimen la próstata
Tanto la hiperplasia prostática benigna corno el cáncer de próstata son dos ternas periférica, formando la llamada cápsula quirúrgica. Su etiopatogenia no está
fundamentales. Probablemente el cáncer sea más importante, sobre todo lo referente al clara; aunque el estím ulo androgénico a través de su forma activa, la dihidro-
tratamiento. Hay que aprenderse muy bien el resumen de la Tabla 4.4. testosterona, es fundamental, su papel exacto no ha sido determinado. las
teorías más recientes abogan por un desequilibrio hormonal de estrógenos/
andrógenos, o por la existencia de factores de crecimiento prostático con un
papel permisivo del ambiente hormonal y alteración de la apoptosis celular.
4.1. Hiperplasia prostática benigna
No existe evidencia de asociación entre HPB y carcinoma prostático.
La hiperplasia prostática benigna (HPB) afecta en mayor o menor grado a la
gran mayoría de los varones a partir de la quinta década de la vida, y alcanza Diagnóstico
el 80-95% de la población masculina de 80 años.
El crecimiento prostático generalmente se produce hacia la zona transicio-
la próst ata se divide clásicamente en cinco lóbulos (anterior, medio, poste- nal alrededor de la uretral, ocasionando obstrucción de esta y dificultando
rior y dos laterales), aunque estos únicamente se encuentran como tales en el vaciamiento vesical (Fig ura 4.2).
la edad fetal. En el adu lto, se puede interpretar la anatom ía de la próstata
dividida en dos partes : una zona periférica, donde se origina principalmente
el carcinoma ; y una zona periuretral o transicional, de la que procede la HPB
(Figura 4. 1 ).

A
Estroma ñbromuscular anterior Zona de transición

lóbulo anterior

Figura 4.2. Ecografía de hiperplasia prostática con crecimiento del


~~
Conductos
B eyacula dores lóbulo medio intravesical
...
~... '..~
, Esto no se manifiesta Inmediatamente, sino que, generalmente, el proceso

¡. pasa por una serie de etapas que incluyen una fase de compensación, una

... , 4 J
. .._..• }-J\ J/••I fase clínica y una de descompensación
l. Fase de compensación . El crecimiento prostático ocasiona un
aumento de la presión uretral durante el vaciado que es compensado

l{/1 por una mayor actividad contráctil del detrusor que se hipertrofia,
encontrando presiones vesicales más elevadas. En esta fase, la clínica
l óbulo puede ser mínima o inexistente.
Glándulas lóbulo posterior
lateral 2. Fase clínica (Fígura 4.3). La elongación de las fibras musculares por
suburetrales
encima de un límite condiciona pérdida de capacidad contráctil. En
F19urn 4. 1. Zonas de McNeal (A} y analomia de la próstata (B) e ste mome nto, aparece retraso d e l Inicio de la micción, dl~m ln ución


,,,.
-,...,. del calibre y de la fuerza del chorro miccional y alargamiento del
04. Hiperplasia y carcinoma prostático

Síntomas obstructivos o de I
Síntomas
I UR

., Síntomas de llenado
vaciado (lo que, en conjunto, se denomina síntomas obstructivos). El vaciado posmiccionaies o "imtativos"
vaciado suele ser incompleto, dando lugar a un residuo posmiccional. Dificultad de inicio miccional Sensación Urgencia miccionai
Disminución del calibre y de vaciado Nicturia
3. Fase de descompensaclón (Tabla 4.1 ). Se produce un vencimiento del
fuerza del chorro micdonal incompleto Polaquiuria diurna
detrusor vesical, que es incapaz de vencer la presión uretral, aumen- Micción intermitente y Goteo Dolor suprapúbico
tando la sintomatología anterior y pudiendo presentarse retención prolongada posrnicdonal Incontinencia por
urinaria . Ocasionalmente puede aparecer dilatación ureteral bilateral Retención de orina urgencia miccional
e Incontinencia por
con deterioro de la función renal. Esto se debe a uropatía obstructiva
rebosamiento
infravesical con pérdida del mecanismo antirreflujo.
Tabla 4.1. Sintomas de HPB

Puede producirse también otro tipo de síntomas, denominados de llenado o


"irritativos", que son debidos a la alteración funcional vesical, y cuya resolu-
ción es más difícil tras la desaparición de la obstrucción. Entre estos s/ntomas
se incluyen polaquiuria, tenesmo, nicturia y urgencia miccional. La HPB es la
causa más frecuente de obstrucción del tracto urinario inferior en el varón.

....
La mejor herramienta para valorar los slntomas urinarios bajos es la escala
internacional de síntomas prostáticos (IPSS [Tabla 4.2]). Si bien no es diag-
nóstica, es un instrumento clave para valorar la respuesta al tratamiento y
calidad de vida de los pacientes.

En la evaluación del síndrome prostático, el tacto rectal continúa siendo


la exploración fundamental, sobre todo para diferenciarlo del carcinoma,
ya que no es infrecuente que ambas entidades coexistan. La clínica es lo
más importante para valorar la indicación de tratamiento de la HPB, ya que
no existe correlación entre el tamaño prostático y el grado de obstrucción.
Cualquier zona sospechosa al tacto debe ser biopsiada.

.... La medición del flujo máximo en una urotlujometría miccional es también


importante, considerándose normal cuando es mayor de 15 ml/s y clara-
mente patológico si es menor de 10 ml/s. El estudio puede completarse con
una ecografía que permita evaluar si existe afectación del tracto urinario
Figu ra 4.3. Hiperplasia prostática en fase clínica. UIV que muestra superior, residuo posmiccional, litiasis vesical u otra patología asociada. El
impronta prostática en vejiga (vejiga "en montera") con uréteres en uso del PSA en la HPB únicamente está indicado para descartar la presencia
"anzuelo" (flecha) de carcinoma en la próstata, ya que no sirve para diagnosticar HPB.

Menos Menos Aprox . Más


Casi
Cuestionario IPSS Ninguna de una vez , de la mitad la mitad dela mitad
siempre
cada 5 veces de las veces de las veces de las veces
Durc1ntc m c1'i. o menos los Ultimas 30 d1as, ¿cuantas vecec; ha tenido
la sflnsac1on de no vc1ciar completc1mente lil vejiga al termmar de o rmJr?
DurdntP n1c1'> o nh•noc, los ultimo"> 30 d1Jc,, ¿cuCmt.1'> Vl'tl'!J hJ tc111do 4ue volver d

orinitr J lds do,; hortlS s iqui t• ntes dC'spués de hrlbPr onn.1do?


Durdnte mr1'> o m,. . nos los últimoc; 30 rl1a s, ¿cuántas Vf1Ce1i ha notado que, al orina r,
pdrdba y comt,,nzi1ba de nuevo van,vi v~ces?
Durdnte """ u nll'no, los últimos 30 dids, ¿cuántJ S V<,ces ha tenido dificultdd para
agudntJrse IJ!J g~Hhl~ de orinar?
Durante Old'i o nwno, lo~ ult1moc,, 30 d1r1~, ¿cu.i nt r1s Vf'CP'i ha observado que C'I flujo
de orm11 e!!. poco fut>rte?

Du111ntP mct.., o m~no, lo -; llltimoc,, JO d1<1!1, ¿c udnta, VP<f>\ h., tPnido q11P aprPt,u o
h.iHt"r fuPr7,1 p,H,1 rnmPn:tcH a o rin ,,r 1
Ninguna 1 vez 2vez 3 vez 4vez 5 vez
Durdnlt· llld'> o nH•no<:. los ultimo'> 30 di,i-., ¿cu.u,t.,., Vt.'lP\ 'ilwle tt..>ntor quP levr1ntc1rc,,e
pctrd ormdr dec.dP qm. . SP va d 111 t anht por lt1 noclw hc1c.t,1 quP ,;,e levont,, por l,1 tndnan,1?

Puntuación: Oa 7 leve, 8 a 19 moderada y 20 a 35 grave

Muy Más bien ! Tan satisfecho \ Más bien ' Muy


Fatal
Encantado como . f h
insatisfecho , msat,s ec 0 insatisfecho
·¡ .
satisfecho satisfecho
- ---·- ·-·- ---- ----··-----·-- - -----
¿(Orno ,;e sen tiri ,1 ..,, tuv1Pri1 qut> p,1c,,,1r L~ I resto d e- l.1 vid,,
con lo'> ~intonM'> pro'ltt1tic11<; tal y <orno lo.-, tien p ahora ?

Tabla 4.2. Escala internacional de síntomas prostáticos (IPSS)


Libro CTO de Medicina y Cirugía, 6.ª ed. ENARM

Tratamiento Retención urinaria reiterada.


Hidronefrosis retrógrada (lesión del parénquima renal por obstrucción
Dentro de las posibilidades terapéuticas, la cirugía continúa siendo el único infravesical).
tratamiento definitivo para la HPB. Puede ser endoscópica (rese cción tran- Infección urinaria de repetición .
suretra l prostática [RTUPJ) o abierta (adenomectomía prostática) (Figura Litiasis vesical.
4.4), dependiendo del tamaño del adenoma. En el 10% de las piezas obte- Hematuria de repetición .
nidas, se encontrarán focos de adenocarcinoma incidental. Existen las Fa lta de respuesta a tratamiento co n médico con alfabloquedor e inhi-
siguientes modalidades de tratamiento de la HPB, según la sintomatología bidores de 5-a-reductasa .
que presente el paciente:
IPSS menor de 8. Se recomienda al paciente realizar modificaciones die-
téticas y conductuales, como disminuir la ingesta de liquido después de 4.2. Carcinoma prostático
las 18:00 horas, y evitar irritantes de la vía urinaria (alcohol, café, té ...).
IPSS entre 8-19. Se recomienda iniciar tratam iento médico, de pri- El carcinoma prostático es el tumor maligno más frecuente del aparato geni-
mera elección los bloqueantes al, dentro de los cuales están tamsu- tourinario masculino y el segundo en frecuencia general, después del pulmo-
losina, doxazosina y silodosina; y cuando la próstata mide más de 40 nar. Sin embargo, si se incluyesen los carcinomas incidentales y los encontrados
g, se recomienda combinar la terapia con inhibidores de la 5-a-reduc- en autopsia, supera al pulmonar. La prevalencia del carcinoma prostático es
tasa, como finasterida y dutasterida, que hacen efecto reduc iendo el del 59% de los hombres a los 79 años. La edad promedio de diagnóstico es a
tamaño de la próstata hasta en un 20% pero hasta los 6 meses de uso. los 68 años. Entre los factores de riesgo comprobados destacan tener circunfe-
Principales inconvenientes de los a-bloqueantes: hipotensión (más rencia abdominal superior a 102 cm, tabaquismo y antecedente de gonorrea.
frecuente en los fármacos antiguos) y la eyaculación retrógrada.
Principales inconvenientes de los inhibidores de la 5-a-reduc- El 95% de los carcinomas prostáticos son adenocarcinomas originados en
tasa: impotencia, reducción del PSA en torno al 50% (dificultando la zona periférica de la próstata. Los carcinomas ductales se originan en los
el diagnóstico del carcinoma, si lo hubiese), tarda una media de 4 conductos prostáticos en lugar de los aclnos, e histológicamente pueden
meses en hacer efecto. corresponder a carcinomas transícíonales, escamosos, endometrioides o
IPSS mayor de 20. Se pueden iniciar medicamentos para mejorar el mixtos. Más raros son los carcinosarcomas (< 1%).
vaciamiento vesical, sin embargo, la mayoría se beneficiará más con
procedimientos quirúrgicos endoscópicos urológicos que pueden ser El adenocarcinoma prostático, con frecuencia, es multifocal y presenta
por medio de resección transuretral de próstata con energía m_onopo- poblaciones en distinto grado de dife renciación. En esta heterogeneidad se
lar, bipolar o láser verde. En caso de volúmenes elevados mayores de basa la clasificación de Gleason, que asigna una puntuación de 1 a 5, según
80 gramos se opta por enucleación con láser Holmio o Tulio. el patrón histológico de cada una de las dos poblaciones más representati-
vas de la masa, sumando ambas puntuaciones para obtener un resultado
final de 2 a 10. Esta escala de Gleason se corresponde con el pronóstico de
la enfermedad, independientemente del estadio. Para la estadificación se
emplea, principalmente, la clasificación TNM (Tabla 4.3 y Figura 4 .5).

TN M
T define el tumor
T1: tumor inaparente dfnlcamente (no palpable ni visible por técnicas
de imagen):
Tl a: hallado incidentalmente. Afectación menor del 5% del tejido resecado
- Tl b: hallado incidentalmente. Afectación mayor del 5% del tejido resecado
- Tl e: tumor identificado por punción-biopsia por aumento del PSA
T2: tumor confinado a la próstata (incluye la invasión de la cápsula
prostática sin exteríoriZacíón del tumor hacía el tejido adiposo periprostático):
- T2a: menos del 5096 de un lóbulo
- T2b: más del 50% de un lóbulo
- T2c: dos lóbulos
Figu ra 4.4 . Pieza de adenomectomía prostática T3: extensión del tumor por fuera de la dpsula:
- T3a: extensión transcapsular (sea unilateral o bilatera0
- T3b: Invasión de la{s) vesícula(s) seminal(es) ·
T4_:tumorfijoolnvadeórganosadyacentesdlstlntosalasvesículasseminales
(cuello vesical, esfínter externo, recto, músculo elevador o pared pellliana)
N. dehne lc1 ..Jfe<.tJoon qJngl,onar
La finasterida también es útil para la alopecia androgénica, donde se NX: no se pueden estud!.r los gangHos regionales
emplea en dosis mucho menores. NO: no metástasis ganglionares
Nl: metástuls a ganglios regionales
Globalmente, sólo un 10% de los pacientes prostáticos precisará cirugía. La
MO: no metástasis
intensidad de las manifestaciones clín icas subjetivas y la mala respuesta al
M 1: metástasis a distancia:
tratamiento médico pueden constituir la indicación para la intervención. Ml a: ganglios linfáticos no regionales
Entre las causas "objetiva s" que suponen indicación absoluta de trata- - Mlb:hueso
miento quirúrgico y que además son las compli caciones mismas que causa - Mlc otras localizaciones
la enfermedad se encuentran: Tabla 4.3. Estadificación del ca rcinoma de próstata


.... 04. Hiperplasia y carcinoma prostático I UR

.... T: tumor primario dlnlca TN

.....
T3a

T2a T2b
TX No puede evaluar el tumor
TO No existen signos de tumor primario T3

Tla Tlb

..... T2c

...
......
< 5% >S%

T1 Tumor no evidente clínicamente, no palpable


ni visible mediante técnicas de imagen:
· T1 a Extensión menor o igual al 5% del tejido resecado
• T1 b Extensión mayor del 5% del tejido resecado
T2 Tumor limitado a la próstata
o a la cápsula, sin sobrepasarla:
• T2a Menos del SO% de un lóbulo
• T2b Más del 50% de un lóbulo
T3 Tumor que se extiende a través
de la cápsula prostática:
· Ha Extensión extracapsular (unilateral o bilateral)
• T3b Tumor invade la vesícula seminal
· Tl e Tumor identificado mediante punción biópsica · T2c Dos lóbulos

...
(consecuencia de un PSA elevado) N: ganglios linfáticos regionales

......--
... T4 Tumor ñjo o que invade estructuras adyacentes diferentes a las vesículas seminales

Figura 4.5. Estadificación del adenocarcinoma d e próstata


NX No se pueden evaluar los ganglios linfáticos regionales
NO No hay metástasis ganglionares regionales
Nl Metástasis en ganglios linfátitos regionales

Clínica México, en la actualidad, el 75% de los cánceres de próstata se diagnostica


ya en estado avanzado, por lo que sería conveniente plantearse la real iza-
Por lo general, el carcinoma prostático es asintomático, ya que se presenta ción de un screening para tratar de aproximar estas cifras a las de Estados
en la zona periférica, sin embargo, cuando produce síntomas habla de un Unidos o Europa, donde más del 70% de estos tumores se diagnostican en
estad io avanzado y puede producir síntomas obstructivos del tracto urinario fases tempranas de la enfermedad.
inferior superponib les a los de la HPB. A ellos puede añadirse la hematuría .
El 25% de los pacientes que refieren retenc ión urina ria aguda presentan un B. Marcadores tumorales
carci noma prostático. Aproximad amente un 25% de los pacientes presentan
metástasis en el momento del diagnóstico; estas pueden producir manifes - Se dispone, fundamentalmente, de dos marcadores tumorales :
taciones como dolor óseo, compresión medular, mieloptisis o coagulopatía. Fosfatasa ácida prostática (FAP). Se emplea en clínica desde hace
Afort un adamente, estos casos se encuent ran en claro descenso gracias a la décadas. Es un marcador específico, pero su elevación suele indica r
incorporación del antígeno prostático específico (APE; en inglés, prostate extensión extraprostática, por lo que no resulta útil en el diagnóstico
specific antigen [PSA]), que facilita el diagnóstico de la enfermedad en esta- precoz.
dios tempranos y comúnmente asintomáticos. Antígeno prosUtlco especifico. El PSA es realmente un marcador de
tejido prostático cuyos niveles suelen encontrarse más elevados en el
Diagnóstico cáncer, pero es inespecífico y también está elevado como consecuen -
cia de patología benigna (infecciones, sondajes, HPB ... ). Por ell o, se
ha intentado aumentar su especificidad para cáncer con otros pará -
A Tacto rectal metros (densidad del PSA, índice PSA/edad, velocidad de cambio del
PSA, PSA libre, calicreínas), y aunque aún no ha quedado esta blecida
El tacto rectal continúa siendo el método fundamental de cribado. Son acce- su ventaja sobre el PSA aislado, sin embargo, los resultados pa recen
sibles al tact o rectal todos los estadios excepto el Tl, que por definición es alentadores.
un hallazgo. Característicamente, el carcinoma es duro, nodular e irregular. PSA menor de 4 ng/ml. Es poco probable que se encuentre un
En general, se aconseja un tacto rectal y un PSA anual a todos los varones cáncer de próstata.
por encima de SO af\os y hasta los 70 af\os, aunque, de momento, la OMS no PSA mayor de 10 ng/ml. Las probabilidades aumentan, lo que
aconseja la realización de cribado poblaciona l si stemático. No obstante, en aconsejaría una biopsia de próstata ecodirig ida .


Libro CTO de Medicina y Cirugía, 6.ª ed. ENARM

PSA entre 4-10 ng/ml. El valor se encuentra en una zona gris, por D. Gammagrafía ósea
lo que los parámetros PSA libre y densidad del PSA tornan mayor
importancia para dirigir el diagnóstico. La gammagrafía ósea se utiliza para la detección de metástasis óseas. Tiene
mayor sensibilidad que la radiología convencional (Figura 4.7), y debe
C. Pruebas de imagen realizarse en todo paciente en quien se sospeche metástasis Gleason > 8,
PSA > 20 ng/rnl, estadio clínico > T2c = riesgo alto. Antes de plantearse
La ecografía transrectal (ETR) (F igura 4.6) es el método de imagen más útil el tratamiento curativo, en ciertos pacientes con altas probabilidades de
para la estadificación local, pudiendo ofrecer información importante sobre encontrarse el cáncer extendido, se debe efectuar una gammagrafía previa
la afectación capsular, de vesículas semi na les, cuello vesical o recto. Aunque para confirmar la no existencia de metástasis óseas, o una TC para descartar
no existe un patrón característico, suele aparecer corno nódulos hipoeco- metástasis ganglionares.
génicos. La ETR ofrece, además, la posibilidad de dirigir la biopsia hacia las
zonas sospechosas . La ecografía abdominal no tiene gran valor en la detec-
ción del carcinoma prostático. La TC y la RM tienen su principal papel en la
estadificación ganglionar y la valoración de metástasis a distancia. Las pri-
meras metástasis deben buscarse a nivel de los ganglios linfáticos de las
cadenas obturatrices e ilíacas.

Actualmente, existen escalas para determinar qué pacientes son de alto


riesgo para enfermedad extraprostática, como la clasificación según riesgo
D'Amico, que considera estadio TNM , niveles de PSA y resultado histopato-
lógico de Gleason de biopsia prostática.

Figura 4.7. Gammagrama óseo positivo a metástasis óseas por cáncer


de próstata. Las flechas muestran los focos de actividad osteoclástica

Las metástasis del cáncer de próstata son osteoblásticas es decir, for-


man, su actividad es mejor valorada en un garnmagrama óseo . Sitio de
metástasis más frecuente: columna .

E. Biopsia prostática

Para la confirmación del diagnóstico, debe realizarse biopsia prostática.


Puede efectuarse vía transrectal o transperineal, guiada por el tacto rectal o
por la ETR, lo que añade efectividad a la prueba. La realización de la biopsia
está indicada siempre que exista una anomalía del tacto rectal, elevación
de los marcadores tumorales o alteración en las pruebas de imagen; y debe
haber al menos 12 muestras durante la biopsia para mejorar la posibilidad
diagnóstica. La punción-aspiración con aguja fina (PAAF) es una alternativa
con menores complicaciones, pero con el inconveniente de que no puede
evaluar el grado histológico (Gleason) .

Son indicaciones de biopsia prostática el tacto rectal sospechoso, la pre-


Figura ll .6 . ETR de adenocarcinoma prostático. (A) ETR corte sencia de un nódulo ecográfico, PSA libre <10 % y PSA > 4 ng/rnl (varia-
transversal. (B) ETR corte longitudinal. (C) Adenocarcinoma, nódulo ble la cifra según criterios).
hipoecoico en lóbulo derecho (flecha)


...... 04. Hiperplasia y carcinoma prostático I UR

...... Tratamiento Quimioterapia. Está indicada en el cáncer de próstata resistente


a la castración ; se utiliza docetaxel y, de segunda intención, caba-

,,,. A. Opciones terapéuticas

Las opciones t erapéuticas son las siguientes:


Promtectomía radical. Está indicada en el cáncer de próstata
zitaxel (ambos mejoran la sobrevida).
Otros. Se están diseñando nuevas moléculas que bloquean los
andrógenos a distintos niveles, como la abiraterona y la enzalu-
tamida, utilizándose cuando el tumor se hace resistente a la hor-

,,,.
localizado, con esperanza de vida superior a 10 años. Como com- monoterapia convencional.
plicaciones, se puede encontrar incontinencia (2-57%), estenosis

......
anastomótica (10%). impotencia (50%) e incluso la muerte (< 5%). B. Tratamiento por estadios
En lín eas generales, suele ir acompañada de linfadenectomía íleo-
obturat riz . Por estadios, el tratam iento es el siguiente :
Radioterapia (RT). Como tratamiento curativo, los resultados en esta- • Estadio Tla . Tiene una mortalidad por la enfermedad del 2% a los 10

.......
dios localizados se acercan a los de la cirugía; la modalidad de elec- años, por lo que no precisan tratamiento, salvo quizá los pacientes
ción es la RT de intensidad modulada. Diarrea crónica, proctitis, cistitis jóvenes(< 60 años) con una elevada esperanza de vida .
rád ica y fístulas urinarias son complicaciones del tratamiento, así corno Estadio T1 b-Tlc. Puede ofrecerse vigilancia activa en pacientes con
incontinencia e impotencia a partir de los 2 años de tratamiento. Se bajo riesgo, sin embargo, el tratamiento de referencia es la prostatec-

......
ha empleado también RT intersticial (braquiterapia) con implantación tomía rad ical. Si hay contra indicación para cirugía, se puede ofrecer
de yodo-123 (1-123), oro-198 (Au-198). paladio e iridio. Su indicación RT; y en pacientes no aptos para ninguna de las anteriores, se ofrece
queda limitada a tumores pequeños de estadio T1 o T2, y sus resulta- tera pia hormonal.
dos son similares a los de la cirugía . En caso de compresión medular o Estadio T2a. Es la indicación más clara de prostatectomía radical. La RT
dolor por metástasis óseas, la RT sobre la metástasis puede conseguir o braquiterapia se reservaría para pacientes de riesgo quirúrgico ele-

... el control local de la enfermedad.


Vlgllancla activa. En los tumores de bajo riesgo, es posible efectuar
vigilancia con controles periódicos y, en caso de progresión, realizar
vado o que no aceptan los efectos secundarios atribuibles a la cirugía .
Estadio T2b y T2c. Un 40% demuestra ser en realidad estadio 3, tras
el análisis de la pieza quirúrgica de prostatectomia rad ical (infraesta-

--.......
tratamiento con intención curativa . dificación). La RT externa o braquiterapia también puede ser útil en
Hormonoterapia. El adenocarcinoma prostático está compuesto pacientes de alto riesgo quirúrgico.
por una pob lación heterogénea de células andrógeno-dependientes Estadio T3a . La indicación quirúrgica es dudosa, así como la RT local,
y andrógeno-independientes. La supresión hormonal frena e! cre- por lo que solamente se propondría a sujetos jóvenes, aun a costa de
cimiento de las primeras, pero no afecta a las andrógeno-indepen- obtener malos resultados. Generalmente, son tratados como el grupo
dientes. Se pueden disminuir los niveles de andrógenos circulares por siguiente .
Estadio T3b, T4. N+. M+. Varón añoso con mal estado general. El t ra-

....
distintos métodos:
castración quirúrgica. Es el método aislado más eficiente, con la tamiento hormona l es la opción indicada. Puede ser preciso el uso de
ventaja de que elimina la necesidad de medicación permanente. RT paliativa sobre la metástasis en caso de dolor. En pacientes asinto-

....,,. Por su rapidez en el efecto supresor hormonal, tamb ién está


indicada en las compresiones medulares por metástasis.
Estrógenos (dietilestilbestrol). In hibe la secreción de LH . Actual-
máticos incluso se puede realizar una actitud expectante y t ratar los
síntomas que apa rezcan conforme la evolución de la enfermedad.

C. Recidivo tumoral posterior e tratamiento

,,,.
mente, este método se ha abandonado debido al alto riesgo car-
diovascular que conlleva. con intención curativa
Progestágenos. Inhiben la secreción de LH y actúan como

,,,. antiandrógenos, al unirse a los receptores de la dihidrotestos- Tras la realización de prostatectomía radical, los pacientes son monitoriza -

,,.
terona . Es preciso añadir estrógenos pa ra evita r el fenómeno de dos generalmente con peticiones de PSA. El valor de PSA esperado tras pros-
escape, que se produce tras varios meses de tratamiento. No son tatectomía radical es de 0,2 ng/dl después de 30 días, y posterior a RT este

......
de uso habitual. se alcanza en 2 años. Cuando las cifras de PSA t ras prostatectomía radical
Agonistas LHRH. Aunque inicialmente ocasionan un aumento son superiores a 0,4 ng/ml, se considera recidiva bioquím ica y debe hacer
de los niveles de testosterona, posteriormente suprimen la sospechar la existencia de metásta sis a distancia o de recidiva a escala local.
secreción de LH y de andrógenos. La elevación transitoria de los

..
andrógenos puede empeorar el cuadro clínico, principalmente Tras la real ización de RT como tratamiento de cáncer de próstata localizado,
si existe compromiso medular por metástasis óseas. Esta eleva- los descensos de PSA van siendo paulatinos (a diferencia de la prostatecto-
ción (f)are-up) se debe suprimir mediante la administración de mía radical) hasta conseguir un valor nadir, que es el valor mínimo alcanzado

,.,,. antiandrógenos, previamente a la introducción de inhibidor de


la LHRH .
Antiandrógenos (blcalutamida, flutamida, acetato de dprote-
rona). Compiten con el receptor androgénico. Suelen utilizarse
con agonistas de la LHRH. El acetato de ciproterona, además de
tras el tratamiento y que se considerará refe rencia para el seguimiento pos-
te ri or. Generalmente, se toma el nivel de PSA posterior a 10 vidas medias
del mismo como nadir, es decir, 32 días. {Vida media de PSA 3,2 días).

Existen diferentes criterios para considerar el diagnóstico de recidiva bioquí-


actuar como antiandrógeno, tiene un efecto pnogestágeno, por mica tras tratamiento con RT:
Cuando se constata la existencia de tres elevaciones sucesivas a partir

,,.
lo que actúa a nivel central, disminuyendo los pulsos de LH .
,111 Antlandrógenos de segunda generación. Se utilizan en el cán- del valor nadir.
cer de próstata resistente a la castrac ión; com prenden enzaluta• Cuando se evidencian niveles nad ir+2 (criterio de la ASCO).
mida, apalutamida y darolutamida . Valores nadir+3 (criterio de Philadelphia).

,.
,¡a

Libro CTO de Medicina y Cirugía, 6.ª ed. ENARM

HPB Adenocarcinoma prostático

Locahzdciun

Clinica Fases: Mayoría asintomáticos:


Compensación Hasta 25%, síndrome prostático
Clínica Hasta 25%, retención aguda
Descompensaclón Hasta 25%, metástasis

Diagnóstico dif~rencial Tacto rectal


ETR (estadificación local)
Gammagrafla ósea (metástasis óseas)
PSA (muy sensible, poco éspecffico). Deséarta cáncer prostático, pero no diagnostica HPB
FAP (muy específica, poco sensible)
Biopsia (confirmación)
Tratc1miento Fitoterapla Localizado: prostatectomía radical más linfadenectomla bilateral,
F~rmacos: finastertda,c,.-bloqueantes radioterapia
Cirugía: adenomectomía: endoscópica o abierta_ _________---·-··-·-·· Avanzado: castración:_qulrúrgica (elección), farmacológica - ·--····-··· ·-
Tabla 5.4. Tabla-resumen de las características de la HPB y del adenocarclnoma prostático

D. Tratamiento de urgencia modo de síntesis de la misma, en la Tabla 5.4 se resumen las características
de la HPS y del adenocarcinoma prostático, cuyos respectivos algoritmos
La compresión medular por el cáncer prostático no tratado puede ser la diagnóstico-terapéuticos se ofrecen en la Figura 4.8 y en la Figur a 4.9.
forma de presentación y constituye una urgencia importante. El objetivo del
tratamiento debe ser la supresión androgénica rápida o la descompres ión
medular mediante laminectomía quirúrgica o RT. Se pueden disminuir los
niveles de andrógenos mediante castración qu irúrgica urgente, ketoconazol
en altas dosis o dietilestilbestrol intravenoso. Para concluir esta unidad, y a

Abordaje diagnóstico del paciente con sospecha de HP8


(>50 años)

1. Historia clfnlca (uso del IPSS y escala de calidad de vida)


2. Exploración física (exploración rectal digital)
3. Examen general de orina y urocultivo
4. Antígeno prostático específico total
S. Ultrasonido vesical y prostático con medición de orina residual postmiccional

Envíe a 2."nivel por


¿APEanormal? - - Sí sospecha de cáncer
1
de próstata

No

¿Complicaciones Evalúe el tracto J


renales por HPB? .,.__--1 urinario superior Sí ¿Creatinina anormal?i
con US renal

No
Tratamiento específico
No de la complicación Evalúe de acuerdo
No----1..i Complementación
en nivel correspondiente ¿STUI por HPB? a diagnóstico
Sí diagnóstica
diferencial
sr

Síntomas leves Síntomas moderados a severos Tratamiento


Tratamiento específico
(IPSS s 7 puntos) (IPSS .! B puntos) especifico
de complicaciones

Tratamiento quirúrgico
1. Vigilancia activa Evalúe riesgos beneficios
2. Medldas y alternativas de tratamiento
deHPS
higiénico-dietéticas

Iniciar tratamiento

Figura 4 .8 . Algori t mo para diagnóstico y tratamiento de la hlperplasla prostática


04. Hiperplasia y carcinoma prostático I UR

Paciente con sospecha de cáncer de próstata


1 1

1
APE elevado de 4 a 1Ong/ml y TR normal TRanormal 1 APE elevado de >1Ong/ml y TR normal /
1
1

i i
Fracción libre de APE > 20% Fracción libre de APE < 20%
Densidad de APE < O,15 ng/ml/cc Densidad de APE >0, 15 ng/ml/cc
Velocidad de APE < 0,75 ng/ml/año Velocidad de APE > 0,75 ng/ml/año
Tiempo de duplicación de APE Tiempo de duplicación de APE
> 3 meses > 3 meses

i
Sólo llevar monitoreo ¡ 1 1 Solicitar biopsia transrectal 1
dirigida por ultrasonido
1 con APE 1 1

! i ¡ 1
...... APE < 10 mg/ml
Gleason ,; 6 y estadio
clínico T1 a T2c
Ca de próstata localizado
APE de 1Oa 20 mg/ml
Gleason de 7 o estadio
clínico T2b a T2c
Ca de próstata localizado
APE de 20 mg/ml
Gleason de 8 a 10
estadio clínico T3
Ca de próstata localmente
APE > 20 mg/ml
Gleason de 8 a 1O
estadio clínico T4
Ca de próstata avanzado

......
Bajo riesgo Riesgo intermedio avanzado Alto riesgo Alto riesgo

! ¡ ! ¡
Vigilancia activa Se discutirá con el paciente Prostatectomla radical Hormonoterapia
Monitoreo con APE si continuar con vigilancia + hormonoterapla

...
y biopsias periódicas o realizar prostatectomía o radioterapia externa +
o radica l hormonoterapia
prostatectomía radical o radioterapia externa
o braquiterapia o braquiterapia

----
o radioterapia externa

- - -- - --- - -- -- ---- - - -- -- -- -- ------ -- -- -- -- -------


Figura 4.9. Algoritmo para diagnóstico y tratamiento del cáncer de próstata

...
~ ✓ La hiperplasia prostática benigna (HPB) suele afectar a la zona periuretral ✓ El cáncer de próstata cada vez se diagnostica con mayor frecuencia en
de la glándula. El cáncer aparece en la zona periférica. fase asintomática. Cuando presenta clínica, puede consistir en síntomas

~ urinarios similares a la HPB.

----
✓ La HPB no guarda relación con el cáncer.
✓ El PSA elevado no es diagnóstico de cáncer de próstata. Puede corres-
✓ Tanto la HPB como el cáncer tienen relación con las hormonas sexuales, ponder a una HPB. El diagnóstico delinitivo de cáncer prostático precisa
y suelen aparecer en varones ancianos. una biopsia.

..-- ✓ El tratamiento médico de la HPB consiste en a-bloqueadores (relajan la


musculatura uretral y del cuello vesical), inhibidores de la 5-a-reductasa
(disminuye el tamaño glandular) y litoterapia. Esta última no ha demos-


Las metástasis lumbares son típicas del cáncer de próstata, pudiendo
producir compresión medular.

El tacto rectal revela una próstata pétrea e irregular en el cáncer de próstata.

----
trado utilidad con parámetros objetivos.
Sin embargo,al principio puede no ser palpable, ni visible en la ecografía (Tl).
✓ El tratamiento delinitivo de la HPB es la cirugía, que puede consistir en
resección transuretral o en cirugía abierta, dependiendo del tamar'lo ✓ La principal complicación quirúrgica del cáncer de próstata es la impotencia.
prostático.

--
--..._.
✓ El cáncer de próstata es casi siempre un adenocarcinoma, con gran fre-
cuencia multifocal.
✓ Ante un síndrome de compresión medular por cáncer de próstata, nun-
ca se deben emplear análogos de la LHRH únicamente. Siempre deben
asociarse antiandrógenos.


Libro CTO de Medicina y Cirugía, 6.ª ed. ENARM

✓ En el cáncer de próstata, la indicación más ciara de prostatectomía radi- ✓ El tratamiento fundamental del cáncer de próstata diseminado es la hor-
cal es el estadio T2a. monoterapia.

Hombre de 6S años, acude a valoración médica por presentar nicturia y dis- 2) Radioterapia externa + hormonoterapia.
minuclón del chorro de la orina. Se realiza medición de antfgeno prostático 3) Prostatectomía radical+ hormonoterapia.
específico reportando 1.20 ng/ml. De acuerdo con las zonas de McNeai, . 4) Prostactectomía radical.
¿en cuál zona se da la hiperplasla prostática?
Es el lugar más frecuente de metástasis a distancia en cáncer próstata:
1) Anterior.
2) Central. 1) Pulmón.
3) Periférica. 2) Esqueleto axial.
4) Transición. 3) Esqueleto paraxial.
4) Cerebro.
No se considera criterio para cirugfa en un paciente con crecimiento prostático:
Acorde al sitio más frecuente de metástasis en cáncer de próstata, ¿cuál es
1) Sintomatología a pesar de tratamiento médico adecuado. el estudio que debe de realizarse?
2) Hematuria recidivante.
3) Uropatía obstruct1va la cual condiciona insuficiencia renal . 1) TC de cráneo.
4) Volumen prostático mayor de 50cc. 2) Biopsia renal.
3) Punción lumbar.
Paciente masculino de 65 años de edad, con hipertensión arterial sistémica 4) Gammagrama óseo.
con tratamiento con losartán, que inicia con disminución de calibre y fuerza
miccional, pujo y tenesmo. Al tacto rectal, se encuentra próstata aumenta- Paciente masculino de S6 años de edad, con antecedentes de importan-
da de tamal'lo, de consistencia dura en ambos lóbulos, superficie nodular y cia de resección transuretral de próstata con resultado histopatológico
límites mal definidos. Trae consigo examen de antfgeno prostático, el cual de cáncer de próstata. Se realiza estatlficación reportándose con Tlc,
es de 4,7 ng/ml. Acorde a estos datos, ¿cuál es el siguiente examen que se NO, MO. Estadio l. Es la estirpe histológica más común de cáncer de
debe de realizar? próstata :

1) Tomografía axial computarizada. 1) Ductal.


2) Gammagrama óseo. 2) Sarcomatoide.
3) No se requieren estudios complementarios debido a que no existen alte- 3) Adenocarcinoma .
raciones en el ant/geno prostático. 4) liposarcoma.
4) Paciente es candidato a realización de biopsia transrectal de próstata.
Según las zonas de McNeal en cuál se da el cáncer de próstata:
El paciente acude nuevamente a consulta con reporte de patologfa en don-
de estratifican Gleason 3 + 4 = 7. Conforme a este hallazgo, ¿cuál es el 1) Anterior.
tratamiento más indicado en este caso? 2) Central.
3) Periferia.
1) Hormonoterapia. 4) Transición .


,,,. 04. Hiperplasia y carcinoma prostático I UR

....
~

.... A 65-year-old man is brought to your institution complalning of voiding


problems. He clalms he has difflculty in starting micturition, weak stream
and postvoid drlbbling. He presents nocturia twice per night. Laboratory
is performed with a resultant diagnosis of prostate adenocarcinoma with
Gleason score of B (4 + 4). Bone sean shows two leslons that probably co-
rrespond to metastatic lesions In the lumbar splne and scapula. Mark the

....
test reveals PSA: 1.37 ng/ml. KUB (kidneys, ureters, bladder) ultrasound incorrect sentence about prostate cancer metastatic lesions:
study shows a normal kidney and a urinary bladder wlth a prostate that
measures 45 ce. What is the best initial treatment? 1) The vertebrae, sternum, pelvis, ribs and femur are the most commonly
affected bones.

.....
1) Th is patient does not need medica! treatment right now. 2) Osteoblastic and osteoclastic activity increase, but osteoclastic activit y
2) A prostate biopsy should be performed. is higher and hence osteolytic pattern is more frequently observed in
3) a-blocker agents should be prescrlbed . radiographic studles.
4) Finasteride treatment is indicated. 3) Pain, inflammation and pathologic fractures are frequent clinical mani-

....
festations .
A 63-year-old patient comes to the physician because he Is found to have a 4) Castration improves symptoms in up to 80% of patients with metastatic
PSA of 5.3 ng/ml on a routlne blood analysis. He reports no urlnary symp- prostate cancer.
toms or sexual dysfunction. In this particular case, which of the following
optlons is correct?

...,,,.
A 71-year-old male who has been found to have a PSA of 16 ng/ml under-
goes a transrectal blopsy that suggestes prostate cancer with a Gleason
1) The next step in the diagnostic workup should be a transrectal ultra- score of 7 (4 + 3). lnltial bone sean and abdomino-pelvic CT sean do not
sound. show any suspicíous lesions. The patient is subjected to a radical prosta-
2) The symptoms are not consistent with prostate cancer. tectomy. Five years later he complains of right scapular pain and laboratory

,,,. 3) The likelihood of having prostate cancer is approximately 25% given his
PSA level.
test reveals PSA 6.02 ng/ml. A new bone sean shows three suspicious le-
slons that suggested metastatic lesions, two of them locallzed in the rlght

,,,. 4) The next step should include a radical prostatectomy. scapula. In this case, which of the following sentences describes the best
action to take?

,,. A 71-year-old patient presents to his physician with the results of a blood

..,,.
test showing PSA levels of 15.3 ng/ml. He says he has sorne voiding pro- 1) Radiotherapy of the metastatic lesions is indicated .
blems, with frequent interruptions and dribbling. He often wakes up at 2) Complete hormone-blocking treatment should be ínitiated .
night to urinate. He says he has no hematuria, incontinence or sexual im· 3) A watch-and-wait approach see is recommended, if no pathologic frac-
pairment. Digital rectal examination reveals an adenomatous grade 111/IV tures occurred.
prostate with a suspicious left-sided nodule. An ultrasound-guided biopsy 4) Curative radiotherapy should be administered.

,,.,.
,,.,,.
,,.,,.
,.,.
,,.,.
,,.
..... •
05
Tumores renales

Esfundamental conocer la estirpe hislopalológica másfrecuente y reconocer el síndrome de


Stauffer. La esclerosis tuberosa y la enfermedad de Von Hippel-Lindau se asocian
también a otro tumor renal, el angiomiolipoma.

5.1. Carcinoma de células renales Presentación


(adenocarcinoma renal, hipernefroma) La tríada clásica de hematuria, dolor y masa en flanco aparece en el 7-10%
de los casos, e informa de estadio avanzado de la enfermedad. Con el
El carcinoma de células renales es el tumor sólido renal más frecuente aumento del uso de la tomografía y de los ultrasonidos por diferentes cau-
(90%) (Figura 5.1). Es un tumor fundamentalmente de la edad adulta, con sas (50% de los casos), se ha aumentado la incidencia diagnóstica de este
mayor incidencia entre los 40-60 años, con predominio en el varón (3:2), a tumor. Las anomalías más frecuentemente encontradas son:
excepción de la variedad cromófoba, típica de las mujeres. Hematuria (60%).
Anemia (40%) .
Masa en flanco (24%).
Hipertensión arterial (20%).
Hipercalcemia (6%).
Eritrocitosis (3%).

El tumor renal produce múltiples síndromes paraneoplásicos (30% de los


casos), siendo el más común el aumento de la VSG, por hipoxia; y en el
hígado produce aumento de las enzimas hepáticas sin evidencia de metás-
tasis (indica necrosis hepática). Casos más raros hacen síndrome PTH-like.

Se debe sospechar de un tumor renal en pacientes con varicocele recid i-


vante, ya que la causa puede ser obstrucción del flujo sanguíneo a nivel de
la vena renal. Asimismo, en pacientes con manifestaciones clínicas de dolor
abdominal y/o lumbar o hematuria macroscópica, se debe realizar explora-
ción tísica dirigida en búsqueda de tumor abdominal palpable, adenopatia
cervical. etc.

Figura 5.1. Carcinoma de células renales Los síntomas de presentación de enfermedad metastásica son el dolor óseo,
tos persistente, pérdida de peso, adenopatías no regionales, edema de
extremidades bilaterales sugestivas de involucro venoso, entre otros.
Los factores etiológicos demostrados son tabaquismo, obesidad e hiper-
tensión; además, en menor medida, se han relacionado otros factores de
riesgo, como hábitos dietéticos, exposición laboral a carcinógenos, ingesta
crónica de paracetamol y AINE, nefrolitiasis, etc. Existen formas hereditarias
que, por lo general, son múltiples y bilaterales, relacionadas con la esclero- La producción de péptidos puede aparecer en el hipemefroma, pero es
más típica de carcinomas epidermoides (pulmón, esófago ... ).
sis tuberosa y la enfermedad de Von Hippel-Lindau. Asimismo, existe una
incidencia aumentada en el riñón poliquístico, en la enfermedad quística
adquirida de la insuficiencia renal crónica y en los riñones malformados, Diagnóstico
como el riñón "en herradura".
El ultrasonido renal es la prueba diagnóstica de primera intención para
El adenocarcinoma renal proviene de células del túbulo contorneado proxi - tumoraciones renales (Figura 5.2), sin embargo, la tomografía abdomi-
mal, y microscópicamente predominan las células claras. nopélvica contrastada es el estudio de primera elección para el diagnóstico

• :j
,,.
...
~ 05. Tumores renales I UR

~
_..
de un tumor renal, ya que informa sobre el tamaño, la forma y la localización Para completar el estudio diagnóstico de extensión, es decir, búsqueda de
del t umor, así como sobre afectación ganglionar y metástasis a órganos. metástasis, es importante realizar una tomografía de tórax. Anteriormente,

...... Específicamente, en los tumores renales, no está indicada la biopsia renal para
el diagnóstico de rutina de cáncer renal , ya que ta tomografía tiene alta sensi-
bilidad y especificidad al diagnóstico sin ser un estudio invasivo (Figura 5.3).
se solicitaba radiografía de tórax para búsqueda de metástasis pulmonares.

Otras pruebas de laboratorio que deberán realizarse con fines de comple-


mentación diagnóstica, estadificación y plan de tratamiento son: biome-
tría hemática, pruebas de función hepática, deshidrogenasa láctica (DHL),
fosfatasa alcalina, calcio, magnesio, depuración de creatinina de 24 horas
en casos seleccionados con sospecha o diagnóstico de falla renal, y RM en

-"
..--...
casos clínicamente indicados.

La citología urinaria y ureteroscopia está indicada en casos de sospecha de


carcinoma uroepitelial o tumor renal central.

.......
El procedimiento diagnóstico ante la presencia de masas renales se puede
observar en la Figura 5.5 .

Mas, renal descubierta accidentalmente

...
• Examen fisico
- Análisis de orina
Figura 5.2. Ecografía de quistes renales simples

-- Quiste complejo
o masa sólida

gp Observar 1

Masa sólida

,,,.
Angiomiolipoma
o quistte complicado

,,,. Nefrectomla
radical
o parcial
No complicado:
observación
Complicado:
- Nefrectomia simple
- Nefrectomfa parcial

....,,. Figura 5.3 . TC de tumor renal izquierdo

Actualmente. la urografía intravenosa ha sido despla_zada por la tomogra-


• Embollzación

Figura S.S. Algoritmo diagnóstico de las masas renales

,,,. fía y prácticamente está en desuso para el diagnóstico de cáncer renal. La


arteriografía renal sólo se indica hoy en día en el caso de que se plantee
Estadificación

,,,. realizar tratam iento quirúrgico conservador, como es la nefrectomía parcial. Se ut1llzan el sistema TNM para estadificar la enfermedad y la escala de
Fuhrman para valorar el grado:

,,.,,,.
La angiotomografía es el mejor método para evaluar de manera adecuada
un trombo actual (Figura 5.4). • T. Tumor primario.
TO. Sin evidencia de tumor primario.
Tl. Tumor con diámetro ~ 7 cm, limitado al riñón.
Tla. Tumor con diámetros 4 cm .

,,,.
Tlb. Tumor con diámetro> 4 cm peros 7 cm .
T2. Tumor con diámetro> 7 cm, limitado al riñón .
T2a. Tumor con diámetro> 7 cm peros; 10 cm.
T2b. Tumor con diámetro> 10 cm, limitado a rií'\ón.
T3. Tumor que se extiende dentro de venas mayores o tejido periné-
frico, pero no sobrepasa Gerota ni glándula suprarrenal.
T3a. Tumor se extiende dentro de vena renal o invade seno renal
pero no sobrepasa Gerota.
T3b. Tumor se extiende dentro de vena cava por debajo del dia-
fragma .
ne. Tumor se extiende dentro de vena cava por arriba del dia -
Figura 5.4. RM de tumor renal con trombo en venas renal y cava fragma .


Libro CTO de Medicina y Cirugía, 6.ª ed. ENARM

T4. Tumor que invade más allá de Gerota o invade glándula suprarrenal. bueno o intermedio, utilizándolo el tiempo necesario para hasta no progre-
N. Ganglios regionales. sión o toxicidad .
NO. Sin ganglios invadidos.
Nl. Metástasis en uno o más ganglios linfáticos .
M. Metástasis a distancia.
MO. Sin metástasis a distancia .
Ml. Con metástasis a distancia.

Tratamiento
5.2. Otros tumores
El trata miento de elección es la resección del tumor, con todo el riñón hasta
su fascia de Gerota (nefrectomía radical) o, si es posible, sólo la extirpación Otros tumores renales son los siguientes;
del t umor (nefrectomía parcial}. La glándula suprarrenal no se extirpa, a • Tumor de Wllms (véase Sección de Pediatría).
menos que haya invasión a ella. Se debe intentar en todos los casos cirugía Tumores renales metastásicos. Pueden encontrarse metástasis en el
conservadora de nefronas, en especial en casos con tumores múltiples, bila - riPl6n de tumores de pulmón (la más frecuente}, mama, melanomas e
terales, riñón único, enfermedad hereditaria y con daño renal previo. infiltración por linfoma.
Tumores benignos:
Actualmente, la nefrectomía parcial es el estándar de oro en el estadio Tl, Adenomas corticales. Son los tumores más frecuentes del
y en casos seleccionados del T2. La linfadenectomía no está indicada de adulto, aunque indistinguibles clínicamente del adenocarcinoma,
rutina, ya que no mejora la supervivencia. Para el estadio Tla (< 4 cm}, se por lo que se tratan como tales. El criterio clásico de tamaño (3
han descrito terap ias de mínima invasión, como radiofrecuencia, crioabla- cm) para su diagnóstico diferencial no es válido en la actualidad.
ción o termoablación, pero ninguna de ellas ha sustituido a la nefrectomía Angiomiolipomas. Se asocian a la esclerosis tuberosa en un
pa rcial. Para la enfermedad avanzada, la nefrectomia citorreductora sólo 50%. Compuestos de una proporción variable de grasa, vasos y
está indicada en casos para mejorar los síntomas o en pacientes en buenas fibras musculares. Cuando son grandes(> 4 cm), pueden ocasio-
condiciones a los que también se puede realizar metastasectomía. nar un síndrome de Wünderlich por sangrado retroperitoneal.
Cuando se asocian a esclerosis tuberosa, suelen ser múltiples y
El adenocarci noma renal tiene mala respuesta a quimioterapia y radiotera- bilaterales, por lo que deben tratarse de forma conservadora.
pia. La rad ioterapia puede ser considerada para controlar la hemorragia y el Oncocltoma. Considerado benigno, aunque en algunos se han
dolor del t umor primario, aliviar los síntomas de las metástasis y controlar detectado metástasis. Hay criterios radiológicos para distinguirlo
las metástasis cerebrales. Lo único que ha demostrado ligero aumento en del adenocarcinoma, pero en la mayoría de los casos, ni estos ni
la sobrevida es la inmunoterapia, la cual ha sido desplazada por la terapia la citología o la biopsia ofrecen garantías suficientes de su benig-
blanco, que son medicamentos antiangiogénicos, por ejemplo sunitinib, nidad, por lo que tienden a ser tratados mediante nefrectomla.
que es el medicamento de primera línea en el tratamiento del cáncer renal Nefroma mesobhbtico (hamartoma fetal). Es el tumor benigno
metastásico o recurrente de células claras, en pacientes con pronóstico más frecuente en recién nacidos y lactantes.

✓ El rumor más frecuente de células renales es el de células claras. ✓ No se debe confundir un quiste simple con un hipernefroma. Los crite-
rios de quiste simple son: contorno liso, contenido transónico y refuerzo
✓ El contexto típico es el paciente varón, obeso, fumador, de la quinta o posterior.
sexta década de la vida.
✓ La primera prueba de imagen, ante la sospecha de hipernefroma, sería la
✓ La tríada clásica consiste en hematuria, dolor y masa en flanco. ·Aaual- ecografia.
mente, lo más habitual es que sea incidentaloma (asintomático}. Si pro-
duce síntomas, el más frecuente es la hematuria. ✓ El tratamiento fundamental es quirúrgico. La radioterapia y la quimiote-
rapia convencional no forman parte del tratamiento.
✓ Hay que sospechar tumor renal ante un varicocele izquierdo, de apari-
ción súbita y que no cede con el decúbito.

✓ El hipernefroma puede producir multitud de síndromes paraneoplási-


cos. Esto puede complicar bastante el diagnóstico, de ahi el sobrenom-
bre de "tumor del internista~


05. Tumores renales I UR

Paciente masculino de 52 años, con índice de masa corparal de 32.S acude Femenino de 64 af'los de edad, hemotipo A+, antecedente de hipertensión
a consulta por presencia de hematuria Intermitente formadora de coágu- arterial sistémica de 14 af\os de evolución en tratamiento con captoprll

~
los filiformes, refiere hematuria desde hace 3 meses, Se realizan estudios 50mg cada 12 horas, además de diabetes mellitus de 5 años de diagnós-
de laboratorio y gabinete observándose en ultrasonido lesión isodensa en tico en tratamiento con Insulina NPH 20 unidades par la maf'lana y 10 por
rillón derecho de 3x4 cm a nivel de polo Inferior la cual no produce sombra la noche, tabaquismo a razón de 2 caJetHlas al día, Inicia padecimiento 6
acústica pasterior, con creatlnina de 2.1 mg/dl y urea de 59 mg/dl. El estu-

......
meses antes de su Ingreso a nuestra unidad con astenia, adinamia, cefalea,

~
dio ideal para estadificar al paciente es: pérdida de peso a razón de 7 kg y dolor en flanco izquierdo, ultrasonido que
reporta tumor dependiente de rli'lón izquierdo de 100 x 95mm. ¿cuál es la
1) Urotomografía. variante histopatológica más frecuente?
2) Resonancia magnética nuclear de abdomen y pelvis .
3} Ultrasonido Doppler renal bilateral. 1) Adenocarcinoma .

--...... 4) Urografía excretora.

¿Cuál es la triada dáslca de cáncer renal y en qué parcentaJe se presenta?

1) Ictericia, dolor en hipocondrio derecho y síndrome febril 10%..


2) Liposarcoma.
3) Angiomiolipoma.
4) Nefroblastoma .

Hombre de 45 años tiene carcinoma de células renales extendido. Los nive-

...... 2) Ascitis, derrame pleural y masa anexial 20%.


3) Hipertensión, bradicardia y alteraciones de la respiración 35%.
4) Masa palpable en flanco, hematuria y dolor 10%.

¿Cuál es la estirpe histológica més común en cáncer renal?


les de AST, fosfatasa alcalina, LDH y a-2 globulina son elevados y el tiempo
de protromblna alargado. El hígado aparece difusamente agrandado, no
existen defectos focales de infiltración intrahepática. La explicación etioló-
gica más probable para estos hallazgos será:

----... 1) Angiomiolipoma.
2) Células claras.
3) Papilar.
4) Tumor de Wilms.
1) Amiloidosis.
2) Secundarismo hepático.
3) Síndrome de Stauffer.
4) Trombosis tumoral que obstruye la vena hepática.

.....
..
A 62-year-old patient comes to the hospital with complaints of difficult darker than usual and sometimes she has seen small amounts of blood In
mlcturition, weak voiding and postvoid drlbbling. Abdominal kidney uitra- her urine. Regarding this case, whlch of the following is correct?
sound reveals a 3 cm dlameter hypoecholc mass in the right kidney. Pos•
terior acoustlc enhancement is also observed. Left kldney appears to be 1) The main diagnostic suspicion is renal adenocarcinoma and the progno-

---- normal. In thls case:

1) A fine needle aspiration biopsy should not be performed.


sis is good since she presents with initial symptoms.
2) The main diagnostic suspicion is renal adenocarcinoma and intravenous
urography should be performed to complete the diagnostic study.

...
.....
2) An ultrasound-guided fine needle aspiration biopsy should be performed.
3) A CT sean guided fine needle aspiration biopsy should be performed .
4) A trucut biopsy should be performed.

An ultrasound shows a right kidney mass that is 3 cm in dlameter and


3) You should suspect a xanthogranulomatous pyelonephritis. Nephrec-
tomy is indicated dueto the patient's symptoms.
4) The main diagnostic suspicion is renal adenocarcinoma and an abdomi-
nal CT sean should be performed to study the extension of the disease .

apparently solld. left kidney seems to be normal. A CT sean is performed A rlght renal mass measurlng more than 7 cm is incidentally found durlng a

... showing that the mass is in the superior half of the right kidney having a
fat-like density, suggestlng an angiomyolipama. Which of the following is
lncorrect re¡¡ardin¡ thls case?
routlne ultrasound. Lab tests show: creatinine 0.9S mg/dl, Na: 138 mEq/L,
K: 4.1 mEq/L, AST: 293 IU/L, ALT: 324 IU/L, PA: 842 U/I, TB: 0.95, Hb: 14.2 g/
di, white blood cell count 9,600/mm•, platelets 180,000/mm•. In this case:

-- 1) We can put the patient at ease regarding his renal mass.


2) A karyotype should be performed to rule out tuberous sclerosis.
3) Annual follow-up with CT sean should be performed.
4) lt could cause Wunderlich syndrome.
1) CT sean will probably disclose metastatic lfver lesions.
2) These findings are due to liver compression by the renal mass, since it's
located within the right kldney.
3} The renal function parameters may lead us to considera partial nephrec-
tomy despite the tumor's size.
A woman presents to the clinlc with a three-month history of flank paín. 4) The hepatotoxic effect of the tumor usually disappears after the nephrec-
5he feels a mass in that area and has lost 12 kg In two months. Her urlne Is tomy.


06
Tumores testiculares

Otros factores relacionados son las hernias inguinales Infantiles (no demos-
trado) y la orquitis urliana secundaria al paramixovirus causante de la paro-
Este tema es sencillo y rentable siempre que se seleccione lo realmente importante: la clínica tiditis (siempre que haya producido atrofia) y, entre los factores tóxicos, la
yel diagnóstico. En cuanto al tratamiento, dado que no existe un protocolo universalmente exposición a radiaciones, fuentes de calor, productos para teñido del cuero y
aceptado, es mejor conocer ideas generales. estrógenos intrauterino durante el primer trimestre del embaraw.

Aunque entre el 8-25% de los pacientes presentan historia de traumatismo


6.1. Etiología y epidemiología testicular, todos los autores parecen estar de acuerdo en que este supone
más bien el motivo por el que se descubre una masa escrotal, y no su origen.
Los tumores testiculares suponen el 1-2% de las neoplasias en varones,
y son las neoplasias sólidas más frecuentes entre los 20 y 35 años. Han
aumentado en la última década en los países desarrollados y tienen máxima 6.2. Anatomía patológica
incidencia en las personas de raza blanca . Su tasa de curación es del 90%,
gracias a su diagnóstico oportuno y su alta sensibilidad a la quimioterapia Los tumores testiculares se suelen originar como una masa testicular intra-
(QT) y radioterapia (RT). parenquimatosa. A partir de ahí, pueden quedarse localizados o metas-
tatizar. Ocasionalmente surgen de forma primaria en retroperitoneo,
Son tumores procedentes de célu las germinales en el 95%, siendo el semi- mediastino y muy rara vez en la glándula pineal (tumores germinales extra -
noma el más frecuente. El 5% restante se reparte entre tumores del estroma gonadales) . La localización más frecuente de metástasis son los ganglios
gonadal (1 -2%), linfomas (1%), gonadoblastomas (células germinales y del retroperitoneales (casi siempre es el primer escalón en la diseminación);
estroma) metástasis y otros. posteriormente puede aparecer la afección mediastínica, de vísceras abdo-
minales y de pulmón.
Dentro de los factores de riesgo se encuentran los siguientes: síndrome
de disgenesia testicular (criptorquid ia, hipospadias, espermatogénesis Hay que tener en cuenta que el testículo izquierdo drena a los ganglios
disminuida o infertilidad), historia familiar de primer grado de neoplasias paraaórticos y preaórticos a nivel de L2 . El derecho drena a ganglios inte-
testiculares, síndrome de Klinefelter, presencia de tumor testicular contra- raortocavos, precavas y preaórticos, y también a nivel del hilio renal. La
lateral, neoplasia intraepitelial testicular, etc. Dentro de los factores más disem inación hematógena es menos frecuente, salvo en el coriocarcinoma,
importantes se encuentra la criptorquid ia (kriptos-oculto, orquis-testículo), vía vasos espermáticos, siendo los puntos más habituales de metástasis
con un aumento en la incidencia de entre 2-8 veces más con el resto de la pulmón, hígado, hueso y sistema nervioso central {SNC), por orden de fre-
población y siendo la situación intraabdominal del teste el factor de mayor cuencia .
riesgo. El teste contralateral, aunque de localización escrotal, también tiene
un aumento en la incidencia, pudiendo presentarse hasta en un 20% de los Los tumores testiculares se dividen en dos grupos: tumores que no derivan
casos el tumor en el teste intraescrotal. Es por esto que la anamnesis del de las células germinales (5%) y tumores que derivan de las células germina-
recién nacido es obligada, y se debe hacer una evaluación minuciosa de la les (95%); a su vez, estos últimos se dividen en seminomatosos y no semino-
presencia de los testes en la bo lsa escrotal. matosos (Tabla 6.1 ). Los no seminomatosos presentan un crecimiento más
rápido y tendencia a dar metástasis. Por su parte, los tumores de células no
A la fecha, la exploración clínica continúa siendo el método de diagnóstico germinales o del estroma tienen su origen en las células de Leydig y Sertoli,
más importante de la criptorquidia, sin embargo en exploraciones difíciles y generalmente son benignos.
se puede realizar ecografía para apoyar el diagnóstico. De notar la ausen- Seminoma. Puede aumentar el tamaño testicular hasta 10 veces sin
cia de una, iniciar el protocolo para lograr el descenso entre el primer y distorsionar su morfología. Ocasionalmente se presenta extratesticu -
segundo año de edad . El descenso del teste no disminuye el riesgo de lar en mediastino (la más frecuente), retroperitoneo, o región pineal
cáncer como tal, pero si facilita su pronto diagnóstico. Una vez alcanzada de forma primaria. Se han descrito tres tipos histológicos : seminoma
la pubertad, un teste criptorquidico no descendido probablemente deba típico, anaplásico y espermatocitico (más frecuente en individuos
extirparse, dado que estos testículos pierden la capacidad de espermato- mayores de 50 años). El seminoma puro no es productor de marca-
génesis y conservan su potencial malignizante. No obstante, hay autores dores tumorales, pero hasta en un 15% de los casos pueden aparecer
que prefieren no extirparlos, siempre y cuando se puedan descender a la células del sincitiotrofoblasto, produciendo elevaciones de la 13-HCG.
bolsa escrotal, ya que aún pueden mantener su func ión endocrina (secre- Carcinoma embrionario. Tiende a metastatizar de forma temprana .
tora de testosterona). Forma masas más Irregulares y heterogéneas que otros tumores.


06. Tumores testiculares I UR

Tumor del seno endodérmico. Fue descrito inicialmente como una


forma rara de tumor infantil. Se encuentran elementos de saco vitelino 6.3. Clínica
en el 38% de los tumores testiculares del adulto.
Coriocarclnoma. En el estudio histológico tiene que contener elemen- La presencia de una masa testicular sólida sin dolor es patognomónico de
tos de sincitiotrofoblasto y citotrofoblasto para ser considerado como tumor de testículo. Puede presentarse dolor en escroto como primer sin-
tal. En el momento del diagnóstico, generalmente existen metástasis toma (20%), dolor local (27%) y dolor de espalda y flanco (11%) . En algu-
a distancia (suelen ser vía hematógena) y un tumor primario testicular nos casos puede haber ginecomastia o bien con un cuadro similar a una
pequeño. Raramente es puro. orquiepididimitis, presentándose como síndrome de escroto agudo hasta
Teratoma. Por definición, se encuentra formado al menos por dos en el 10% de los pacientes.
capas distintas de células germinales (endodermo, mesodermo o
ectodermo).
Teratocarcinoma .Tumor mixto con áreas de teratoma y de carcinoma
embrionario. Un 64% tiene también áreas de seminoma.
Gonadoblastoma. Contiene grandes células similares al seminoma y
Existen algunas en_fermedades, como la sarcoidosis, que aumentan el
otras menores, como células de Sertoli inmaduras o de la granulosa. tamaí'lo testicular sin existir un tumor.
Se asocia preferentemente a las gónadas disgenéticas y estados inter-
sexuales.
Tumores de células de Sertoli. Forman áreas tubulares similares a los
túbulos del testículo prepuberal normal. De comportamiento benigno, 6.4. Diagnóstico
es excepcional que metastaticen .
Tumores de células de Leydig. Pueden verse cristaloides en su cito- La exploración física de una masa indolora, sin signos inflamatorios y de
plasma. Generalmente son benignos. Pueden ser hormonalmente largo periodo de evolución, debe hacer sospechar un tumor testicular
activos, produciendo pubertad precoz o feminización. (Tabla 6.2).
Llnfoma. Tanto como localización secundaria o como origen primario
(menos frecuente), una masa testicular en un varón mayor de 50 años TNM
sugiere, en primer lugar, el diagnóstico de linfoma. Tras la orquiec-
pT, s
tomía o la biopsia testicular para el diagnóstico de certeza, el trata- pTl Testículo y epidídimo sin invasión vascular/linfática
miento no varía respecto a los linfomas habituales. pT2
pTJ
Testículo y epidfdirno con invasión vascular/linfática o túnica vaginal
pT4 Afectación de cordón espermático

Nl Ganglios menores de 2 cm
Tumores de células de Leydlg N2
Tumores de células de Sertoli Ganglios entre 2·5 cm
N3
Seminoma: Tumores estructuras g~ales Ganglios mayores de S cm
- Típico primitivas
• i\naplásico MlJ Me~stasis en ganglios no regionales o pulmón
• ··Espermatocftico Mlb Me~stasis viscerales no pul manares
No semlnomatosos: s, Marcadores tumorales séricos no disponibles
Carcinoma embrionario
so
Pollembrloma
Tumor del saco vitelino (seno
endodérmico)
Sl
S2 •
...
Niveles de marcadores normales
111

LDH entre 1,5 x n y 1Ox n o ¡3-HCG entre 5.000 y 50.000 o AFP entre 1.000
Coriocardnoma y 10.000
53
- Teratoma:
• Maduro LDH ;, 1O x n o ¡3-HCG ;, 50.000 o AFP > 10.000
• Inmaduro n = limite superior normal de LDH
» Con transformación maligna

Twnor~s de rn ..,~ d'--' un trpo


h1~tolo<1 1co
Estadio O
Estadio 1
Teratocarclnoma
Otros Estadio IIA
Tumores germ1nalec;. t es tr on1,1 lec;. Estadio 118 Cualquier pT

Gonadoblastoma Estadio IIC Cualquier pT


Estadlolil Cualquier pT Cualquier N.
Tabla 6.1. Tumores de testículo. Clasificación histológica
• No tiene en cuenta marcadores tumorales
Tabla 6.2. Estadificación clínica y patológica

Un elemento importante, tanto para su diagnóstico como para el segui-


miento, son los marcadores tumorales (en conjunto, el 70% de los tumores
El tumor de células de Sertoll se ha asociado al síndrome de Peutz-
Jeghers testiculares producen algún marcador, luego existe hasta un 30% de tumo-
res con marcadores negativos al diagnóstico) (Tabla 6.3):


Libro CTO de Medicina y Cirugía, 6.ª ed. ENARM

a-fetoproteína (AFP). Es sintetizada por células del saco vitelíno y, por genérico que se denomina •escroto agudo", caracterizado por el aumento
tanto, está presente en tumores de saco vitel ino o seno endodérmico, doloroso de volumen del contenido escrotal, acompañado o no de signos
y en los carcinomas embrionarios. El seminoma nunca produce AFP. inflamatorios (Figura 6.1 y Tabla 6.4).
En cualquier caso, se debe tener en cuenta que la AFP es un marcador Orqulepidldlmltis. Suele presentarse con dolor intenso, enrojeci-
inespecífico, y se podría encontrar elevado en enfermedades hepáti- miento cutáneo, fiebre, y a veces sintomatologla miccional. En la
cas benignas o malignas, algunas neoplasias pancreáticas y de la vía exploración, la elevación del testículo (signo de Prehn) disminuye el
biliar o en la ataxia telangiectasia. dolor (signo de Prehn positivo) . En su etiología se deben considerar
Fracción Pde la gonadotropina coriónica humana (13-HCG). Es produ- gérmenes de transmisión sexual en pacientes adultos menores de 35
cida por las células del sincitiotrofoblasto presentes en el coriocarcinoma años, y uropatógenos (E. coli) si superan esta edad.
y, también de forma ocasional y de forma aislada, en algunos seminomas. Torsión del cordón espermático. Suele aparecer en la infancia o la
Deshidrogenasa láctica (DHL). Es un marcador de destrucción tisular, adolescencia, con dolor de aparición brusca y signos cutáneos infla-
elevado en el 80% de los tumores avanzados. Tiene relación con el matorios crecientes a medida que progresa el cuadro. El teste se
tamaño tumoral y es de mayor utilidad para la vigilancia. encuentra horizontalizado, y ocasionalmente puede palparse la espiral
Fosfatasa alcallna placentaria (FAP). Puede elevarse en el seminoma del cordón torsionado. En este caso, la elevación del testículo incre-
puro. Casi en el 40% de los casos se eleva en enfermedad avanzada. menta la sensación dolorosa.
Hidrocele y espermatocele. Cuadros que raramente se presentan de
La vida media de la a-fetoproteína es de 7 días, frente a 3 días de la ~-HCG . forma brusca y con dolor agudo, fácilmente diferencia bles por la explo-
Este dato es importante a la hora de valorar la posible presencia de enfer- ración y su transiluminación positiva y, ante la duda, por ecografía.
medad residual, ya que la elevación persistente de uno de estos marcadores
después del tratamiento supone la existencia de tumor no eliminado.

Tumores germinales: marcadores tumorales


Tumor con una palabra aumenta ~HCG:
- Coriocarcinoma
- Seminoma (en la mujer, el equivalente es el disgerminoma)
Tumor con dos palabl'IIS aumenta a-fetoprotefna:
- Carcinoma embrionario
- Tumor del seno endodérmico
Tabla 6.3. Marcadores tumorales en tumores germinales (válidos.para
ovario y testículo)

La AFP también se eleva en líquido amniótico en alteraciones del cierre


del tubo neural.

La ecografía testicular es un método sencillo y fiable para la diferenciación


entre masas sólidas y quisticas, y su localización exacta intratesticular o depen-
diente de los anejos. Cuando, a pesar de los marcadores, los datos ecográffcos
son sugerentes de tumor, está indicada la exploración quirúrgica a través de
una incisión inguinal, para evitar la posibilidad teórica de Implantes tumorales Figura 6. 1. Pieza macroscópica de tumor testicular
en la piel escrotal y tener mejor control de pedículo vasculolinfático a nivel del
cordón espermático. Si la exploración confirma la presencia de una masa, el Orquitis Polaridad conservada
testículo debe ser extirpado de forma urgente (orquiectomía radical) . · Prehn MEJORA el dolor
Dolor
Torsión del cordón Testfculo horlzontalizado
La evaluación de la extensión tumoral se completará mediante TC toracoab- espermático Prehn EMPEORA el dolor
dominal. Así se constatará si la enfermedad está limitada al testículo (esta- Sin dolor Tumor testicular Masa palpable Indolora
dio 1), o existe afectación de ganglios infradiafragmáticos (estadio 11), o bien Tabla 6.4. Diagnóstico diferencial de los tumores testiculares
si hay incluso extensión supradlafragmática o a órganos sólidos (estadio 111) .
El sistema de estadificación utiliza múltiples variaciones, pero quizá la clasi-
ficación más aceptada sea la expuesta en la Tabla 6.2. 6.6. Tratamiento
Hasta la fecha, no existe un protocolo único de tratamiento, y este puede
6.5. Diagnóstico diferencial variar incluso de un centro a otro. Siempre se realizará orquiectomía radical
vía inguinal. Posteriormente, el patólogo informará de la estirpe histopa-
Aunque frecuentemente la existencia de un tumor testicular no plantea tológica. A continuación, el estudio de extensión mediante TC toracoab-
dudas diagnósticas, existe una serie de patologías testiculares que, junto dominopélvica y nuevos marcadores postorquiectomía. En función de la
con los tumores, pueden presentarse en algún momento dentro del cuadro histología y del estadio, se asignará un tratamiento a cada paciente .


06. Tumores testiculares I UR

Estadio llc-111. Antes de la aparición de la actual QT, la supervivencia


era del 5-10%. Actualmente, el esquema de tratamiento mayoritaria-
mente aceptado es QT primaria .
La orquiectomía en el cáncer de testículo es vía inguinal y de carácter
urgente. En el cáncer de próstata, vía escrotal bilateral y con cirugía
El tratamiento de la neoplasia testicular se resume en la Figura 6.2.
programada

Tumor testicular
Tratamiento del seminoma
Orqulectomia radical
El sem inoma se caracteriza por su gran radiosensibilidad, de ahí que la RT
haya sido la base del tratamiento de estos tumores. Actualmente, la QT
Seminoma Noseminoma
obtiene resultados sim ilares.
Estadio l. El tumor teóricamente está limitado al testículo. No obs-
tante, se sabe que hasta un 20% de pacientes presentan micrometás- IA

tasis a ganglios retroperitoneales, que en el momento del diagnóstico


no se detectan. La presencia de micrometástasis se ha relacionado con · Observación Radioterapia · Observación ++++ Quimioterapia
· Si FR: > 4 cm Quimioterapia • Unfadenectomía Invasión ~ - - -~
dos factores de riesgo: tumor de más de 4 cm e invasión tumoral de rererestis (EEUU) vascular
la ret e testis . En estos pacientes se aconseja administrar RT o QT (cis- • Quimioterapia o
(UE) linfática
platino). Con esto, frente a los pacientes que optaron únicamente por
• Radioterapia
observación, la recidiva desciende del 20% al 5%. • Quimioterapia Seminoma / No semi noma
IIC-111
Estadio lla-llb. En este caso, el tumor ya está extendido a ga nglios
retroperitoneales, y requie re, por tanto, tratam iento agresivo. Se dis-
Quimioterapia
pone de dos opciones : RT sobre las cadenas afectadas (teniendo en
cuenta el teste afectado, se irradiará a unas cadenas ganglionares u
Figura 6.2. Algoritmo de tratamiento de la neoplasia testicular
otras), o QT BEP (cisplatino, etopósido y bleomicina}. Ambos obtienen
resultados muy similares.
Estadio llc-111 (estadios avanzados). El tumor tiene metástasis ganglio-
nares retroperitoneales superiores a 5 cm, o afectación supradiafrag-
mática o de vísceras sólidas . La RT deja de ser una opción terapéutica .
La QT (BEP) es la única posibilidad, presentando una tasa de curación
de alrededor del 80%.
Tratamiento de las masas residuales
Se define como masa residual la existencia de conglomerados adenopáti-
cos tras tratamiento quimioterápico o radioterápico. Su tratamiento depen-
derá de si el tumor primario es un seminoma o un tumor no seminomatoso
El cisplatino produce vómitos con mucha frecuencia. Otro efecto secun-
(Figura 6.3):
dario es su nefrotoxicidad.

Masa residual
Tratamiento de los tumores
no seminomatosos No semlnoma 1

El t ratamiento de los tumores no sem inomatosos es el siguiente:


Estadio l. Existen tres posibilidades terapéuticas tras la orquiectomía :
Observación y seguim iento periódico. Tasas de recaídas del 20%.
~
Linfadenectomía retroperitoneal de estadificación, pues así se
tiene certeza del estadio y se reducen al 10% las recidivas (en -----'-~ ~- ± PET+ ----'--~
Europa no está extendida esta práctica) . Observación ..--0 ' ~
Qu imioterapia profiláctica (cisplatino) . Así se reducen al 5% las Exéresis de la masa residual
recidivas.
En aquellos casos en los que exista invasión vascular en la pieza de
50 % necrosis 135 % teratoma j 15 % tumor viable
orquiectomía, la tasa de recidivas asciende al 50%. Parece lógico, en
estos casos, inclinarse de entrada por una de las dos últimas opciones. Figura 6.3. Algoritmo de tratamiento de las masas residuales
La tasa de curación alcanza el 98%.
Estadio lla-llb. Históricamente se realizaba linfadenectomía retroperi- Cuando el tumor primario es un seminoma :
tonea l completa como único tratamiento, pero ante tasas de recidiva Si la masa residual es inferior a 3 cm , tiene muy pocas probabili-
no desdeilables, actualmente se indica QT de inicio (BEP}. La tasa de dades de contener tumor residual y no requiere más que obser-
supervivencia supera el 95%. vación .


Libro CTO de Medicina y Cirugía, 6.ª ed . ENARM

Si la masa residual es superior a 3 cm, se debe realizar una La histología de estas masas residuales, una vez extirpadas, puede ser tejido
tomografía por emisión de positrones (PET). si existiera esta necrótico (hasta en un 50% de las ocasiones). tumor viable (15%) y teratoma
posib ilidad a nivel técnico, pues detecta con alta sensibilidad (35%). Estos, dejados a libre evolución, pueden convertirse en teratomas
y especificidad la presencia de tumor residual. Si no se dis- malignos o producir procesos compresivos con su crecimiento .
pone de PET, o si esta es positiva, se realizará cirugía de la
masa.
Cuando el tumor primario es un tumor no seminomatoso, se debe
realizar exéresis de la masa residual siempre, con independencia del
tamaño.

✓ El tumor testicular es la neoplasia sólida más frecuente en el varón joven. ✓ El seminoma nunca produce a-fetoproteína.

✓ La tasa de curación es mayor del 90%. ✓ Como tratamiento, la orquiectomía por vía Inguinal se realiza en todos
los casos.
✓ El tumor testicular más frecuente es el seminoma. No obstante, esto es
muy variable según la edad del paciente. ✓ El seminoma es radiosensible. Por ello puede utilizarse radioterapia
como tratamiento en los primeros estad ios. Si se trata de un estadio
✓ Una masa testicular por encima de los SO años debe hacer pensar en avanzado, se emplearía quimioterapia.
un linfoma.
✓ Los tumores no seminomatosos se tratan con orquiectomía + quimiote-
✓ El tumor típico de las disgenesias gonadales es el gonadoblastoma. rapia. No obstante, si es un tumor limitado al testículo, puede plantearse
la vigilancia tras la orquiectomía.
✓ La clínica más frecuente es una masa escrotal indolora.

Masculino de 21 al'los de edad, con antecedente de importancia criptorqui- Posterior a orquiectomía radical derecha se confirma el diagnóstico y se
dia, acude por detectarse crecimiento testicular derecho, no doloroso a la estratifica en una etapa clínica 1 sin invasión a rete testis, tumor de 3 cm,
exploración. ¿Cuál es la probable etiología del padecimiento del paciente? ¿Cuál es el tratamiento Indicado en este caso?

1) Orquiepididimitis. 1) Vigilancia.
2) Torsión del cordón espermático. 2) Radioterapia .
3) Hidrocele. 3) Quimioterapia asociada a radioterapia.
4) Seminoma. 4) Quimioterapia .


._,,
,. 06. Tumores testiculares I UR

e:,,.
,,,. A 35-year-old male presents to your outpatient clinic with a three-month
history of painless swollen testicle. Tumor markers a-FP y 13-HCG are eleva-
Which of the followlng options Is the most likely diagnosis of a patient pre-
senting with a scrotal mass that feels like a bag of worms?

-,,,,.,_
ted. Which of the following is the most approprlate answer about this case?
1) Pure seminoma .
1) The most likely diagnosis is apure seminoma. 2) Varicocele.
2) The most likely diagnosis is a yolk sac tumor. 3) Hydrocele.
3) Tumor markers can be used for follow-up. 4) Testicular torsion .

......
4) The elevated 13-HCG is directly related to the presence of hematogenous
metastatic lesions. A 32-year-old patient reports to the physician a three-week history of pal-
pable lump in his testicle. Physical examination revea Is increased size of the
A 32-year--old patient presents to his physicían saying that his testicles have right testicle with a painless petrous nodule. Testicular echography revea Is
increased in size fer the last two months. Physical examlnation reveals a an intraparenchymatous lesion that measures 2 cm and has no echogeni-
palpable lump in his right testicle. Ultrasound shows an intraparenchyma- clty. Which of the following tests ar maneuvers wouid not be indicated in

.... tous lesion with calcifications. Mark the correct answer: this patient?

....,,,. 1) An inguinal orchiectomy is indicated .


2) A transscrotal orch iectomy is lndicated.
3) A contralateral testicle biopsy should be performed.
1) Thoraco-abdomino-pelvic CT sean in arder to stage the disease.
2) A blood sample in order to determine 13-HCG levels.
3) Schedule a radical orchiectomy as soon as possible.

,,,. 4) The patient should receive neoadjuvant chemotherapy. 4) Perform a bi lateral testicular biopsy.

,.--,,,.
,,,,.
,,,.
,,.,,.
,,.,,.
,,.
,,,.
,,,.
--,,,.
,.,..
~
_. •
Carcinomas del
07
tracto urinario

Histología e historia natural


Es fundamental la parte de tratamiento. Haciendo referencia al carcinoma de células transicionales, hay que dife-
renciar tres formas de la enfermedad con comportam iento, pronóstico y
tratamiento completamente distintos (Tabla 7.2 y Figura 7.1 ). El 75% son
7 .1. carcinoma vesical superficiales y se encuentran localizados en la mucosa. Un 10% son sóli-
dos, con invasión tentacular en profundidad y extensión linfática y vascular
El carcinoma vesical es la segunda neoplasia urológica en frecuencia, Aparece temprana . El 20% restante son formas mixtas. El primer grupo suele corres-
más frecuentemente en varones (2-3 :1) y más en población blanca que negra. ponder a tumores superficiales de bajo grado histológico, mientras que los
Su edad de máxima incidencia se sitúa entre los 60-70 años. De ellos, el 90% sólidos, con mayor frecuencia, son tumores infiltrantes de grado histológico
son carcinomas transicionales, el 8% escamosos y el resto adenocarcinomas. más elevado.
El epitelio urotelial recubre el tracto urinario desde las papilas caliciales hasta
la uretra prostática, ambas inclusive. En cualquiera de estos niveles pueden La principal característica de los tumores papilares superficiales es la
desarrollarse los tumores uroteliales, correspondiendo la mayoría a la vejiga recurrencia, que ocurre en un 50-75%, según el grado y estadio. El 25%
(> 90%) y, más raramente, al tracto urinario superior (5%) o la uretra (1%). recurrirán y progresarán en grado y estadio, y únicamente el 15% acabará
desarrollando un tumor infiltrante o metastásico. La mayoría de los tumores
Entre los factores etiológicos (Tabla 7.1 ), se implican las aminas aromáti- infiltrantes se encuentran confinados a la vejiga en el momento del diagnós-
cas, presentes en las industrias textiles, químicas y del caucho. El humo del tico, y sólo un 20-25% presentan extensión ganglionar o metastásica. El 50%
tabaco es el principal factor de riesgo (el 50-60% aparecen en fumadores), desarrollarán metástasis a distancia, a pesar del tratamiento.
aumentando el riesgo a mayor consumo . También pueden jugar un papel
importante los edulcorantes artificiales (sacarina, ciclamato), la ciclofosfa- TNM

mlda, los acetiladores lentos (mayor riesgo) y muchas otras posibles etio- T d pf1ru>+=>ltun1or
logías. La infestación por Schistosomo hoemotobium aumenta la incidencia lis: carcinoma In situ (plano)
de carcinoma escamoso vesical, así como la presencia de infección crónica Ta: carcinoma papilar no infiltrante
o catéter vesical permanente. Tl: tumor que invade tejido conjuntivo subepitelial
T2: tumor que Invade músculo:
• T2a: tumor que invade la mitad interna
El adenocarcinoma primario vesical es un tumor raro, aunque es el que se - T2b: rumor que invade la mitad externa
ha visto asociado a la extrofia vesical con mayor frecuencia . T3: tumor que Invade tejido perivesical:
- T3a: microscóplcamente
- T3b: rnacroscópicameme (masa extravesical)
Carcinomas Factores etiológicos T4a: rumor que invade próstata, útero o vagina
Transicionale <. Aminas aromáticas (2-naftilamlna): tabaco, Industria T4b: tumor que invade pared pélvica o pared abdominal
(90~ o) textil, industria del caucho, colorantes N d e fl! w l.1 nf..-c t rtc ,,m q,.;nqhon ,u
{mejor pronó\t1c o¡ Fenacetinas crónicas
Sacarina, ciclamato Nx: metástasis ganglionar regional desconocida
Clclofosfamlda (acrolelna) NO: ausencia de metástasis ganglionar regional
Tabaco: ortofenoles. triptófano N1 : metástasis a un solo ganglio entre 2 y 5 cm
N2: metástasis en un ganglio mayor de 5 cm o múltiples no mayores de s cm
Escdmosos (8% ) Schlsrosoma hoemarob/um N3: metástasis mayores de 5 cm
Litiasis, Infecciones. catéteres
Adcinocarcinom ,1.: . Cistitis glandular
(2 'l,,) Extraña vesical • MO: no metástasis
• M1: metástasis a distancia
Tabla 7.1, Factores etiológicos de los carcinomas del tracto urinario
Tabla 7.2. Estadificación del carcinoma vesical

El tercer grupo, que merece mención aparte. es el carcinoma in situ. A pesar


de encontrarse limitado al urotelio, por lo que es superficial, está formado
por células poco diferenciadas con displasia grave. Tiene una alta tasa de
No hay que confundir Schistosomo haematobium con Schistosoma
manson/, que produce hipertensión portal. recidiva y progresa hacia tumor infiltrante en el 50-75% de los casos. Este
mal pronóstico le conftere un carácter completamente distinto del carci-


07. Carcinomas del tracto urinario I UR

noma in situ de otras regiones, en las que se considera el estadio inicial de la TC, la urografia o la biopsia múltiple. Son especialmente útiles en el segui-
la enfermedad tumoral. El carcinoma in situ puede estar asociado a focos de miento de pacientes sometidos a resección transuretral en combinación
carcinoma superficial (26%) o infiltrante (60%), o encontrarse de forma ais- con la cistoscopia .
lada, siendo generalmente multifocal tanto en vejiga como en otros puntos
del urotelio. Debe tenerse en cuenta que el carcinoma urotelial que infil- Entre las pruebas rad iológicas destacan la ecografía (con una sensib ilidad
tra músculo lleva un protocolo de manejo totalmente diferente al músculo del 80%, pero poco útil para el diagnóstico de las neoplasias del tracto uri-
invasor. Es por eso que incluso en la literatura se manejan como dos enfer- nario superior {Fig u ra 7.21) y la urografía intravenosa (capaz de detectar la
medades di ferentes. presencia de tumor en el 60% de los casos) . Actualmente desplazada por la
urotomografía.

En la urografía intravenosa, además de la presencia de defectos de reple-


ción, se puede sugerir el diagnóstico de tumor vesical, rigidez y falta de dis-
tensibilidad vesical, la obstrucción de un uréter o el desplazam iento de la
vejiga, entre otros. Actualmente, la TC helicoidal con contraste intravenoso
en fase de eliminación aporta imágenes más precisa s. En caso de dudas
sobre el tracto urinario superior, se recurrirá a la pielografía retrógrada en
el momento de realizar la cistoscopia . Esta es fundamental para la evalua-

~
ción del tumor vesical ; puede realizarse bajo anestesia local cuando exis-

_,. tan dudas con las pruebas real izadas previamente, pero si el diagnóstico de
presunción es firme, y dado que en todo tumor debe realizarse resección

,,,,. transuretral para evaluar el grado de infiltración, se puede esperar a tener

,,,.
al paciente en quirófano bajo anestesia general o raquídea para practicarla.

En un 10% de los casos de cistoscopia con citologías positivas no se encuen -

,,,. 1. Mucosa - - - - - - - Ta, Tls 4.Grasa -----► T3


tra tumor en la vej iga, lo que puede ser debido a la presencia de carcinoma

...,,.
2. Submucosa Tl S. Órganos vecinos -► T4
in situ, tumor en vías urinarias altas, carcinoma ductal de próstata o a un
3. Muscular T2 falso positivo de la prueba (generalmente por inflamación de la pared
vesical o por tratamiento concomitante con rad ioterapia o quimioterapia
Figura 7.1. Esquema de la estadificación del tumor vesical
endovesical) . La RM consigue mejores imágenes de la cúpula vesical por sus
cortes sagitales, pero no aporta mayor información que la TC.
En el ámbito vesical, se pueden encontrar distintas lesiones benignas que
no se asocian con el desarrollo de cáncer: los nidos de Von Brunn, la cistitis La cistoscopia con luz blanca es el gold standard para diagnóstico de tumo-
quística y glandular originados en procesos inflamatorios o irritativos cróni- res uroteliales a nivel vesical.
cos, y que probablemente sean distintas manifestaciones de un mismo pro-
ceso, aunque pueden plantear el diagnóstico diferencial (ocasionalmente
se han descrito adenocarcinomas vesicales asociados a la cistitis glandular) .
Otras lesiones benignas serían el adenoma nefrogénico, el pólipo simple, el
papiloma invertido y el papiloma velloso .

Diagnóstico
La hematuria macroscópica o microscópica monosintomática es el hallazgo
más frecuente, presente en el 75% de los pacientes. La presencia de micro-
ematuria asintomática, descubierta durante estudios de cribado, sólo
se relaciona con enfermedad significativa en menos del 2% de los casos.
Pueden encontrarse síntomas irritativos (escozor, polaqu iuria, tenesmo) en
e 25-30%, solos o acompañando a la hematuria . La presencia de un sín-
d orne cistítico no justificado por infección o litiasis debe hacer sospechar la
¡;¡-esencia de un carcinoma vesical, especialmente por su asociación con el
carcinoma in situ. Con menor frecuencia, el paciente consulta por dolor en
aneo por obstrucción ureteral, pélvico o por edema en miembros inferio-
es (extensión linfática) . La exploración física suele ser irrelevante, sa lvo en Fi gura 7 .2 . Ecografía. Tumor vesical con zonas calcificadas
a enfermedad avanzada.

s ci tologías urinarias son una prueba sencilla y fiable que debe ser rea- Tratamiento
¡ ada en todos los casos de hematuria asintomática o sospecha de tumor
. esica l. Su sensibilidad depende del grado de diferenciación del tumor vesi- Todo el proceso diagnóstico va encaminado a establecer si el tumor vesical
C21, alcanzando el 75-100% en tumores de alto grado y carcinoma in situ, es superficial o infiltrante, ya que el tratamiento varia radicalmente en fun-
, e do en este último un método diagnóstico más rentable que la ecografía, ción de este hecho.


Libro CTO de Medicina y Cirugía, 6.ª ed. ENARM

Los tumores superficiales son manejados mediante resección transuretral. vejiga, y hasta el 60% de los tumores uroteliales altos son invasivos al diag-
Dada la alta frecuencia de recidivas, la mayoría se tratan posteriormente nóstico comparados con el de vejiga (sólo el 15-2S%).
con instilaciones endovesicales (QT o inmunoterapia local) que disminuyan
la aparición de nuevos tumores . Entre los quimioterápicos empleados se Su pico de incidencia aparece a los 70-90 años. En el 70-80% de los pacien-
encuentran la mitomicina, la tiotepa, la adriamicina o la epirrubicina. Con tes aparece hematuria macroscópica, siendo el dolor cólico por obstrucción
todos ellos se consigue reducir las recidivas alrededor de un 20%. ureteral la segunda queja en frecuencia de aparición.

La inmunoterapia endovesical con bacilo Calmette-Guérin (BCG) es, sin


duda, la más eficaz, disminuyendo el porcentaje de recidivas en un 40%, y
es también tratamiento de elección en el carcinoma in situ, con una eficacia
del 70%. Sin embargo, no se utiliza en los tumores vesicales como primera La hematuria también es la manifestación más frecuente del tumor ma-
línea, reservándose para los tumores de riesgo o con múltiples recidivas, ya ligno de riñón .
que también se acompaña de mayor número de efectos secundarios y de
complicaciones, entre los que se encuentran cistitis febril, síndrome pseu-
dogripal, y las más graves de sepsis, prostatitis granuloma tosa, neumonitis Diagnóstico
e incluso muerte. Estos cuadros precisan tratam iento tuberculostático com-
pleto al menos de 3-6 meses. La sistemática diagnóstica es básicamente la misma que para el tumor
vesical. Es decir, la ecografía como estudio inicial (detecta hidronefrosis),
En la enfermedad invasora o infiltrante, tras la resección transuretral para sin embargo, el estudio de elección es la urotomograña en fase de eli-
evaluar la afectación parietal, el tratamiento de elección es la cistectomía minación para valorar los defectos de llenado compatibles con un tumor
radical. La QT sistémica, adyuvante o neoadyuvante, no parece mejorar los (Figura 7.3) .
resultados de la cirugía aislada. Igualmente se puede decir de la RT, que
únicamente ha demostrado incrementar el tiempo libre de recidiva local, sin En estos tumores , la citología urinaria aumenta su eficacia si se obtiene de
aumentar la supervivencia . forma selectiva, cateterizando el uréter del lado afectado. Otros elemen-
tos de diagnóstico son las biopsias por cepillado y la ureteropieloscopia,
La QT combinada con M-VAC (metotrexato, vinblastina, adriamicina, cis- elemento que, además de diagnóstico, puede tener un fin terapéutico en
platino) se reserva para pacientes en los que existe afectación ganglionar tumores de pequeño tamaño y aspecto superficial (Figura 7.4) .
o metastásica, con respuestas completas en el 15-30% y parciale~ en el
30-40%. Actualmente, los tratamientos basados en gemcitabina y cisplatino
han mostrado menores efectos adversos que la terapia clásica M-VAC.

Tras la cistectomía, los uréteres son derivados generalmente a segmentos


intestinales o a piel, pudiendo realizarse estomas no continentes o reservo-
rios continentes directamente al remanente uretral.

La adriamicina (doxorrubicina) es un quimioterápico que puede produ-


cir cardiotoxicidad.

7 .2. Tumores del tracto urinario Figura 7 .3. TC en fase excretora con tumoración plélica derecha

superior Tratamiento

Los tumores uroteliales del tracto urin ario superior (uréter y pelvis renal) El tratamiento estándar es la resección en bloque del riñón con su uretero
comprenden hasta el 10% de todos los tumores uroteliales, es decir, en y su sitio de llegada a la vejiga, es decir, la nefroureterectomía radical con
vejiga aparecen el 90%. los tumores en la pelvis renal son dos veces más rodete vesical. La QT y la RT no tienen papel de importancia en este tipo de
frecuentes que los del uretero. Son mucho más agresivos que el cáncer de tumores .


--
07. Carcinomas del tracto urinario I UR

--
Fumador+ hematuria

Sospecha de tumor urotelial

-- Cltologfas + pruebas de imagen

No concluyente Concluyente

-.....
para el tumor vesical para el tumor vesical

..
Biopsia vesical 1

Sospechar tumor
Tracto urinario superior
• Ureterorrenoscopia

.....
· Pielografla retrógrada
· Otologlas selectivas Cistoscopia
· Cepillado ureteral + citología Recidiva Tis Cistectomfa Recidiva

Figura 7.4. Algoritmo diagnóstico-terapéutico en el tumor de vejiga

,,,. ✓ El carcinoma vesical más frecuente es el urotelial, siendo el tabaco el


principal factor de riesgo.

El carcinoma escamoso se relaciona con la esquistosomiasis (S. haema-



Cuándo se trata de un CIS: síntomas irritativos (polaquiuria, disuria, te-
nesmo ... ).

El mejor método para la estadificacíón local es la resección transuretral.

... ✓
robium).

El adenocarcinoma vesical se relaciona con el antecedente de extrofia


✓ Prueba más sensible para el diagnóstico de CIS: citología urinaria.

- ✓


vesical.

El carcinoma papilar superficial y el carcinoma in situ (CIS) son muy re-


currentes.

Clínica más frecuente del carcinoma urotelial: hematuria, más típico con


Conducta ante un CIS: tratar con bacilo Calmette-Guérin (BCG) y revisio·
nes (cistoscopia y citologías).

Actitud ante un tumor superficial: resección transuretral. Posteriormen-


te, revisiones (cistoscopia y citologías).

coágulos . ✓ Actuación ante un tumor infiltrante (afectación capa muscular): cistectomía.

...,,. Masculino de 64 a/los de edad, tabaquismo suspendido hace 3 meses, ¿Cuál es el examen inicial a realizar en este paciente?

,,,.
acude a consulta de manera subsecuente por continuar con polaquiuria
y disuria. Además, menciona que presentó hematuria macroscópica con 1) USG.
presencia de coágulos la cual se autolimitó. ¿Cuál podría ser la causa del 2) TC.
cuadro clínico de este paciente? 3) RMN.
4) Cistoscopia.
1) Urolitiasis.

...,,,.
2) Cistitis por Klebsie/la. ¿cuál es el examen definitivo para realizar el diagnóstico de este paciente?
3) c_arcinoma de células transicionales.
4) Cistitis gonocócica . 1) USG.

,,,. •
Libro CTO de Medicina y Cirugía, 6.ª ed . ENARM

2) TC. 1) Obesidad.
3) RMN . 2) Hipertensión arterial.
4) Resección endoscópica con toma de biopsia . 3) 1nfección por Asearis.
4) Tabaquismo .
Acorde a las estadísticas, ¿cuál es el carcinoma más frecuente del tracto
urinario superior? Es la estirpe histológica más común de cáncer de vejiga :

1) Tumor de células escamosas. 1) Epidermoide.


2) Carcinoma urotelial. 2) Adenocarcinoma .
3) Adenocarcinoma . 3) Células transicionales.
4) Carcinoma de células pequeñas. 4) Células en anillo.

Paciente masculino de 57 años de edad, con antecedentes de importancia Es un medicamento utilizado para el tratamiento con inmunoterapia intra-
de tabaquismo intenso (indice tabáquico de 43 puntos) y diabetes mellitus vesical en cáncer de vejiga :
en adecuado control (última HbAlc de 5.6%), acude a consulta por presen-
cia de hematuria total formadora de coágulos amorfos, la cual remite en 1) Docetaxel.
48 horas. Se realiza ultrasonido vesical observándose presencia de masa 2) Adriamícina.
intravesical de 2x3 cm móvil, con flujo sanguíneo detectado por ecografía 3) BCG.
Doppler. Es el factor más importante para el desarrollo de cáncer vesical: 4) Metotrexate.

A 56-year-old male presents to his physician with complaints of self-limited 4) Urologic echography is recommended.
hematuria with no accompanylng symptoms. He says he had a similar epi-
sode slx weeks ago. He has been smoking more than one pack a day for 35 A patlent who underwent a transurethral resection of a urinary bladder
years. He has hypertension and Is currently seeing a neurologist beca use he lesion has been subsequently prescribed BCG immunotherapy. Which of
has severe headaches. Regarding thls patlent's dlagnostlc workup, which of the following is the incorrect answer?
the following clauses is correct?
1) BCG immunotherapy produces only local symptoms that are very well
1) lf he has an urothelial tumor, cytology will be always positive. tolerated.
2) A negative echography rules out a urinary tract tumor. 2) The mechanism of actlon of BCG consists of an immune-boosting
3) A negative cystoscopy rules out the presence of a papillary tumor in the effect.
urinary bladder. 3) BCG therapy should not be used in immune-deficient patients such as
4) Cytology is only indicated if we have more than one negative cystoscopy, patients with AIDS.
in arder to rule out in situ carcinoma. 4) BCG therapy should be indicated in patients with intermediate risk tu-
mors and elevated risk of recurrence or progression.
A 58-year-old female complains of irrltatlve mlcturitlon syndrome for the
last few months. Severa! urinary cultures have been performed with nega- A 65-year-old male undergoes a transurethral resection of a urinary blad-
tive results. She usually smokes, has type 2 diabetes and hypertension. Uri- der tumor. Pathology report describes the lesion as a T2G3. Which of the
nalysis shows microhematuria. Which of the following optlons is incorrecti' following is the correct management of this patlent?

1) lf the cytology is positive, stopping smoking would help diminish the re- 1) BCG therapy should be initiated.
currences . 2) Close follow-up .
2) A transurethral resection may be necessary in order to stage the disease. 3) Radical cystoprostatectomy.
3) A urine PCR test should be performed. 4) A complete staging study should be performed .


Uropatía obstructiva
08
rro, goteo term inal, hematuria, escozor al orinar, orina turbia,
retención aguda de orina o incontinencia paradójica (" micción
Se debe tener una idea general yaprenderse bien las Ideas clave. por rebosamiento" ).
Tracto urinario superior (uréter y ril'lón). Estos pacientes pre-
sentan dolor en flanco (riñón y uréter proximal), dolor en flanco
con irradiación a genitales (uréter med io) o síndrome miccional
(uréter terminal) .
8.1. Características
Después de resolverse una obstrucción, sobre tod o si es crónica, puede
Las principales características de la uropatía obstructiva son las siguientes: producirse una fa se de pol iuria . Esto se debe a que, a nivel tubular,
Detención del flujo de orina en cualqu ier punto del sistema urinario cuando se ralentiza cró nicam ente el flujo ur inario, se genera una insen-
entre los cálices y su sa lida por el meato uretral (Tabla 8.1 ). sibilidad a la ADH transitoria (diabete s in sípida nefrogén ica), de ahí la
Dependiendo de la severidad de la obstrucción y de su duración, poliuria.
se puede ocasiona r una insuficiencia renal. Es ahí la importancia de
detectar rápidamente dicha patología.

...
Una obstrucción con duración de entre 4-6 semanas puede dar lugar a
un daño renal funcional y estructural permanente .

En pacientes menores de SO años, sospechar litiasis renoureteral; en


pacientes mayores de SO años, sospechar crecimiento prostático.

La uropatía obstructiva puede producir glomerulonefritis focal y seg-


mentaria .
8.3. Diagnóstico
Si bien el ultrasonido es el estudio diagnóstico inicial por su accesib ilidad,
8.2. Clínica inocuidad y precio, tiene una baj a se nsibilidad y especificidad, por lo que
convierte a la tomografía en el estud io de elección, que cuenta co n sensibi-
La forma de presentación depende de los siguientes factores : lidad por encima del 90% y permite, además, guiar opciones terapéuticas a
Etiología de la obstrucción . Presenta la clínica propia de la enferme- seguir en el manejo de la uropatía . Además, son útiles anamnesis y explo-
dad de base . ración física, radio logía simple (conveniente en litiasis rad ioopaca), urogra-
Tiempo de evolución. La aguda suele cursar con dolor {cólico nefrí- fia intravenosa {confirma una posible anoma lía funcional y es útil en litiasis
tico), siendo la crónica más frecuentemente asintomática. radiotransparente), cistografia, estudio metabólico (útil en prevención de
Lugar de obstrucción : recidiva s), TC abdom inal, ecografía transrectal, biopsia prostática dirigida,
Tracto urinario inferior (uretra y vejiga). Cursa con retraso para pielografía retrógrada, nefrostografía, cistoscopia, flujometría, cistomano-
iniciar la micción, disminuc ión de fuerza y del tamaño del cho- metría y citología urinaria.

lntraluminal lntra arieta! Extrapanetal . Extraparietal


p (compres,on extrinseca) (d1sfunc1on neuromuscular)

Litiasis Estenosis congénita Uréter retrocavo Vejiga neurógena


Tumores (hlpernefrona urotelloma) Estenosis postinfecciosa Riñón en herradurá Vejiga automática: lesión sobre L1-l3
Necrosis papilar Estenosis postraumática Fibrosls retroperítoneal Vejiga átona: lesión bajo L1-L3
Coágulos Estenosis isquémica Tumores (próstata, vejiga, Disfunción de la unión pieloureteral
ginecológicos) Reflujo vesicoureteral
Hiperplasia prostática
Ligadura iatrógena de uréteres
----- -·· ·------•-····-····--·- · - - - - -
Tabla 8.1. Clasificación de la uropatfa obstructiva


Libro CTO de Medicina y Cirugía, 6.ª ed. ENARM

8.4. Tratamiento Manejo de la uropatla obstructiva

Es ne cesario restablecer el flujo urinario. La mayor parte de las veces se rea-


liza mediante manejo endoscópico (litotricia, litolapaxia, resección transu- lnfravesical Supra vesical
retral o colocación de catéteres) o corrección quirúrgica (ureterolitotom fa, (globo vesical) (no globo vesical)

prostatectomía suprapúbica ... ).


Sonda vesical o Ecografía renal
cistostomía
Si la obstrucción es aguda y/o bilateral, la desobstrucción es urgente y
puede lograrse mediante un sondaje vesical, catéter uret eral o nefros-
t f t
No dilatación Dilatación Dilatación
tomfa, sobre todo cuando se sospeche sobreinfección, lo que obliga a unilateral bilateral
diferir el tratamiento definitivo, y realizar un drenaje y manejo con anti-
bióticos. En caso contrario, hay que valorar el grado de sufrimiento renal 1 Estudio médico 1
y su reversibilidad . En los casos en los que hay destrucción irreversible

~~
de la vía urinaria, es necesario realizar una derivación urinaria definitiva
(Figura 8. 1).
r - ' - - ~- ~
Estudio (UIV, TCJ Fracaso renal

Derivación de vía
Pacientes con litiasis renoureteral que presenten datos de infección urinaria superior
(EGO patológico, leucocitosis, fiebre), sólo derivar la v/a urinaria y diferir (nefrostomía/doble J)
tratamiento definitivo.
Figura 8.1. Procedimiento de actuación frente a la uropatía obstructiva

✓ La uropatfa obstructiva puede producir insuficiencia renal si no se re- ✓ Después de resolver una uropatía obstructiva puede producirse una fase
suelve a tiempo. de poliuria.

✓ El dolor suele estar presente en la obstrucción aguda. Sin embargo, en la


crónica, es frecuente su ausencia.

A 78-year old patient is brought to the emergency department, complai- 1) A suprapubic cystostomy should be performed.
ning of urlnary incontinence. He says he has no trouble to start urinating, 2) Foley catheterization should be tried .
however he complains about presenting lncontinence ali day long. He re- 3) Anticholinerglc agents are indlcated.
fers that few years ago he had similar symptoms and had notable improve- 4) Pelvic floor exercise are recommended.
ment after lnltlatlng treatment with tamsulosin. In this patient:


..,,,.
,.
.... 09
.... Disfunción eréctil

..... .. , ,.,,.. -·

...... Fármacos. Aquellos que causan hiperprolactinemia, que disminuyen

....
los niveles de testosterona, pslcótropos y antihipertensivos (Tabla 9.2).
Se debe incidir enlos factores de riesgo, eltratamiento ysuscontraindicaciones. Consumo de drogas (cocaína, heroína ... r.
Trastornos afectivos (depresión ...).

9 .1. Introducción Enfermedad cardÍCllascular (hipertensión,

.....
cardiopatía lsquémlca, vasculopada periférica)
La disfunción eréctil (DE) se define como la incapacidad persistente o recu- Diabetes mellitus
rrente para conseguir o manten er la suficiente rigidez del pene que per- Hiperlipldemia
mita una relación sexual satisfactoria, con un impacto físico y psicosocial Tabaquismo
que repercute en la calidad de vida. Debe t ener una duración mínima de 3 Cirugía pélvica mayor (PR) o radioterapia
meses.
Desórdenes degenerativos (esclerosis múltiple,
enfermedad de Parkinson, etc.)
Trauma o enfermedad medular
9 .2. Prevalencia

... En Esta dos Unidos, en un estud io en varone s de entre 40- 70 años, se estimó
q ue la prevalencia global era del 52% .

9 .3. Etiología (Tabla9.1)


Tumores del sistema nervioso central
Diabetes mellitus 1 y 2

Grugía de uretra
Hipospadias, epíspadías
La etiología de la DE puede clasificarse de la siguiente forma :

..,.
1. Organica Enfermedad de Peyronie
l. Vasculogénica
Hipogonadismo
A) Arteri ogénica
Hiperprolactinemla
B) Cavernosa

....
C) Mi xta Hiper e hipotiroidismo

11. Neurogénica Hiper e hipocortisolismo


(enfermedad de Cushing)
111. Anatómica
IV. Endrocrinologica Antihlpertensivos (diuréticos tiazídicos, etc.)
Antidepresivos

.....
2. Psicógena
l. Generalizada Antlpsicóticos
11. Situacio nal Antiandrógenos
Drogas recreacionales (alcohol, heroína, cocaína,
marihuana, drogas sintéticas)
9 .4. Factores de riesgo
,.
..,.
Los principales fa ctores de riesgo de DE son :
Edad. Factor independiente. Tabla 9.1 . Causas de disfunción eréctil
Diabetes. Es la enfermedad endocrina más frecuente asociada a DE.
Signifi ca una probabilidad tres veces superior de presentar DE. Están

....
implicados mecanismos vasculares, neuropáticos y disfunción ganada!.
Enfermedad cardlovascular. Cardiopatía, hipertensión arterial, enfer-
medad vascular periférica y descenso del colesterol HDL se han rela-
La disfunción eréctil es un factor de riesgo para desarrollar enfermedad
cionado de manera clara con la DE. cardiovascular.
Tabaquismo. Factor independ iente .

.. •
Libro CTO de Medicina y Cirugía, 6.ª ed. ENARM

Administración concomítante con nítratos o fármacos donadores


de NO por el riesgo de hipotensión grave (dinitrato/mononitrato
de isosorbida, molsidomina, nicorandil, nitroglicerina, nitropru-
Diurético ahorrador de K
siato sódico).
Antagonistas centrales simpático
Pacientes en los que esté desaconsejada la activi dad sexual
Antagonista simpático periférico
Vasodilatadores (angina inestable, insuficiencia cardíaca o infarto reciente, hace
Alfa bloqueadores menos de 6 meses).
Alfa-beta bloqueadores Las dosis de medicamentos inhibidores de fosfodiesterasa 5 son las
Beta bloqueadores siguientes:
Sildenafilo 50-100 mg diarios.
Vardenafilo 20 mg 3 veces por semana.
Fenotlazlna
Vardenafilo 5 -10 mg diarios.
Tadalafilo 30 mg ínter-diarios
Tad alafilo 5 mg diarios.
Antagonistas H2 Apomorfina. Agonista dopaminérgico que actúa a nivel central sobre
Antagonistas dopaminérgicos el mecanismo de la erección. Está contraindicado en sujetos que ten -
Antlandrógenos gan desaconsejada la actividad sexual.
Análogo LHrh
Bomba de vado. Permite la congestión sanguínea de los cuerpos caver-
nosos, con el cierre de estos mediante un anillo. Pese a ofrecer buenos
resultados, son poco usadas por sus múltiples efectos adversos.
Tabla 9.2. Fármacos causantes de dísfunción eréctil
Terapia de ondas extracorpóreas. Terapia aún sin evidencia suficiente
para su total recomendación.
9 .5. Diagnóstico
Segunda línea
El diagnóstico debe basarse en los siguientes componentes:
Historia clínica y sexual. Investigar los posibles factores de riesgo Terapia íntracavernosa. Alprostadil (PGEl), median te inyección
implicados. directa en los cuerpos cavernosos. Otros fármacos son la papaverina
EMploración física. Encaminada a descartar enfermedad vascular, y la fentolamina.
enfermedades neurológicas, trastornos genitales y endocrinopatfas.
En varones mayores de 50 años se incluirá tacto rectal. Tercera línea
Determinaciones analíticas. Glucemia basal, perfil lipídico, testoste-
rona total y libre, y prolactina. Además, es conveniente solicitar hemo- Prótesis de pene (abordaje penoescrotal o suprapúbico).
grama, función renal y hepática. Cirugía de revascularización.
Pruebas especializadas. Prueba de rigidez nocturna, prueba de inyec-
ción intracavernosa, ultrasonido Doppler del pene, arteriograña y eva- Identificar causas curables de disfunción eréctil
luación psiquiátrica . Estas últimas, únicamente en casos seleccionados.
L Modificar o eliminar factores de riesgo
Educación y terapia a paciente y pareja

9.6. Tratamiento Favorecer acuerdos de pareja y fomentar


(Fígura9.1)
_ _ __ ___l_riic~?_?~tratamiento juntos
.J
Los pacientes deben recibir una valoración de riesgo cardiovascular: a los
lnhibidores de fosfodiesterasa o dispositivos
de bajo riesgo, se les puede instaurar ei tratamiento de inmediato; a los de de constricción al vacío
riesgo intermedio, se les debe realizar una prueba de esfuerzo y con base al
resultado, iniciar manejo o enviar al cardiólogo; a todos aquellos con riesgo Evaluar eficacia. Evaluar efectos adversos
alto, se les debe enviar al card iólogo previo a iniciar manejo.
Respuesta inadecuada o reacciones adversas serlas o intolerables
El tratamiento se puede estructurar en tres escalones o etapas, que serán
superados en función del fracaso del escalón previo y que siempre deben ir E}-{ _ ______Te_r_ap_i_a lntracavernosa
acompañados de cambios de estilo de vida respecto a los factores de riesgo
modificables. Respuesta inadecuada y efectos adversos serios o intolerables

Fármacos orales Considerar prótesis peneana

Fig ura 9.1. Tratamiento de la disfunción eréctil


lnhibidores de la fosfodiesterasa S (sildenafllo, vardenafilo, tadala-
fllo). Constituyen el tratamiento de elección. Inducen la relajación de
músculo liso del cuerpo cavernoso, liberando óxido nítrico (NO). El NO
es el principal neurotransmisor de la erección. Precisa deseo sexual
y estimulación previa para que tenga efecto. Las contraindicaciones
absolutas del slldenafilo son:
t4',, P7J1:i
---· ----- -------- - --------
*@ • ; - -- -~" --- -- - - ..,..-
.1


09. Disfunción eréctil I UR

~
✓ La causa más frecuente de disfunción eréctil es la vascular. ✓ El sildenañlo está contraindicado en pacientes que toman nitratos o
fármacos donadores de óxido nítrico, en pacientes con iníarto agudo
✓ La enfermedad endocrina más relacionada con ella es la diabetes me- de miocardio en los últimos 6 meses, y en pacientes con insuficiencia
llitus. cardíaca grave o angina inestable.

e:,,.
.... Paciente de 63 afias, en tratamiento a demanda con citrato de slldenafilo
por presentar disfunción eréctil de al'los de evolución. Sel'lale cuál de los
1) Diabetes mellitus .
2) Enfermedad de Parkinson.

..,. siguientes fármacos NO asociaría en ningún caso a su tratamiento: 3) Tumor de pene .


4) Adicción a cocaína.
1) Verapamllo .
2) Digoxina. El mecanismo de acción del sildenafllo, es por medio de:

,,.,,.
3) lndapamida.
4) Mononitrato de isosorbida. 1) Inhibición de la 5 fasfodiesterasa .
2) Antagonista del receptor alfa 1 específico.
Es el factor de riesgo más importante para el desarrollo de disfunción 3) Agonista del receptor alfa 1 específico.

,,.
eréctil: 4) lnhibidor competitivo de la enzima 5 alfa reductasa.

..,,,.
,,.,,. A 65-year-old patient, with a history of hypertension, goes to his physi-
cian asking far a diagnostic study far hls erectile dysfunction. Which of the
following options would not be included in the lnitial dlagnostic workup?
1) Most patients with erectile dysfunction will be treated with non-specific
treatment or therapies .
2) The basic diagnostic workup of such a patient should include the identi-
fication of reversible rlsk factors.

,,,.
1) Sexua l history. 3) Lifestyle changes and controlling risk factors have no role in the current
2) Digital rectal examination. treatment of erectile dysfunction.

,.,. 3) EKG.
4) Measuring his total and free testosterone levels.

A 78-year-old male goes to your outpatient clinic complaining of erectile


4) The early use of an elevated dose of sildenaftl after a radical prostatec-
tomy is associated with preservation of smooth muscle fibers in the hu-
man cavernous bodies.

,.,. dysfunction. He has never taken any specific medical treatment. Mark the
incorrect statement:

,,,.
....,,,.
,.,,.
....,.

10
Infertilidad masculina

10.2. Clínica
Es fundamental conocer las causas de infertilidad, así como recordar al varicocele como la Durante la exploración clínica, la anamnesis debe ir dirigida a identificar
principal causa de infertilidad de origen testicular. factores de riesgo o hábitos de comportamiento, haciendo especial énfa-
sis en el sistema reproductor (ITS, criptorquidia) y respiratorio (infeccio-
nes de repetición que orienten hacia fibrosis quística), así como preguntar
10.1. Generalidades por cirugías prevías, sobre todo cirugías en área inguinal, retroperitoneal
o genital.
La infertilidad se define como la incapacidad para lograr un embarazo
espontáneo después de un año de relaciones sexuales sin protección. Casi El examen físico debe iniciar con la inspección general del paciente; exami-
15% de las parejas experimentan infertilidad, y de este grupo, alrededor del nar el pene, incluyendo la posición del meato urinario y estigmas de ITS;
20% tienen un factor masculino como único responsable, el cual contribuye palpar los testículos, evaluando tamaño, consistencia o presencia de zonas
a la infertilidad de pareja en el 50% de las ocasiones. Cuando se está frente induradas; los epidídimos deben ser identificados, eval uando la presencia
a un factor masculino, casi siempre se observará una alteración cuantitativa de dilatación, induración o dolor a la palpación . Por último, es necesario
o cual itativa de uno o más parámetros seminales. identificar ambos conductos deferentes, puesto que la ausencia de estos
puede ser causa de azoosperm ia obstructiva por una agenesia congén ita
Las causas de infertilidad masculina varían de manera extensa (Tabla 10.1). bilateral de conductos deferentes.
Pueden ser congénitas o adquiridas, así como secundarias a patología
pre-testicular (eje hipotálamo-hipófisis-gónadas), testicular o post-testicular
(desde testículo hasta conductos eyaculadores). Sin embargo, en ocasiones 10.3. Diagnóstico
no puede identificarse el motivo de la infertilidad o de un análisis de semen
ano rmal, y se le considera idiopática. Se asume que la infertilidad idiopática Espermatobioscopía
está causada por múltiples fac tores, como alteraciones endocrinas, estrés
oxidativo, alteraciones genéticas y epigenéticas, entre otros. El análisis de los parámetros seminales es fundamental para obtener infor-
mación sobre la producción de espermatozoides y la permeabilidad del
Causas pre -testiculares aparato reproductor; aunqu e, no es una medida de la fertilidad . Un análisis
Hipogonadismo hipogonadotrófo congénito anormal de semen lo único que sugiere es la probabilidad de menor ferti-
Patología hipofisiaria: tumores, enfermedades infiltrativas e infartos lidad. En la Tabla 10.2 se exponen los valores normales de los parámetros
Patología suprarrenal: tumores e hiperplasia suprarrenal congénita medidos en la espermatobioscopia , de los cuales, los relacionados con la
Infecciones sistémicas, virales, tuberculosis cifra y la movilidad son los que se correlacionan mejor con la fertilidad .
Neoplasias sistémicas
Abuso de esteroides anabólicos
En la espermatob ioscopía, un volumen sem inal bajo de eyaculado(< 1.5 mi)
Causas testiculares puede indicar eyaculación retrógrada, recolección incompleta, obstrucción
Varicocele de los conductos eyaculadores o deficiencia de andrógenos. La movilidad
Síndrome de Klinefelter de los espermatozoides se evalúa de acuerdo con la fracción de esperma-
Mlcrodeieciones del cromosoma Y
tozoides que se está moviendo, y con la calidad del movimiento (rapidez y
Tumores: de células germinales, células de Leydig o de Sertoli
Falla testicular idiopátlca rectitud con la que nadan).
Daño vascular o traumático
Orquitis prevía Los diferentes tipos de alteraciones reportadas en la espermatobioscopía o
Exposición a gonadotoxinas: quimioterapia, radiación, fármacos. espermiograma son:
Calor
Normozoospermia: número total de espermatozoides (o concentra-
Causas post-testiculares ción ), movilidad y morfología normales.
Obstrucción epididimaria o de conductos deferentes (por causa congénita, Aspermia: ausencia de emisión de semen (puede ser o no por eyacu-
iatrogénica o postinflamatoria) lación retrógrada) .
Obstrucción de conduaos eyaculadores Oligozoospermia: concentración espermática baja (15 millones/mi) .
Disfunción sexual o eyaculatoria
No se debe diagnosticar azoospermia (ausencia de espermatozoides
Tab la 10.1. Causas de infertilidad ma sculina en el eyaculado) hasta centrifugar la muestra . Si poster iormente se


10. Infertilidad masculina I UR

_. confirma la ausencia de espermatozoides, se debe continuar el estu-


dio para diferenciar si es que se está frente a una azoospermia obs-
Pruebas especializadas

...,,. tructíva o no obstructiva.


Astenozoospermia: movilidad progresiva disminuida (<32%) o una
movilidad total <40%.
Teratozoospermia: se define como <4% de formas normales, v se ha
Las pruebas especializadas se encargan de valorar la integridad del DNA. Las
dos más utilizadas son la prueba de estructura de la cromatina espermática
(SCSA) y el marcaje de sitio terminal a través de deoxinucleotidil transfe-
rasa (TUNEL). La primera es un ensayo de citometría de flujo que mide la

_.
relacionado con fracaso en técnicas de reproducción asistida como fragmentación y madurez del ADN espermático; esta prueba se completa
fertilización in vitro o inseminación intrauterina. con espermatozoides congelados (sin crioprotectores), crudos recolectados

,,. Criptozoospermla: concentración de espermatozoides de O mill/ml,


pero tras la centrifugar la muestra se encuentra algún espermato-
después de dos o tres días de abstinencia sexual. En cuanto a la prueba
TUNEL su función es detectar fragmentación de ADN marcando el frag-

......
zoide. mento terminal de los ácidos nucleicos.

Parámetro Valor

V•ilun;en <-t•rn1 n,il 10.4. Tratamiento

..
N tot,11 dt• espt--r matcJZOHif!C.. >39 mili/eyaculado
Con(entrdc1o n de e!lµe rm dt ozoide s >15milVmL varicocele

..,. Mov1/1d ac1 tota l 1 p rngr e~i v.1

Mov i1,ddd proqrt-'~ivct


t n o progre~1va)

V1t,1 l 1dr1d :t.'~ PL' rm utozo1de'"> vivuc,:-

Morfolog1a 1fnrmilc. normalf''>•

µH ~t:'ITl Íll d l
.r, •·,
Consiste en una dilatación excesiva del plexo venoso pampiniforme del
cordón espermático, que puede ser identificado clínicamente o a través de
visualización de reflujo venoso en ecografía Doppler. Frecuentemente apa-
rece en la pubertad precoz, cuando las funciones endocrinas y exocrinas de
los testículos aumentan de manera importante, junto con el flujo de sangre
testicular. La prevalencia es de 12 a 15% de los adolescentes masculinos. Se
,i/1 Leucoc,tos <1 milVmL
Tabla 10.2. Parámetros de espermatobioscopía. Valores de referencia
considera la causa, corregible quirúrgicamente. más frecuente de infertili-

_.,,,
dad masculina.
,i/1 (OMS2010).

En su etiología, se destaca el aumento de presión en la vena renal izquierda


causada por anastomosis venosas colaterales e incompetencia valvular.
Anatómicamente, la vena espermática izquierda drena a la vena renal
izquierda, mientras que la vena espermática derecha drena directamente
De los parámetros medidos en la espermatobioscopía, la cifra total y la en la vena cava.

~
movilidad son los que se correlacionan mejor con la fertilidad.
El varicocele se puede clasificar en:
Subclinico: No se puede observar ni palpar. Solo se identifica con eco-
~ Evaluación endocrina grafía Doppler testicular.
Grado 1: Palpable solo con maniobra de Valsalva.
~
_.. Evaluar el eje hipotálamo-hipófisis-gonadal aporta información valiosa
sobre el estado de la producción de espermatozoides, puesto que las altera-
• Grado 2: Palpable en reposo, pero no visible.
Grado 3: Visible y palpable en reposo.

_.. ciones de este eje son causa frecuente de infertilidad masculina. A través de
una adecuada evaluación, se pueden diferenciar fallas testiculares primarias
(hipogonadismo hipergonadotrófico) de fallas hipofisiarias o hipotalámicas
(hipogonadismo hipogonadotrófico).
La exploración ffsica correcta sigue siendo la referencia del diagnóstico
la evaluación básica con FSH y testosterona, está indicada en los pacientes de varicocele.
que presenten alteraciones en la espermatobioscopía, en la función sexual
(baja libido, disfunción eréctil). o ante hallazgos clínicos sugestivos de endo-
crinopatía. En estos, si el nivel de testosterona es bajo, es necesario comple- El tratamiento del varicocele se realiza por medio de varicocelectomía y las
tar el estudio con hormona luteinizante y prolactina. indicaciones son 4:
l. Dolor testicular intenso y persistente.
Pruebas genéticas 2. Aumento del volumen testicular o escrotal (ocasionando molestias
severas en la zona inguinoescrotal).
En pacientes con alteración de la espermatogénesis, son comunes las anor- 3. Atrofia o contracción testicular (consecuencia del déficit circulatorio) .
malidades cariotipicas y microdeleciones del cromosoma Y. pudiéndose 4. Presencia de varicocele (grado 3).
observar alteraciones cromosómicas, como deleciones, translocaciones,
duplicaciones e inversiones en el 6% de los hombres con infertilidad. Es El tratamiento puede ser por vía laparoscópica o vía inguinal. la técnica más
recomendable solicitar consejeria genética, cariotipo v microdeleciones del utilizada es la ligadura venosa vía inguinal mediante microscopio quirúrgico.
cromosoma Y a todos los pacientes con oligozoospermia severa (< 5 millo- las complicaciones de la cirugía pueden ser: hematoma, lesión nerviosa,
nes/mi) o azoospermia no obstructiva. lesión arterial e infección.


Libro CTO de Medicina y Cirugía, 6.ª ed. ENARM

Hipogonadismo En casos de obstrucción epididimaria o deferencia!, el sitio preciso de obs-


trucción puede ser determinado a través de una vesículo-deferentograffa o
un nivel de testosterona disminuido, aunado a valores bajos de FSH y LH exploración quirúrgica . La cirugía de corrección debe ser rea lizada por un
son compatibles con hipogonadismo hipogonadotrófico, pudiendo ser con- cirujano con experiencia en microcirugía .
génito (p . ej. síndrome de Kallma n), adquirido (p. ej. tumores hipofisiarios)
o idiopático. Los adenomas hipofisiarios productores de prolactina son una Azoospermia no obstructiva
causa frecuente de esta condición, pudiendo ser tratados con fármacos ago-
nistas dopaminérgicos como la cabergolina. Existen múltiples condiciones que producen falla testicular, que a su vez
pueden dar lugar a una azoospermia no obstructiva. Este tipo de azoos-
Azoosperm ia obstructiva permia refleja un daño severo de la espermatogénesis, y clínicamente se
caracteriza por testículos pequeños y blandos, asociados a FSH elevada. Su
Esta obstrucción puede ser a nivel intratesticular, epididimaria, deferencia! tratamiento se ha revolucionado desde que se describió la extracción esper-
o en los conductos eyaculadores, y en ella se observan testículos de tamaño mática desde testículo (TESE), pudiéndose optimizar al realizarla con micro-
normal, epidídlmos dilatados, bajo volumen seminal y FSH normal. Sus disección (m-TESE). En esta técnica, se pueden reconocer visualmente los
dos tratamientos principales son la corrección quirúrgica y la recuperación túbulos seminíferos que más probablemente contienen espermatozoides,
espermática seguida de inyección intracítoplasmática de espermatozoides . para su posterior extracción.

✓ Existe un factor masculino en aproximadamente la mitad de las parejas ✓ Teratozoospermia se define como menos de 4% de formas normales.
in fértiles.
✓ La evaluación endocrinológica se realiza por medio de: testosterona,
✓ La ausencia de ambos conductos deferentes puede estar en el contexto FSH, LH y prolactina.
de agenesia congénita bilateral de conductos deferentes (CBAVD).
✓ El varicocele se considera la causa, corregible quirúrglcamente, más fre-
✓ Oligozoospermia se define como una concentración espermática baja cuente de infertilidad masculina.
(15 millones/mi).

Paciente masculino de 28 años de edad el cual acude a consulta por pre- 1) Grado t.
sencia de dolor a nivel de testículo izquierdo, A la exploración ffsica se logra 2) Grado 11.
palpar presencia de varicocele con la maniobra de Valsalva . El grado de 3) Grado 111.
varicocele de este paciente es: 4) Subclínico .


,,..,.
.... 11
....
..,,. Trasplante renal

....
.... Lo más Importante de este tema son los tipos de rechazo. Se debe insistir en el rechazo agudo.
Expectativa de vida reducida por enfermedad de base del
paciente.
Presencia de anticuerpos preformados frente a antígenos del
donante.

.... 11.1. Indicaciones


Contraindicaciones relativas. Se han ido modificando a lo largo de
los años, gracias a la mejora en las terapias de inmunosupresión y al
trabajo multidisciplinario que han mejorado la técnica y los cuidad<?s
prequirúrgicos y posquirúrgicos. En muchas ocasiones, el trasplante

....
Las dos enfermedades que más comúnmente abocan a una insuficiencia plantea menor riesgo que una hemodiálisis crónica. Actualmente, se
renal terminal irreversible, tratable mediante un trasplante renal, son la consideran contraindicaciones relativas las siguientes:
glomerulonefritis y la diabetes mellitus insulinodependiente. Otras causas Edad avanzada .
importantes son: Oxalosis.

..,.
Poliquistosis renal. Amiloidosis.
Nefroesclerosis hipertensiva. Enfermedad iliofemoral oclusiva .
Enfermedad de Alport. Anomalías del tracto urinario inferior.
Nefropatía lgA. Alteraciones psiquiátricas graves .
Lupus eritematoso sistémico. Recidiva histológica de la nefropatía original. Es poco frecuente

..... Nefroesclerosis.
Nefritis intersticial.
Pielonefritis .
Uropatía obstructiva.
(< 10%), pero debe considerarse. tal es el caso de la enfermedad
por depósito de cadenas ligeras, oxalosis y anticuerpos antimem-
brana basal glomerular.

..,. Los mejores receptores son individuos jóvenes cuyo fallo renal no se deba
a una enfermedad sistémica que pueda dañar el riñón trasplantado o cau-
sar la muerte por causas extrarrenales. Generalmente, se suele mantener
al receptor en tratamiento con diálisis durante un cierto tiempo previo al
11.3. Complicaciones
Las complicaciones que se pueden presentar son las siguientes:
Rechazo (Tabla 11.1 ).

,.,. trasplante. Recurrenda de la enfermedad en el riñón trasplantado.


Complicaciones técnicas. Complicaciones vasculares, hemorragia,
hipertensión por estenosis de la arteria renal, trombosis venosa , com-
plicaciones del tracto urinario {estenosis ureteral, refl ujo renourete-

,.
ral), necrosis tubular aguda, linfoceles y abscesos de pared.
Complicaciones no técnicas. Infecciones bacterianas y oportunistas
La nefrectomía laparoscópica para la donación es el gold standard y ha

.....
venido a aumentar la tasa de donador vivo. en relación con la inmunosupresión, hiperglucemias, complicaciones
gastrointestinales, hiperparotiroidismo y tumores (cáncer de piel y
de labios, carcinoma in situ de cérvix, linfomas no Hodgkin; guardan
relación con el tratamiento inmunosupresor) . Puede aparecer hiper-
11.2. Contraindicaciones tensión debida a enfermedad en los riñones originales, como conse-

,. cuencia de rechazo, por estenosis de la anastomosis de la arteria renal

..
El trasplante renal es un procedimiento multidisciplinario, involucra a urólo- o por toxicidad renal por ciclosporina.
gos, angiólogos, psiquiatras y anestesiólogos, entre otros, los cuales deben
valorar si el paciente es apto para recibir un trasplante .
Contraindicaciones absolutas:
Infección activa {cribado para VHB, VHC, VIH, CMV y TB).

....
~
Enfermedad maligna que no pueda ser erradicada.
Sospecha de no cumplimiento terapéutico del protocolo inmu-
nosupresor.
Glomerulonefritis activa .
En pacientes con diabetes insulinodependiente, se debe considerar un
trasplante renal y de páncreas. En pacientes con oxalosis, considerar
trasplante combinado de hígado y riñón.

.. •
Libro CTO de Medicina y Cirugía, 6.ª ed. ENARM

Re~hazo Inicio Patogenia Pat ología asociad a Tratamiento

Hipe r,1gudo

lv:.t del complemento


Daño enclotelial
Acel~rado • . Celular (+Acl Vasculitis necrotizante Bolos de esteroldes
Respuesta secundarla Ac monoclonales
aAg·HLA
Agudo Celular (+Ac) • Forma vascular: mediada por Ac Bolos de esteroldes Oa vascular suele ser resistente)
Infiltrado de Unfodtos Forma celular: tubulointerstlcial Ac monoclonales
Croni<. u Humoral y celular Intima arterial aumentada No hay; control de HTA
Atrofia tubular
Glomerulopatla
- - -- ---- -- ··· -·-·-·--- ··- ·--·--····-··
Tabla 11. 1. Rechazo en el trasplante renal

11.4. lnmunosupresión en Oclosporina A y tacrollmus (inhibidores de la callcrelna): su principal


mecanismo de acción es la inhibición de la activación de las células
trasplante T-CD4 ante estímulos antigénicos
Slrollmus (rapamidna) y everollmus (inhlbldores de mTOR): se pue-
El uso de fármacos inmunosupresores tiene como finalidad prevenir o con- den usar en la inmunosupresión primaria con o sin inhibidores de la
trolar la respuesta inmunológica del receptor contra el órgano trasplantado. calicreína.
para conseguir la máxima supervivencia del injerto y del paciente, con la
mejor calidad de vida . Los fármacos inmunosupresores se utilizan en dos
situaciones distintas: como inmunosupresión primaria o de base (terapia
de inducción + terapia de mantenimiento) y como tratamiento del rechazo
agudo. De entre las distintas clasificaciones de estos fármacos, expondre-
mos la que se basa en su mecanismo de acción:
• Corticoesteroldes: potente acción antiinflamatoria y capacidad-inmu-
nosupresora.

✓ Las causas más frecuentes de insuficiencia renal crónica son la diabetes ✓ El rechazo agudo produce oligurla, no poliuria.
mellitus y las glomerulonefritis.

✓ La clínica característica del rechazo agudo es: fiebre. hipertensión y dolor


en el área del injerto.

C.::>-sos Clí"",cos
Un paciente de 35 años con Insuficiencia renal crónica, secundaria a y disminución en la concentración urinaria de sodio. El diagnóstico más
plelonefrltis crónica, recibe un trasplante renal de cadáver con el que probable sería:
compartía dos identidades en A y B y una en DR. Recibe tratamiento in-
munosupresor con ciclosporina A y corticoides en dosis estándar. En el 1) Infección respiratoria.
posoperatorio inmediato se observa buena diuresis, y no es necesario 2) Pielonefritis aguda del injerto renal.
el tratamiento sustitutivo con hemodiálisis. En el quinto día de evolu- 3) Recidiva de su enfermedad renal.
ción, el paciente presenta fiebre de 38ºC, TA de 180/110 mmHg. oliguria 4) Rechazo agudo del injerto renal.


......
...... 12
....
..... Traumatismos
del aparato genitourinario

...... Los traumatismos renales se pueden clasificar en:


• Traumatismo renal menor (85%), Grados I y 11 de la clasificación de
Se han de repasar bien las Ideas clave. la American Association far the Surgery of Trauma (AAST). Incluyen la
contusión del parénqu ima (los más frecuentes), el hematoma capsular

.....
y las laceraciones corticales superficiales. Rara vez requieren explora-
ción quirúrgica.
Las lesiones del aparato genitourinario son frecuentes en los pacientes Traumatismo renal mayor (15%). Grados 111, IV y V de la AAST.
politraumatizados, así que, ante todo paciente con fracturas costales bajas, Laceraciones corticomedulares profundas que pueden afectar al
equimosis o masa en flancos, fracturas de las apófisis transversas, fractu- sistema colector, con extravasación de orina al espacio perirrenal.
ras de los cuerpos vertebrales y/o fracturas pélvicas, debe sospecharse una Se acompaña a menudo de hematomas retroperitoneales y perirre-
lesión de este tipo. Son más frecuentes en jóvenes (15-45 años) y con pre- nales.
dominio en varones (2:1). lesión vascular (1% de los traumatismos contusos) . Grado V de la
AAST.

12.1. Lesiones del riñón Grado Descripción

Contusión o hematoma subcapsular no expansivo


Son las lesiones más frecuentes del aparato urinario, siendo el mecanismo
más frecuente (80-85%) el traumatismo contuso directo en el abdomen, Hematoma perirrenal no expansivo. Laceración cortical < 1 cm sin
extravasación urinaria
flanco o región dorsal.
3 Laceración cortical> 1 cm con extravasación urinaria
Clasificacíón patológíca (Figura 12.1 y Tabla 12.1) 4 Laceración del tejido corticomedular hasta los sistemas colectores
o daño vascular (arteria renal segmentaría o daño a la vena con
hematoma contenido)

Gradol Gradoll Gradolll S Rlñón lacerado o daño al pedículo vascular (avulsión)

Tabla 12,1, Escala de severidad de daño renal

Clínica
La hematuria macroscópica o microscópica despué s de un trauma-
tismo indica lesión del aparato urinario, aunque no aparece en todos
los casos (p. ej ., ante lesiones del pedículo vascular o avulsión del uré-
ter). El grado de hematuria no siempre se corresponde con el grado de
la lesión. Otros síntomas y signos son : dolor abdominal o en un flanco,
equimosis en los flancos o cuadrantes superiores del abdomen, masa
palpable secundaria a hematoma retroperitoneal o a urinorna y disten-
sión abdominal.

En todo paciente que se sospeche de complicaciones (fiebre, dolor intenso


y constante, descenso de hematocrito), debe realizarse un estudio de ima-
gen de control.
Complicaciones inmediatas más frecuentes : hemorragia, extravasa-
ción urinaria que secundariamente da lugar a un absceso y septicemia .
Grado IV Grado V Complicaciones tardías más importantes: hipertensión, hidronefrosis,
fístula arteriovenosa, formación de cálculos, y pielonefritis y hemorra-
Figura 12.1. Clasificación del traumatismo renal de la AA'Sf
gia tardía .


Libro CTO de Medicina y Cirugía, 6.ª ed. ENARM

Diagnóstico sis y fibrosis reactiva, junto con hidronefrosis; si se extravasa hacia la cavidad
peritoneal, da lugar a una peritonitis aguda.
Es indispensable una buena evaluación física para determinar la severidad
de la lesión. Los signos vitales y los cambios en ellos pueden orientar ¡¡ la En las pruebas de laboratorio, si hay una lesión por traumatismo externo,
severidad de la lesión. Se debe realizar un examen general de orina en busca aparece hematuria microscópica en el 90% de los casos. El diagnóstico se
de hematuria, así como una biometría hemática (sobre todo, basarse en el realiza mediante una UIV o uretrografía retrógrada . En el periodo posope-
hematocrito) y una urea y creatinina. El objetivo de los estudios de ima- ratorio inmediato, la urografía es el mejor método para descartar la lesión
gen es medir la severidad de la lesión, y deben realizarse cuando haya una ureteral o bien una tomografía con contraste intravenoso en busca de la
hematuria macroscópica o microscópica e hipotensión (< 90 mmHg), o si el extravasación del medio de contraste en la fase de eliminación.
mecanismo de lesión sugiere un traumatismo severo. La urotomograffa es
el estudio de elección en pacientes hemodinámicamente estables, ya que Tratamiento
permite evaluar y clasificar el tipo de lesión; y en pacientes inestables, aún
se utiliza la pielografía intraoperatoria para evaluar la integridad y funciona- La primera medida es la derivación urinaria mediante nefrostomía . Requ ie-
lidad del riñón contralateral. ren tratamiento quirúrgico inmediato.
Lesión del tercio Inferior del uréter. El procedimiento de elección
Si no se determina en su totalidad la extensión de la lesión, puede asociarse es la reimplantación en la vejiga . La uretero-ureterostomía primaria
una nefrotomografía o una TC, que además muestra el estado de los órga- puede indicarse si hay un corte transversal del uréter. Se usa transure-
nos vecinos tero-ureterostomía, si hay urinoma extenso e infección pélvica.
Lesión del tercio medio y superior. Uretero-ureterostomía primaria o
La arteriografía se indica cuando el riñón no se observa bien en la uroto- sustitución ureteral (Tabla 12.2).
mografía o cuando se sospecha de lesión vascular y con fines terapéuticos
como la embolización selectiva. Las causas más importantes que no per- Es frecuente dejar un catéter de doble-J transanastomótico, que se retira
miten la adecuada observación en la urotomograffa son: rotura total del después de 3-4 semanas de cicatrización, con objeto de conservar el uréter
pedículo, trombosis arterial. contusión intensa que causa espasmo vascular en una posición adecuada con un calibre constante, impedir la extravasa-
y ausencia de riñón . ción urinaria y conservar la desviación urinaria.

Los exámenes con isótopos, en la evaluación de urgencia, son menos sensi- Sitio de lesión Opciones de manejo
bles que la arteriografía o la TC.
Ur~tt?ro prox tmi,I Uretero-ureteroanastomosi5
Transuretero-ureteroanastomosls·
Tratamiento Uretero-callcoanastomosis

Urt-tero medio Uretero-ureteroanastomosis


Las medidas terapéuticas iniciales deben dirigirse a la estabilización hemo- Transuretero-ureteroanastomosis
dinámica y a la reanimación completa del paciente, con tratamiento del Reimplante ureteral
shock, control de la hemorragia y evaluación de las lesiones concurrentes.
Uretero distal Reimplante ureteral
Pacientes con trauma renal 1-IV con estabilidad hemodinámica deben ser Reimplante ureteral con técnica psoas Hitch
manejados de forma conservadora, con reposo absoluto y monitorización
Completo Injerto con neon
estrecha de signos vitales y niveles de hemoglobina y hematocrito.
Autotrasplante

El tratamiento quirúrgico está indicado en: Tabla 12.2. Manejo de la lesión ureteral según sitio de lesión
Todo paciente inestable (hemorragia retroperitoneal, lesión del
pedículo renal, extravasación urinaria).
Traumatismos renales por penetración (salvo si se ha podido deter- 12.3. Lesiones de la vejiga
minar el grado de la lesión y resulta ser una lesión menor del parén-
quima sin extravasación urinaria) . Incluso traumatismos grado IV si el Las lesiones de vejiga se presentan frecuentemente debido a fuerzas
paciente se encuentra estable pueden ser manejados de forma con- externas y asociadas a fracturas pélvicas. La lesión iatrógena ocurre en
servadora. cirugías pélvicas, ginecológicas, herniorrafias e intervenciones transure-
Tratamiento de complicaciones como el urinoma retroperitoneal o el trales.
absceso perirrenal, la hipertensión maligna que requiere reparación
vascular o nefrectomía, y algunos casos de hidronefrosis. La rotura puede ser:
Extraperitoneal. Perforaciones por fragmentos de fracturas pélvicas.
Son las más frecuentes.
12.2. Lesiones del uréter lntraperitoneal. Golpes directos cuando la vejiga está llena.
Mixta.
Son raras, pero pueden ocurrir durante el curso de intervenciones quirúrgi-
cas pélvicas, por heridas de balas, desaceleraciones rápidas en accidentes, Clínicamente, pueden manifestarse con dificultad para la micción, hema-
manipulaciones endoscópicas de cálculos o resecciones transuretrales. La turia macroscópica, dolor pélvico o en hemiabdomen inferior, abdomen
ligadura del uréter conduce a la aparición de hidronefrosis con fiebre, dolor agudo (indica rotura intraperitoneal). Como complicación tardía, puede
en flanco, náuseas, vómitos e íleo, y si es bilateral, anuria. Si lo que ocurre es aparecer una incontinencia parcial en las lesiones que se extienden hasta
extravasación, se forma un urinoma que secundariamente provoca esteno- el cuello vesical.


12. Traumatismos del aparato genitourinario I UR

La cistograña es el estudio de elecc ión, permite identificar una ruptura


vesical y diferenciar si esta es intraperitoneal (observando el medio de
contraste en el contorno de las asas) o extraperitoneal (con la "imagen de
llamas" ). La cistoscopia no está indicada por la poca visibilidad secundaria
al sangrado en la mayor/a de los casos, y por lo invasiva de la misma.

Tratamiento
Se tratan mediante cateterización de 4-6 semanas, posterior a ello se retira
y se realiza nueva cistograña para corroborar el selle de la lesión. De no ser
pos ible la cateterización uretral, la cistostom ía suprapúbica es una opción
vi able de tratamiento.

12.4. Lesiones de la uretra Figura 12.2. Fractura de pene a la exploración física

Las lesiones de la uretra son poco frecuentes, más habituales en varones


y generalmente asociadas a fracturas pélvicas y a contusiones directas. Se 12.6. Lesiones de los testículos
consl aera uretra posterior la porción proximal al diafragma urogenital; y
uretra anterior, la distal. Para definir el daño, se realiza ultrasonografía. Si hay rotura, se trata quírúr-
glcamente . En el resto de los casos, el tratamiento es conservador.
Lesiones de la uretra posterior
(prostática y membranosa) A modo de resumen de todo lo anteriormente expuesto, la Figura 12.3
recoge los procedimientos generales de actuación frente a los traumatis-
mos urológicos.
La uretra membranosa se lesiona con mayor frecuencia. Los pacientes
aquejan dolor abdom inal bajo e incapacidad para la micción . El signo más
Sospecha de traumatismo urológico
importante es la presencia de sangre en el meato de la uretra (uretrorrªgia) .
En un tacto rectal, puede revelarse la presencia de hematoma pélvico y des-
plazamiento de la próstata hacia arriba .
Tracto inferior Tracto superior
(uretrorragia) (hematuria)
La prueba diagnóstica más importante es la uretrograña; cateterización y
ureteroscopia no deben realizarse porque conllevan un alto riesgo de pro-
Uretrografía
ducir hematoma e infección y daño más amplio de los desgarros parcia-
les de la uretra. Entre las complicaciones, destacan estenosis, impotencia
sexual e incontinencia. Su tratamiento es con cistostomia, seguida de cirugía Normal
diferida.

Sondaje vesical
Lesiones de la uretra anterior
(pendular y bulbar)
Sospecha lesión vesical
(si hay hematuria}
Generalmente, hay antecedente s de caída o maniobras con instrumen- Evaluar según Arteriografía
Cistografía(+250 mi) grado
ta ción. Se manifiestan con hemorragia y dolor en el pe ri né, pud iendo
existi r infección por extrava sación y estenosis tardía . No debe intentarse
pa sar catéter uretral y debe evitarse la micción ha sta descartar la exi s-
Rotura
tencia de extravasación . Se diagnostican mediante uretrografía retró- extraperitoneal
grada . (lo más frecuente)

Tratamiento Cirugía Sondaje


reparadora
Si existe laceración, debe realizarse cistostomía suprapúbica .
figura 12 .3. Manejo de los traumatismos urológicos

12.5. Lesiones del pene


Durante las relaciones sexuales, puede ocurrir rotura de la túnica albugínea
(fractura de pene). Provoca dolor y hematoma, y precisa tratamiento qui-
rúrgico (Figura 12.2).


Libro CTO de Medicina y Cirugía, 6.ª ed. ENARM

✓ La hematuria macroscópica o microscópica después de un traumatismo ✓ Requieren tratamiento quirúrgico los pacientes inestables, los traumatis-
indica lesión del aparato urinario. mos renales por penetración con extravasación urinaria, urinomas retro-
peritoneales o abscesos perirrenales.
✓ Para determinar el grado de lesión renal y su función, se puede realizar
una urografla. En el paciente politraumatlzado, puede ser más útil la ✓ Las lesiones vesicales pueden ser extraperitoneales, que al igual que las
TC. lesiones de uretra, se asocian a fracturas pélvicas; e intraperitoneales,
producidas por golpes directos cuando la vejiga está llena.

Paciente que, tras una caída a horcajadas, refiere sangrado por el meato 2) Es una lesión frecuente del ligamento del pene que no impide continuar
uretral con dificultad para la micción. Presenta un hematoma perineal Im- con la actividad sexual.
portante, ¿Cuál es la actitud que le parece más correcta? 3) Acudir inmediatamente a Urgencias ya que se sospecha una fractura de
pene.
1) Debe realizarse sondaje inmediato para prevenir complicaciones. 4) En su mayoría resuelve con compresión local y analgésicos.
2) Habrá que realizar TC abdominal para descartar lesiones asociadas,
como en cualquier lesión de uretra posterior. Paciente que ha presentado una calda desde 4 metros de altura mientras
3) Debe realizarse reconstrucción quirúrgica inmediata en caso de una ro- limpiaba unos cristales. Tras valoración y estabilización iniciales, se decide
tura completa. su traslado, durante el cual el paciente presenta una micción hematúrica.
4) Debe realizarse uretrografía retrógrada para valorar la gravedad de la ¿Cuál es la actitud que le parece más correcta?
lesión.
1) Se debe probablemente a una contusión vesical y no debe dársele mayor
Paciente masculino de 21 afias refiere que, al estar teniendo relaciones importancia.
sexuales, él y su pareja escuchan un chasquido. Al instante, nota aumento 2) Puede deberse a lesiones a varios niveles del aparato urinario y una TC
del volumen del pene, así como hematoma de este. ¿Cuál es la actitud que abdominal será útil para identificarla.
le parece más correcta? 3) Esto permite focalizar la lesión, descartando asl otras lesiones abdominales.
4) No se debe colocar una sonda vesical porque no ha presentado uretro-
1) Suspender la actividad y aplicar hielo local. rragia previa a la micción.


Recommended reading 1

An abdominal ultrasound is performed on a 42-year-old patient for ano- When complementary abdominal examinations are performed, particularly
ther reason. Durlng the study, an image such as the one shown below is ultrasounds, it is common far renal masses to be found incidentally, such
observed in the kidney. lndicate which one you consider to be the most that, currently, the number of renal carcinomas detected in a casual man-
likely diacnosis [Figure 1a]: ner exceeds the number of symptomatic renal carcinomas. However, in this
l. Hydronephrosis. case it is not a hypernephroma, but a completely benign cond ition : a simple
2. Hypernephroma. renal cyst (we may observe two of them in the ultrasound shown) (option
3. Simple renal cyst. no. 3 correct).
4. Renal abscess.
5. Anglomyollpoma. An abdom inal ultrasound makes it possible to distinguish a sol id mass from
a cystic mass. This feature is essential for the study of renal masses, since
most solid masses correspond to adenocarcinomas, w hereas cystic masses
are almost always simple renal cysts.

A simple renal cyst is a benign, non-neoplastic mass that contains fluid . it is


very frequent, since it affects over half the popuiation ove r 55 years of age.
In arder to make a definitive diagnosis, the following criteria must be met:
Imperceptible wall in the ultrasound.
• That the limits thereof are well-defined
Water density, without int erna! echoes.
Presence of posterior reinforcement in the ultrasound
• In the event that a CT oran intravenous urography is performed, there
should be no evidence of contrast being captured.

Fuente: García Macarrón J. Casos clínicos en imágenes. Madrid. CTO Edito-


rial, 2012.
Figure l a.
1

Recommended reading 2

A 35-year-old man vlslts the physlclan because he has palpated a mass in Testicular tumours are the most frequent malignant neoplasias in males
his left testlcle. He has no history of trauma or hlgh-rlsk sexual relatlons. between 20 and 35 years of age, except for leukaemias. The most frequent
The examlnation confirms the presence of a mass and the attached ultra- form of clinical presentation is as a scrotal mass.
sound Is performed. What Is the most adequate approach? [Figu re 2ai:
l. FNAC of the mass. A testicular ultrasound is a simple, reliable method used to distinguish
2. Surgical blopsy of the mass. between solid and cystic masses. lt is also very useful to determine whether
3. Inguinal orchlectomy, following collectlon of blood sample to deter- a mass is dependent on the testicle or the testicular annexes, as well as its
mine AFP and 13-hCG. intratesticular position. In the ultrasound shown, we observe a very intense
4. Transcrotal orchlectomy with hemiscrotal resection. destructuration of the testicular parenchyma, which has a clearly heteroge-
5. Monitorlng by ultrasound, slnce lt is a normal testicular IIN!ge. neous appearance. Below, we show the appearance of a normal testicular
ultrasound, such that you may appreciate the differences.

When, after the clinícal examínation and the ultrasound, the diagnosis is
still not clear, a transinguinal surgical examination should be performed. lf
this examination confirms the presence of a mass, the testicle should be
removed (answer n. 3 correct).

Overall, 70% of testicular tumours manífest sorne marker. lt is important to


determine these markers, not only for diagnostic purposes, but primarily
with a víew to the subsequent follow-up, sínce an increase in them could
warn us early on about a recurrence . For this reason, before and after the
orchíectomy we should measure the levels of hCG and AFP. Hígh concentra -
tions of hCG and AFP decrease accordíng to first-order kínetics; the half-life
of hCG is 24-36 hours and that of AFP is 5-7 days. These two markers should
be seríally analysed duríng and after the treatment. lf any or both of these
markers increase again or do not decrease accordíng to the expected half-
life, we should suspect tumour persístence or recurrence.

Fuente: García Macarrón J. Cosos clínicos en imágenes. Madrid. CTO Edito-


Figure 2a.
rial, 2012 .

Figure 2b-2c. To the right, normal testícle, the parenchyma whereof is visuallsed as homogeneous in the ultrasound. On the contrary, the left ímage
shows a destructured parenchyma, wíth heterogeneous echogenicíty and s~me calcification, which suggests malignancy


-
••
..
.....
.... Solucionario
Casos clínicos/Case Study

..•
.... Tema

3
. .
Caso chn1co

2
Respuesta
correcta

3
4
2
3
4
2
PHii@IIB
2
3
3
2

•..
4 4 2 4
2 5 3 3 3

..... 3
4
5
2
3
4
2
4

3
4

5
3
2
4

....
6 3 2 2 4
7 4 3 4 3 4
8 4 4 2 4

....
9 2 5 4 3
10 3 6 3 2 3

11 2 7 3 3 2
12 3 4 4 2

,._. 2
3
4

2
2
3 2
3
2
4

_. 4 4 4 4

,.
5 4 4 3
6 4 2 3 2

.... 7

8
9
10
4

2
3 2 3
4

2
3
4

3
2

_. 4 3

..,. 2
3
4
4
4
4
4 4

2
3

,.
5 2 2 3

6 4
7 3
8 3

~
~
• •

También podría gustarte